Vous êtes sur la page 1sur 129

anada vs Angara

Justiciable Question; Theory of Auto-Limitation; Declaration of


Principles and State Policies

TANADA VS ANGARA
G.R. No. 118295 May 2, 1997

Wigberto E. Tanada et al, in representation of various taxpayers and as non-governmental


organizations, petitioners,
vs.
EDGARDO ANGARA, et al, respondents.

Facts:
This is a case petition by Sen. Wigberto Tanada, together with other lawmakers, taxpayers, and various NGO’s
to nullify the Philippine ratification of the World Trade Organization (WTO) Agreement.

Petitioners believe that this will be detrimental to the growth of our National Economy and against to the
“Filipino First” policy. The WTO opens access to foreign markets, especially its major trading partners,
through the reduction of tariffs on its exports, particularly agricultural and industrial products. Thus, provides
new opportunities for the service sector cost and uncertainty associated with exporting and more investment in
the country. These are the predicted benefits as reflected in the agreement and as viewed by the signatory
Senators, a “free market” espoused by WTO.

Petitioners also contends that it is in conflict with the provisions of our constitution, since the said Agreement
is an assault on the sovereign powers of the Philippines because it meant that Congress could not pass
legislation that would be good for national interest and general welfare if such legislation would not conform
to the WTO Agreement.

Issues:

1. Whether or not the petition present a justiciable controversy.


2. Whether or not the provisions of the ‘Agreement Establishing the World Trade Organization and the
Agreements and Associated Legal Instruments included in Annexes one (1), two (2) and three (3) of that
agreement’ cited by petitioners directly contravene or undermine the letter, spirit and intent of Section 19,
Article II and Sections 10 and 12, Article XII of the 1987 Constitution.
3. Whether or not certain provisions of the Agreement unduly limit, restrict or impair the exercise of
legislative power by Congress.
4. Whether or not certain provisions of the Agreement impair the exercise of judicial power by this
Honorable Court in promulgating the rules of evidence.
5. Whether or not the concurrence of the Senate ‘in the ratification by the President of the Philippines of the
Agreement establishing the World Trade Organization’ implied rejection of the treaty embodied in the
Final Act.

Discussions:

 1987 Constitution states that Judicial power includes the duty of the courts of justice to settle actual
controversies involving rights which are legally demandable and enforceable, and to determine whether or
not there has been a grave abuse of discretion amounting to lack or excess of jurisdiction on the part of
any branch or instrumentality of the government.
 Although the Constitution mandates to develop a self-reliant and independent national economy controlled
by Filipinos, does not necessarily rule out the entry of foreign investments, goods and services. It
contemplates neither “economic seclusion” nor “mendicancy in the international community.” The WTO
itself has some built-in advantages to protect weak and developing economies, which comprise the vast
majority of its members. Unlike in the UN where major states have permanent seats and veto powers in
the Security Council, in the WTO, decisions are made on the basis of sovereign equality, with each
member’s vote equal in weight to that of any other. Hence, poor countries can protect their common
interests more effectively through the WTO than through one-on-one negotiations with developed
countries. Within the WTO, developing countries can form powerful blocs to push their economic agenda
more decisively than outside the Organization. Which is not merely a matter of practical alliances but a
negotiating strategy rooted in law. Thus, the basic principles underlying the WTO Agreement recognize
the need of developing countries like the Philippines to “share in the growth in international trade
commensurate with the needs of their economic development.”
 In its Declaration of Principles and State Policies, the Constitution “adopts the generally accepted
principles of international law as part of the law of the land, and adheres to the policy of peace, equality,
justice, freedom, cooperation and amity, with all nations. By the doctrine of incorporation, the country is
bound by generally accepted principles of international law, which are considered to be automatically part
of our own laws. A state which has contracted valid international obligations is bound to make in its
legislations such modifications as may be necessary to ensure the fulfillment of the obligations
undertaken. Paragraph 1, Article 34 of the General Provisions and Basic Principles of the Agreement on
Trade-Related Aspects of Intellectual Property Rights (TRIPS) may intrudes on the power of the Supreme
Court to promulgate rules concerning pleading, practice and procedures. With regard to Infringement of a
design patent, WTO members shall be free to determine the appropriate method of implementing the
provisions of TRIPS within their own internal systems and processes.
 The alleged impairment of sovereignty in the exercise of legislative and judicial powers is balanced by the
adoption of the generally accepted principles of international law as part of the law of the land and the
adherence of the Constitution to the policy of cooperation and amity with all nations. The Senate, after
deliberation and voting, voluntarily and overwhelmingly gave its consent to the WTO Agreement thereby
making it “a part of the law of the land” is a legitimate exercise of its sovereign duty and power.

Rulings:

1. In seeking to nullify an act of the Philippine Senate on the ground that it contravenes the Constitution, the
petition no doubt raises a justiciable controversy. Where an action of the legislative branch is seriously
alleged to have infringed the Constitution, it becomes not only the right but in fact the duty of the judiciary
to settle the dispute. As explained by former Chief Justice Roberto Concepcion, “the judiciary is the final
arbiter on the question of whether or not a branch of government or any of its officials has acted without
jurisdiction or in excess of jurisdiction or so capriciously as to constitute an abuse of discretion amounting
to excess of jurisdiction. This is not only a judicial power but a duty to pass judgment on matters of this
nature.”
2. While the Constitution indeed mandates a bias in favor of Filipino goods, services, labor and enterprises,
at the same time, it recognizes the need for business exchange with the rest of the world on the bases of
equality and reciprocity and limits protection of Filipino enterprises only against foreign competition and
trade practices that are unfair. In other words, the Constitution did not intend to pursue an isolationist
policy. It did not shut out foreign investments, goods and services in the development of the Philippine
economy. While the Constitution does not encourage the unlimited entry of foreign goods, services and
investments into the country, it does not prohibit them either. In fact, it allows an exchange on the basis of
equality and reciprocity, frowning only on foreign competition that is unfair.
3. By their inherent nature, treaties really limit or restrict the absoluteness of sovereignty. By their voluntary
act, nations may surrender some aspects of their state power in exchange for greater benefits granted by or
derived from a convention or pact. After all, states, like individuals, live with coequals, and in pursuit of
mutually covenanted objectives and benefits, they also commonly agree to limit the exercise of their
otherwise absolute rights. As shown by the foregoing treaties Philippines has entered, a portion of
sovereignty may be waived without violating the Constitution, based on the rationale that the Philippines
“adopts the generally accepted principles of international law as part of the law of the land and adheres to
the policy of cooperation and amity with all nations.”
4. The provision in Article 34 of WTO agreement does not contain an unreasonable burden, consistent as it is
with due process and the concept of adversarial dispute settlement inherent in our judicial system.
5. The assailed Senate Resolution No. 97 expressed concurrence in exactly what the Final Act required from
its signatories, namely, concurrence of the Senate in the WTO Agreement. Moreover, the Senate was well-
aware of what it was concurring in as shown by the members’ deliberation on August 25, 1994. After
reading the letter of President Ramos dated August 11, 1994, the senators of the Republic minutely
dissected what the Senate was concurring in.
Case Brief: Tanada v Angara
NOVEMBER 24, 2014JEFF REY

G.R. No. 118295 May 2, 1997


WIGBERTO E. TAÑADA et al, petitioners,
vs.

EDGARDO ANGARA, et al, respondents.


Facts:
Petitioners prayed for the nullification, on constitutional grounds, of the concurrence
of the Philippine Senate in the ratification by the President of the Philippines of the
Agreement Establishing the World Trade Organization (WTO Agreement, for brevity)
and for the prohibition of its implementation and enforcement through the release and
utilization of public funds, the assignment of public officials and employees, as well
as the use of government properties and resources by respondent-heads of various
executive offices concerned therewith.

They contended that WTO agreement violates the mandate of the 1987 Constitution to
“develop a self-reliant and independent national economy effectively controlled by
Filipinos x x x (to) give preference to qualified Filipinos (and to) promote the
preferential use of Filipino labor, domestic materials and locally produced goods” as
(1) the WTO requires the Philippines “to place nationals and products of member-
countries on the same footing as Filipinos and local products” and (2) that the WTO
“intrudes, limits and/or impairs” the constitutional powers of both Congress and the
Supreme Court.

Issue:
Whether provisions of the Agreement Establishing the World Trade Organization
unduly limit, restrict and impair Philippine sovereignty specifically the legislative
power which, under Sec. 2, Article VI, 1987 Philippine Constitution is ‘vested in the
Congress of the Philippines.

Held:
No, the WTO agreement does not unduly limit, restrict, and impair the Philippine
sovereignty, particularly the legislative power granted by the Philippine Constitution.
The Senate was acting in the proper manner when it concurred with the President’s
ratification of the agreement.

While sovereignty has traditionally been deemed absolute and all-encompassing on


the domestic level, it is however subject to restrictions and limitations voluntarily
agreed to by the Philippines, expressly or impliedly, as a member of the family of
nations. Unquestionably, the Constitution did not envision a hermit-type isolation of
the country from the rest of the world. In its Declaration of Principles and State
Policies, the Constitution “adopts the generally accepted principles of international
law as part of the law of the land, and adheres to the policy of peace, equality, justice,
freedom, cooperation and amity, with all nations.” By the doctrine of incorporation,
the country is bound by generally accepted principles of international law, which are
considered to be automatically part of our own laws. One of the oldest and most
fundamental rules in international law is pacta sunt servanda — international
agreements must be performed in good faith. “A treaty engagement is not a mere
moral obligation but creates a legally binding obligation on the parties x x x. A state
which has contracted valid international obligations is bound to make in its
legislations such modifications as may be necessary to ensure the fulfillment of the
obligations undertaken.”

By their inherent nature, treaties really limit or restrict the absoluteness of


sovereignty. By their voluntary act, nations may surrender some aspects of their state
power in exchange for greater benefits granted by or derived from a convention or
pact. After all, states, like individuals, live with coequals, and in pursuit of mutually
covenanted objectives and benefits, they also commonly agree to limit the exercise of
their otherwise absolute rights. Thus, treaties have been used to record agreements
between States concerning such widely diverse matters as, for example, the lease of
naval bases, the sale or cession of territory, the termination of war, the regulation of
conduct of hostilities, the formation of alliances, the regulation of commercial
relations, the settling of claims, the laying down of rules governing conduct in peace
and the establishment of international organizations. The sovereignty of a state
therefore cannot in fact and in reality be considered absolute. Certain restrictions enter
into the picture: (1) limitations imposed by the very nature of membership in the
family of nations and (2) limitations imposed by treaty stipulations. As aptly put by
John F. Kennedy, “Today, no nation can build its destiny alone. The age of self-
sufficient nationalism is over. The age of interdependence is here.”

The WTO reliance on “most favored nation,” “national treatment,” and “trade without
discrimination” cannot be struck down as unconstitutional as in fact they are rules of
equality and reciprocity that apply to all WTO members. Aside from envisioning a
trade policy based on “equality and reciprocity,” the fundamental law encourages
industries that are “competitive in both domestic and foreign markets,” thereby
demonstrating a clear policy against a sheltered domestic trade environment, but one
in favor of the gradual development of robust industries that can compete with the
best in the foreign markets. Indeed, Filipino managers and Filipino enterprises have
shown capability and tenacity to compete internationally. And given a free trade
environment, Filipino entrepreneurs and managers in Hongkong have demonstrated
the Filipino capacity to grow and to prosper against the best offered under a policy of
laissez faire.
WHEREFORE, the petition is DISMISSED for lack of merit.
[G.R. No. 132451. December 17, 1999]

CONGRESSMAN ENRIQUE T. GARCIA, petitioner, vs. HON. RENATO C.


CORONA, in his capacity as the Executive Secretary, HON.
FRANCISCO VIRAY, in his capacity as the Secretary of Energy,
CALTEX PHILIPPINES INC., PILIPINAS SHELL PETROLEUM
CORP. and PETRON CORP., respondents.

DECISION
YNARES-SANTIAGO, J.:

On November 5, 1997, this Court in Tatad v. Secretary of the Department of


Energy and Lagman, et al., v. Hon. Ruben Torres, et al.,[1] declared Republic Act No. 8180,
entitled An Act Deregulating the Downstream Oil Industry and For Other Purposes,
unconstitutional, and its implementing Executive Order No. 392 void.
R.A. 8180 was struck down as invalid because three key provisions intended to promote free
competition were shown to achieve the opposite result. More specifically, this Court ruled that its
provisions on tariff differential, stocking of inventories, and predatory pricing inhibit fair
competition, encourage monopolistic power, and interfere with the free interaction of the market
forces.
While R.A. 8180 contained a separability clause, it was declared unconstitutional in its
entirety since the three (3) offending provisions so permeated the law that they were so
intimately the esse of the law. Thus, the whole statute had to be invalidated.
As a result of the Tatad decision, Congress enacted Republic Act No. 8479, a new
deregulation law without the offending provisions of the earlier law. Petitioner Enrique T.
Garcia, a member of Congress, has now brought this petition seeking to declare Section 19
thereof, which sets the time of full deregulation, unconstitutional. After failing in his attempts to
have Congress incorporate in the law the economic theory he espouses, petitioner now asks us, in
the name of upholding the Constitution, to undo a violation which he claims Congress has
committed.
The assailed Section 19 of R.A. 8479 states in full:

SEC. 19. Start of Full Deregulation. --- Full deregulation of the Industry shall start
five (5) months following the effectivity of this Act: Provided, however, That when
the public interest so requires, the President may accelerate the start of full
deregulation upon the recommendation of the DOE and the Department of Finance
(DOF) when the prices of crude oil and petroleum products in the world market are
declining and the value of the peso in relation to the US dollar is stable, taking into
account relevant trends and prospects; Provided, further, That the foregoing provision
notwithstanding, the five (5)-month Transition Phase shall continue to apply to LPG,
regular gasoline and kerosene as socially-sensitive petroleum products and said
petroleum products shall be covered by the automatic pricing mechanism during the
said period.

Upon the implementation of full deregulation as provided herein, the Transition Phase
is deemed terminated and the following laws are repealed:

a) Republic Act No. 6173, as amended;

b) Section 5 of Executive Order No. 172, as amended;

c) Letter of Instruction No. 1431, dated October 15, 1984;

d) Letter of Instruction No. 1441, dated November 20, 1984, as amended;

e) Letter of Instruction No. 1460, dated May 9, 1985;

f) Presidential Decree No. 1889; and

g) Presidential Decree No. 1956, as amended by Executive Order No. 137:

Provided, however, That in case full deregulation is started by the President in the
exercise of the authority provided in this Section, the foregoing laws shall continue to
be in force and effect with respect to LPG, regular gasoline and kerosene for the rest
of the five (5)-month period.

Petitioner contends that Section 19 of R.A. 8479, which prescribes the period for the
removal of price control on gasoline and other finished products and for the full deregulation of
the local downstream oil industry, is patently contrary to public interest and therefore
unconstitutional because within the short span of five months, the market is still dominated and
controlled by an oligopoly of the three (3) private respondents, namely, Shell, Caltex and Petron.
The objective of the petition is deceptively simple. It states that if the constitutional mandate
against monopolies and combinations in restraint of trade[2] is to be obeyed, there should be
indefinite and open-ended price controls on gasoline and other oil products for as long as
necessary. This will allegedly prevent the Big 3 --- Shell, Caltex and Petron --- from price-fixing
and overpricing. Petitioner calls the indefinite retention of price controls as partial deregulation.
The grounds relied upon in the petition are:
A.

SECTION 19 OF R.A. NO. 8479 WHICH PROVIDES FOR FULL


DEREGULATION FIVE (5) MONTHS OR EARLIER FOLLOWING THE
EFFECTIVITY OF THE LAW, IS GLARINGLY PRO-OLIGOPOLY, ANTI-
COMPETITION AND ANTI-PEOPLE, AND IS THEREFORE PATENTLY
UNCONSTITUTIONAL FOR BEING IN GROSS AND CYNICAL
CONTRAVENTION OF THE CONSTITUTIONAL POLICY AND COMMAND
EMBODIED IN ARTCLE XII, SECTION 19 OF THE 1987 CONSTITUTION
AGAINST MONOPOLIES AND COMBINATIONS IN RESTRAINT OF TRADE.
B.

SAID SECTION 19 OF R.A. No. 8479 IS GLARINGLY PRO-OLIGOPOLY, ANTI-


COMPETITION AND ANTI-PEOPLE, FOR THE FURTHER REASON THAT IT
PALPABLY AND CYNICALLY VIOLATES THE VERY OBJECTIVE AND
PURPOSE OF R.A. NO. 8479, WHICH IS TO ENSURE A TRULY COMPETITIVE
MARKET UNDER A REGIME OF FAIR PRICES.
C.

SAID SECTION 19 OF R.A. No. 8479, BEING GLARINGLY PRO-OLIGOPOLY,


ANTI-COMPETITION AND ANTI-PEOPLE, BEING PATENTLY
UNCONSTITUTIONAL AND BEING PALPABLY VIOLATIVE OF THE LAWS
POLICY AND PURPOSE OF ENSURING A TRULY COMPETITIVE MARKET
UNDER A REGIME OF FAIR PRICES, IS A VERY GRAVE AND GRIEVOUS
ABUSE OF DISCRETION ON THE PART OF THE LEGISLATIVE AND
EXECUTIVE BRANCHES OF GOVERNMENT.
D.

PREMATURE FULL DEREGULATION UNDER SECTION 19 OF R.A. NO. 8479


MAY AND SHOULD THEREFORE BE DECLARED NULL AND VOID EVEN
AS THE REST OF ITS PROVISIONS REMAIN IN FORCE, SUCH AS THE
TRANSITION PHASE OR PARTIAL DEREGULATION WITH PRICE
CONTROLS THAT ENSURES THE PROTECTION OF THE PUBLIC INTEREST
BY PREVENTING THE BIG 3 OLIGOPOLYS PRICE-FIXING AND
OVERPRICING.[3]

The issues involved in the deregulation of the downstream oil industry are of paramount
significance. The ramifications, international and local in scope, are complex. The impact on the
nations economy is pervasive and far-reaching. The amounts involved in the oil business are
immense. Fluctuations in the supply and price of oil products have a dramatic effect on economic
development and public welfare.As pointed out in the Tatad decision, few cases carry a
surpassing importance on the daily life of every Filipino. The issues affect everybody from the
poorest wage-earners and their families to the richest entrepreneurs, from industrial giants to
humble consumers.
Our decision in this case is complicated by the unstable oil prices in the world market. Even
as this case is pending, the price of OPEC oil is escalating to record levels. We have to
emphasize that our decision has nothing to do with worldwide fluctuations in oil prices and the
counter-measures of Government each time a new development takes place.
The most important part of deregulation is freedom from price control. Indeed, the free play
of market forces through deregulation and when to implement it represent one option to solve the
problems of the oil-consuming public. There are other considerations which may be taken into
account such as the reduction of taxes on oil products, the reinstitution of an Oil Price
Stabilization Fund, the choice between government subsidies taken from the regular taxpaying
public on one hand and the increased costs being shouldered only by users of oil products on the
other, and most important, the immediate repeal of the oil deregulation law as wrong
policy. Petitioner wants the setting of prices to be done by Government instead of being
determined by free market forces. His preference is continued price control with no fixed end in
sight. A simple glance at the factors surrounding the present problems besetting the oil industry
shows that they are economic in nature.
R.A. 8479, the present deregulation law, was enacted to implement Article XII, Section 19
of the Constitution which provides:

The State shall regulate or prohibit monopolies when the public interest so
requires. No combinations in restraint of trade or unfair competition shall be allowed.

This is so because the Government believes that deregulation will eventually prevent
monopoly. The simplest form of monopoly exists when there is only one seller or producer of a
product or service for which there are no substitutes. In its more complex form, monopoly is
defined as the joint acquisition or maintenance by members of a conspiracy, formed for that
purpose, of the power to control and dominate trade and commerce in a commodity to such an
extent that they are able, as a group, to exclude actual or potential competitors from the field,
accompanied with the intention and purpose to exercise such power.[4]
Where two or three or a few companies act in concert to control market prices and resultant
profits, the monopoly is called an oligopoly or cartel. It is a combination in restraint of trade.
The perennial shortage of oil supply in the Philippines is exacerbated by the further fact that
the importation, refining, and marketing of this precious commodity are in the hands of a cartel,
local but made up of foreign-owned corporations. Before the start of deregulation, the three
private respondents controlled the entire oil industry in the Philippines.
It bears reiterating at the outset that the deregulation of the oil industry is a policy
determination of the highest order. It is unquestionably a priority program of Government. The
Department of Energy Act of 1992[5] expressly mandates that the development and updating of
the existing Philippine energy program shall include a policy direction towards deregulation of
the power and energy industry.
Be that as it may, we are not concerned with whether or not there should be
deregulation. This is outside our jurisdiction. The judgment on the issue is a settled matter and
only Congress can reverse it.Rather, the question that we should address here is --- are the
method and the manner chosen by Government to accomplish its cherished goal offensive to the
Constitution? Is indefinite price control in the manner proposed by petitioner the only feasible
and legal way to achieve it?
Petitioner has taken upon himself a most challenging task. Unquestionably, the direction
towards which the nations efforts at economic and social upliftment should be addressed is a
function of Congress and the President. In the exercise of this function, Congress and the
President have obviously determined that speedy deregulation is the answer to the acknowledged
dominion by oligopolistic forces of the oil industry. Thus, immediately after R.A. 8180 was
declared unconstitutional in the Tatad case, Congress took resolute steps to fashion new
legislation towards the objective of the earlier law. Invoking the Constitution, petitioner now
wants to slow down the process.
While the Court respects the firm resolve displayed by Congress and the President, all
departments of Government are equally bound by the sovereign will expressed in the commands
of the Constitution. There is a need for utmost care if this Court is to faithfully discharge its
duties as arbitral guardian of the Constitution. We cannot encroach on the policy functions of the
two other great departments of Government. But neither can we ignore any overstepping of
constitutional limitations. Locating the correct balance between legality and policy,
constitutional boundaries and freedom of action, and validity and expedition is this Courts
dilemma as it resolves the legitimacy of a Government program aimed at giving every Filipino a
more secure, fulfilling and abundant life.
Our ruling in Tatad is categorical that the Constitutions Article XII, Section 19, is anti-trust
in history and spirit. It espouses competition. We have stated that only competition which is fair
can release the creative forces of the market. We ruled that the principle which underlies the
constitutional provision is competition. Thus:

Section 19, Article XII of our Constitution is anti-trust in history and in spirit. It
espouses competition. The desirability of competition is the reason for the prohibition
against restraint of trade, the reason for the interdiction of unfair competition, and the
reason for regulation of unmitigated monopolies. Competition is thus the underlying
principle of section 19, Article XII of our Constitution which cannot be violated by
R.A. No. 8180. We subscribe to the observation of Prof. Gellhorn that the objective of
anti-trust law is to assure a competitive economy, based upon the belief that through
competition producers will strive to satisfy consumer wants at the lowest price with
the sacrifice of the fewest resources. Competition among producers allows consumers
to bid for goods and services, and thus matches their desires with societys opportunity
costs. He adds with appropriateness that there is a reliance upon the operation of the
market system (free enterprise) to decide what shall be produced, how resources shall
be allocated in the production process, and to whom the various products will be
distributed. The market system relies on the consumer to decide what and how much
shall be produced, and on competition, among producers to determine who will
manufacture it.[6]

In his recital of the antecedent circumstances, petitioner repeats in abbreviated form the
factual findings and conclusions which led the Court to declare R.A. 8180 unconstitutional. The
foreign oligopoly or cartel formed by respondents Shell, Caltex and Petron, their indulging in
price-fixing and overpricing, their blockade tactics which effectively obstructed the entry of
genuine competitors, the dangers posed by the oil cartel to national security and economic
development, and other prevailing sentiments are stated as axiomatic truths. They are repeated in
capsulized context as the current background facts of the present petition.
The empirical existence of this deplorable situation was precisely the reason why Congress
enacted the oil deregulation law. The evils arising from conspiratorial acts of monopoly are
recognized as clear and present. But the enumeration of the evils by our Tatad decision was not
for the purpose of justifying continued government control, especially price control. The
objective was, rather, the opposite. The evils were emphasized to show the need for free
competition in a deregulated industry. And to be sure, the measures to address these evils are for
Congress to determine, but they have to meet the test of constitutional validity.
The Court respects the legislative finding that deregulation is the policy answer to the
problems. It bears stressing that R.A. 8180 was declared invalid not because deregulation is
unconstitutional. The law was struck down because, as crafted, three key provisions plainly
encouraged the continued existence if not the proliferation of the constitutionally proscribed evils
of monopoly and restraint of trade.
In sharp contrast, the present petition lacks a factual foundation specifically highlighting the
need to declare the challenged provision unconstitutional. There is a dearth of relevant, reliable,
and substantial evidence to support petitioners theory that price control must continue even as
Government is trying its best to get out of regulating the oil industry. The facts of the petition
are, in the main, a general dissertation on the evils of monopoly.
Petitioner overlooks the fact that Congress enacted the deregulation law exactly because of
the monopoly evils he mentions in his petition. Congress instituted the lifting of price controls in
the belief that free and fair competition was the best remedy against monopoly power. In other
words, petitioners facts are also the reasons why Congress lifted price controls and why the
President accelerated the process.The facts adduced in favor of continued and indefinite price
control are the same facts which supported what Congress believes is an exercise of wisdom and
discretion when it chose the path of speedy deregulation and rejected Congressman Garcias
economic theory.
The petition states that it is using the very thoughts and words of the Court in
its Tatad decision. Those thoughts and words, however, were directed against the tariff
differential, the inventory requirement, and predatory pricing, not against deregulation as a
policy and not against the lifting of price controls.
A dramatic, at times expansive and grandiloquent, reiteration of the same background
circumstances narrated in Tatad does not squarely sustain petitioners novel thesis that there can
be deregulation without lifting price controls.
Petitioner may call the industry subject to price controls as deregulated. In enacting the
challenged provision, Congress, on the other hand, has declared that any industry whose prices
and profits are fixed by government authority remains a highly regulated one.
Petitioner, therefore, engages in a legal paradox. He fails to show how there can be
deregulation while retaining government price control. Deregulation means the lifting of control,
governance and direction through rule or regulation. It means that the regulated industry is freed
from the controls, guidance, and restrictions to which it used to be subjected. The use of the word
partial to qualify deregulation is sugar-coating. Petitioner is really against deregulation at this
time.
Petitioner states that price control is good. He claims that it was the regulation of the
importation of finished oil products which led to the exit of competitors and the consolidation
and dominion of the market by an oligopoly, not price control. Congress and the President think
otherwise.
The argument that price control is not the villain in the intrusion and growth of monopoly
appears to be pure theory not validated by experience. There can be no denying the fact that the
evils mentioned in the petition arose while there was price control. The dominance of the so-
called Big 3 became entrenched during the regime of price control. More importantly, the
ascertainment of the cause and the method of dismantling the oligopoly thus created are a matter
of legislative and executive choice. The judicial process is equipped to handle legality but not
wisdom of choice and the efficacy of solutions.
Petitioner engages in another contradiction when he puts forward what he calls a self-
evident truth. He states that a truly competitive market and fair prices cannot be legislated into
existence. However, the truly competitive market is not being created or fashioned by the
challenged legislation. The market is simply freed from legislative controls and allowed to grow
and develop free from government interference. R.A. 8479 actually allows the free play of
supply and demand to dictate prices. Petitioner wants a government official or board to continue
performing this task. Indefinite and open-ended price control as advocated by petitioner would
be to continue a regime of legislated regulation where free competition cannot possibly
flourish. Control is the antithesis of competition. To grant the petition would mean that the
Government is not keen on allowing a free market to develop. Petitioners self-evident truth thus
supports the validity of the provision of law he opposes.
New players in the oil industry intervened in this case. According to them, it is the free
market policy and atmosphere of deregulation which attracted and brought the new participants,
themselves included, into the market. The intervenors express their fear that this Court would
overrule legislative policy and replace it with petitioners own legislative program.
The factual allegations of the intervenors have not been refuted and we see no reason to
doubt them. Their argument that the co-existence of many viable rivals create free market
conditions induces competition in product quality and performance and makes available to
consumers an expanded range of choices cannot be seriously disputed.
On the other hand, the pleadings of public and private respondents both put forth the
argument that the challenged provision is a policy decision of Congress and that the wisdom of
the provision is outside the authority of this Court to consider. We agree. As we have ruled
in Morfe v. Mutuc[7]:

(I)t is well to remember that this Court, in the language of Justice Laurel, does not
pass upon question or wisdom, justice or expediency of legislation. As expressed by
Justice Tuason: It is not the province of the courts to supervise legislation and keep it
within the bounds of propriety and common sense. That is primarily and exclusively a
legislative concern. There can be no possible objection then to the observation of
Justice Montemayor: As long as laws do not violate any Constitutional provision, the
Courts merely interpret and apply them regardless of whether or not they are wise or
salutary. For they, according to Justice Labrador, are not supposed to override
legitimate policy and x x x never inquire into the wisdom of the law.

It is thus settled, to paraphrase Chief Justice Concepcion in Gonzales v. Commission


on Elections, that only congressional power or competence, not the wisdom of the
action taken, may be the basis for declaring a statute invalid. This is as it ought to
be. The principle of separation of powers has in the main wisely allocated the
respective authority of each department and confined its jurisdiction to such a
sphere. There would then be intrusion not allowable under the Constitution if on a
matter left to the discretion of a coordinate branch, the judiciary would substitute its
own. If there be adherence to the rule of law, as there ought to be, the last offender
should be the courts of justice, to which rightly litigants submit their controversy
precisely to maintain unimpaired the supremacy of legal norms and prescriptions.The
attack on the validity of the challenged provision likewise insofar as there may be
objections, even if valid and cogent, on its wisdom cannot be sustained.

In this petition, Congressman Garcia seeks to revive the long settled issue of the timeliness
of full deregulation, which issue he had earlier submitted to this Court by way of a Partial
Motion for Reconsideration in the Tatad case. In our Resolution dated December 3, 1997, which
has long become final and executory, we stated:

We shall first resolve petitioner Garcias linchpin contention that the full deregulation
decreed by R.A. No. 8180 to start at the end of March 1997 is unconstitutional. For
prescinding from this premise, petitioner suggests that we simply go back to the
transition period, price control will be revived through the automatic pricing
mechanism based on Singapore Posted Prices. The Energy Regulatory
Board x xx would play a limited and ministerial role of computing the monthly price
ceiling of each and every petroleum fuel product, using the automatic pricing
formula. While the OPSF would return, this coverage would be limited to monthly
price increases in excess of P0.50 per liter.

We are not impressed by petitioner Garcias submission. Petitioner has no basis in


condemning as unconstitutional per se the date fixed by Congress for the beginning of
the full deregulation of the downstream oil industry. Our Decision merely faulted the
Executive for factoring the depletion of OPSF in advancing the date of full
deregulation to February 1997. Nonetheless, the error of the Executive is now a non-
issue for the full deregulation set by Congress itself at the end of March 1997 has
already come to pass. March 1997 is not an arbitrary date. By that date, the transition
period has ended and it was expected that the people would have adjusted to the role
of market forces in shaping the prices of petroleum and its products. The choice of
March 1997 as the date of full deregulation is a judgment of Congress and its
judgment call cannot be impugned by this Court.[8]

Reduced to its basic arguments, it can be seen that the challenge in this petition is not
against the legality of deregulation. Petitioner does not expressly challenge deregulation. The
issue, quite simply, is the timeliness or the wisdom of the date when full deregulation should be
effective.
In this regard, what constitutes reasonable time is not for judicial determination. Reasonable
time involves the appraisal of a great variety of relevant conditions, political, social and
economic. They are not within the appropriate range of evidence in a court of justice. It would be
an extravagant extension of judicial authority to assert judicial notice as the basis for the
determination.[9]
We repeat that what petitioner decries as unsuccessful is not a final result. It is only a
beginning. The Court is not inclined to stifle deregulation as enacted by Congress from its very
start. We leave alone the program of deregulation at this stage. Reasonable time will prove the
wisdom or folly of the deregulation program for which Congress and not the Court is
accountable.
Petitioner argues further that the public interest requires price controls while the oligopoly
exists, for that is the only way the public can be protected from monopoly or oligopoly
pricing. But is indefinite price control the only feasible and legal way to enforce the
constitutional mandate against oligopolies?
Article 186 of the Revised Penal Code, as amended, punishes as a felony the creation of
monopolies and combinations in restraint of trade. The Solicitor General, on the other hand, cites
provisions of R.A. 8479 intended to prevent competition from being corrupted or
manipulated. Section 11, entitled Anti-Trust Safeguards, defines and prohibits cartelization and
predatory pricing. It penalizes the persons and officers involved with imprisonment of three (3)
to seven (7) years and fines ranging from One million to Two million pesos. For this purpose, a
Joint Task Force from the Department of Energy and Department of Justice is created under
Section 14 to investigate and order the prosecution of violations.
Sections 8 and 9 of the Act, meanwhile, direct the Departments of Foreign Affairs, Trade
and Industry, and Energy to undertake strategies, incentives and benefits, including international
information campaigns, tax holidays and various other agreements and utilizations, to invite and
encourage the entry of new participants. Section 6 provides for uniform tariffs at three percent
(3%).
Section 13 of the Act provides for Remedies, under which the filing of actions by
government prosecutors and the investigation of private complaints by the Task Force is
provided. Sections 14 and 15 provide how the Department of Energy shall monitor and prevent
the occurrence of collusive pricing in the industry.
It can be seen, therefore, that instead of the price controls advocated by the petitioner,
Congress has enacted anti-trust measures which it believes will promote free and fair
competition. Upon the other hand, the disciplined, determined, consistent and faithful execution
of the law is the function of the President. As stated by public respondents, the remedy against
unreasonable price increases is not the nullification of Section 19 of R.A. 8479 but the setting
into motion of its various other provisions.
For this Court to declare unconstitutional the key provision around which the laws anti-trust
measures are clustered would mean a constitutionally interdicted distrust of the wisdom of
Congress and of the determined exercise of executive power.
Having decided that deregulation is the policy to follow, Congress and the President have
the duty to set up the proper and effective machinery to ensure that it works. This is something
which cannot be adjudicated into existence. This Court is only an umpire of last resort whenever
the Constitution or a law appears to have been violated. There is no showing of a constitutional
violation in this case.
WHEREFORE, the petition is DISMISSED.
PROVINCE OF NORTH COTABATO VS GOVERNMENT OF THE
REPUBLIC OF THE PHILIPPINES
Posted by kaye lee on 9:43 PM

G.R. No. 183591 October 14 2008


Province of North Cotabato vs Government of the Republic of the Philippines

FACTS:

On August 5, 2008, the Government of the Republic of the Philippines and the Moro Islamic Liberation Front
(MILF) were scheduled to sign a Memorandum of Agreement of the Ancestral Domain Aspect of the GRP - MILF
Tripoli Agreement on Peace of 2001 in Kuala Lumpur, Malaysia.

Invoking the right to information on matters of public concern, the petitioners seek to compel respondents to
disclose and furnish them the complete and official copies of the MA-AD and to prohibit the slated signing of the
MOA-AD and the holding of public consultation thereon. They also pray that the MOA-AD be declared
unconstitutional. The Court issued a TRO enjoining the GRP from signing the same.

ISSUES:

1. Whether or not the constitutionality and the legality of the MOA is ripe for adjudication;

2. Whether or not there is a violation of the people's right to information on matters of public concern (Art 3 Sec. 7)
under a state policy of full disclosure of all its transactions involving public interest (Art 2, Sec 28) including public
consultation under RA 7160 (Local Government Code of 1991)

3. Whether or not the signing of the MOA, the Government of the Republic of the Philippines would be binding
itself

a) to create and recognize the Bangsamoro Juridical Entity (BJE) as a separate state, or a juridical, territorial or
political subdivision not recognized by law;

b) to revise or amend the Constitution and existing laws to conform to the MOA;

c) to concede to or recognize the claim of the Moro Islamic Liberation Front for ancestral domain in violation of
Republic Act No. 8371 (THE INDIGENOUS PEOPLES RIGHTS ACT OF 1997),

particularly Section 3(g) & Chapter VII (DELINEATION,

RECOGNITION OF ANCESTRAL DOMAINS)

RULINGS:
1. Yes, the petitions are ripe for adjudication. The failure of the respondents to consult the local government units or
communities affected constitutes a departure by respondents from their mandate under EO No. 3. Moreover, the
respondents exceeded their authority by the mere act of guaranteeing amendments to the Constitution. Any alleged
violation of the Constitution by any branch of government is a proper matter for judicial review.

As the petitions involve constitutional issues which are of paramount public interest or of transcendental importance,
the Court grants the petitioners, petitioners-in-intervention and intervening respondents the requisite locus standi in
keeping with the liberal stance adopted in David v. Macapagal- Arroyo.

In Pimentel, Jr. v. Aguirre, this Court held:

x x x [B]y the mere enactment of the questioned law or the approval of the challenged action, the dispute is
said to have ripened into a judicial controversy even without any other overt act . Indeed, even a singular
violation of the Constitution and/or the law is enough to awaken judicial duty.x x x x

By the same token, when an act of the President, who in our constitutional scheme is a coequal of Congress, is
seriously alleged to have infringed the Constitution and the laws x x x settling the dispute becomes the duty and the
responsibility of the courts.

That the law or act in question is not yet effective does not negate ripeness.

2. Yes. The Court finds that there is a grave violation of the Constitution involved in the matters of public concern
(Sec 7 Art III) under a state policy of full disclosure of all its transactions involving public interest (Art 2, Sec 28)
including public consultation under RA 7160 (Local Government Code of 1991).

(Sec 7 ArtIII) The right to information guarantees the right of the people to demand information, while Sec 28
recognizes the duty of officialdom to give information even if nobody demands. The complete and effective exercise
of the right to information necessitates that its complementary provision on public disclosure derive the same self-
executory nature, subject only to reasonable safeguards or limitations as may be provided by law.

The contents of the MOA-AD is a matter of paramount public concern involving public interest in the highest order.
In declaring that the right to information contemplates steps and negotiations leading to the consummation of the
contract, jurisprudence finds no distinction as to the executory nature or commercial character of the agreement.

E.O. No. 3 itself is replete with mechanics for continuing consultations on both national and local levels and for a
principal forum for consensus-building. In fact, it is the duty of the Presidential Adviser on the Peace Process to
conduct regular dialogues to seek relevant information, comments, advice, and recommendations from peace
partners and concerned sectors of society.

3.

a) to create and recognize the Bangsamoro Juridical Entity (BJE) as a separate state, or a juridical, territorial or
political subdivision not recognized by law;

Yes. The provisions of the MOA indicate, among other things, that the Parties aimed to vest in the BJE the
status of an associated state or, at any rate, a status closely approximating it.
The concept of association is not recognized under the present Constitution.

No province, city, or municipality, not even the ARMM, is recognized under our laws as having an “associative”
relationship with the national government. Indeed, the concept implies powers that go beyond anything ever granted
by the Constitution to any local or regional government. It also implies the recognition of the associated entity as
a state. The Constitution, however, does not contemplate any state in this jurisdiction other than the Philippine
State, much less does it provide for a transitory status that aims to prepare any part of Philippine territory for
independence.

The BJE is a far more powerful entity than the autonomous region recognized in the Constitution. It is not
merely an expanded version of the ARMM, the status of its relationship with the national government being
fundamentally different from that of the ARMM. Indeed, BJE is a state in all but name as it meets the criteria of
a state laid down in the Montevideo Convention, namely, a permanent population, a defined territory, a
government, and a capacity to enter into relations with other states.

Even assuming arguendo that the MOA-AD would not necessarily sever any portion of Philippine territory, the spirit
animating it – which has betrayed itself by its use of the concept of association – runs counter to the national
sovereignty and territorial integrity of the Republic.

The defining concept underlying the relationship between the national government and the BJE being itself contrary
to the present Constitution, it is not surprising that many of the specific provisions of the MOA-AD on the formation
and powers of the BJE are in conflict with the Constitution and the laws. The BJE is more of a state than an
autonomous region. But even assuming that it is covered by the term “autonomous region” in the constitutional
provision just quoted, the MOA-AD would still be in conflict with it.

b) to revise or amend the Constitution and existing laws to conform to the MOA:

The MOA-AD provides that “any provisions of the MOA-AD requiring amendments to the existing legal
framework shall come into force upon the signing of a Comprehensive Compact and upon effecting the necessary
changes to the legal framework,” implying an amendment of the Constitution to accommodate the MOA-
AD. This stipulation, in effect, guaranteed to the MILF the amendment of the Constitution .

It will be observed that the President has authority, as stated in her oath of office, only to preserve and defend the
Constitution. Such presidential power does not, however, extend to allowing her to change the Constitution, but
simply to recommend proposed amendments or revision. As long as she limits herself to recommending these
changes and submits to the proper procedure for constitutional amendments and revision, her mere recommendation
need not be construed as an unconstitutional act.

The “suspensive clause” in the MOA-AD viewed in light of the above-discussed standards.

Given the limited nature of the President’s authority to propose constitutional amendments, she cannot
guarantee to any third party that the required amendments will eventually be put in place, nor even be
submitted to a plebiscite. The most she could do is submit these proposals as recommendations either to Congress
or the people, in whom constituent powers are vested.

c) to concede to or recognize the claim of the Moro Islamic Liberation Front for ancestral domain in violation of
Republic Act No. 8371 (THE INDIGENOUS PEOPLES RIGHTS ACT OF 1997),

particularly Section 3(g) & Chapter VII (DELINEATION,

RECOGNITION OF ANCESTRAL DOMAINS)

This strand begins with the statement that it is “the birthright of all Moros and all Indigenous peoples of Mindanao
to identify themselves and be accepted as ‘Bangsamoros.’” It defines “Bangsamoro people” as the natives or original
inhabitants of Mindanao and its adjacent islands including Palawan and the Sulu archipelago at the time of conquest
or colonization, and their descendants whether mixed or of full blood, including their spouses.

Thus, the concept of “Bangsamoro,” as defined in this strand of the MOA-AD, includes not only “Moros” as
traditionally understood even by Muslims, but all indigenous peoples of Mindanao and its adjacent islands. The
MOA-AD adds that the freedom of choice of indigenous peoples shall be respected. What this freedom of choice
consists in has not been specifically defined. The MOA-AD proceeds to refer to the “Bangsamoro homeland,” the
ownership of which is vested exclusively in the Bangsamoro people by virtue of their prior rights of occupation.
Both parties to the MOA-AD acknowledge that ancestral domain does not form part of the public domain.

Republic Act No. 8371 or the Indigenous Peoples Rights Act of 1997 provides for clear-cut procedure for the
recognition and delineation of ancestral domain, which entails, among other things, the observance of the free and
prior informed consent of the Indigenous Cultural Communities/Indigenous Peoples. Notably, the statute does not
grant the Executive Department or any government agency the power to delineate and recognize an ancestral
domain claim by mere agreement or compromise.

Two, Republic Act No. 7160 or the Local Government Code of 1991 requires all national offices to conduct
consultations beforeany project or program critical to the environment and human ecology including those that may
call for the eviction of a particular group of people residing in such locality, is implemented therein. The MOA-AD
is one peculiar program that unequivocally and unilaterally vests ownership of a vast territory to the Bangsamoro
people, which could pervasively and drastically result to the diaspora or displacement of a great number of
inhabitants from their total environment.

CONCLUSION:

In sum, the Presidential Adviser on the Peace Process committed grave abuse of discretion when he failed to carry
out the pertinent consultation process, as mandated by E.O. No. 3, Republic Act No. 7160, and Republic Act No.
8371. The furtive process by which the MOA-AD was designed and crafted runs contrary to and in excess of the
legal authority, and amounts to a whimsical, capricious, oppressive, arbitrary and despotic exercise thereof. It
illustrates a gross evasion of positive duty and a virtual refusal to perform the duty enjoined.

The MOA-AD cannot be reconciled with the present Constitution and laws. Not only its specific provisions but the
very concept underlying them, namely, the associative relationship envisioned between the GRP and the BJE, are
unconstitutional, for the concept presupposes that the associated entity is a state and implies that the same is on its
way to independence.
Case Digest: GR No. 183591
2/4/2015

4 Comments

Province of North Cotabato, Province of Zamboanga Del Norte, City of Iligan,


City of Zamboanga, petitioners in intervention Province of Sultan Kudarat, City
of Isabela and Municipality of Linnamon, Intervenors Franklin Drilon and Adel
Tamano and Sec. Mar Roxas

-vs-

Ermita Exec.Sec., Romulo Sec DFA, Andaya Sec DBM, Ventura Administrator
National Mapping & Resource Information Authority and Davide Jr. and
respondents in intervention Muslim Multi-Sectoral Movement for Peace and
Development and Muslim Legal Assistance Foundation Inc.,

Facts:

Subject of this case is the Memorandum of Agreement on the Ancestral Domain


(MOA-AD) which is scheduled to be signed by the Government of the Republic of
the Philippines and the MILF in August 05, 2008. Five cases bearing the same
subject matter were consolidated by this court namely:-

 GR 183591 by the Province of Cotabato and Vice Governor Pinol on its


prayer to declare unconstitutional and to have the MOA-AD disclosed to the
public and be open for public consultation.
 GR 183752 by the City of Zamboanga et al on its prayer to declare null and
void said MOA-AD and to exclude the city to the BJE.
 GR 183893 by the City of Iligan enjoining the respondents from signing the
MOA-AD and additionally impleading Exec. Sec. Ermita.
 GR 183951 by the Province of Zamboanga del Norte et al, praying to declare
null and void the MOA-AD and without operative effect and those
respondents enjoined from executing the MOA-AD.
 GR 183692 by Maceda, Binay and Pimentel III, praying for a judgment
prohibiting and permanently enjoining respondents from formally signing and
executing the MOA-AD and or any other agreement derived therefrom or
similar thereto, and nullifying the MOA-AD for being unconstitutional and
illegal and impleading Iqbal.

The MOA-AD is a result of various agreements entered into by and


between the government and the MILF starting in 1996; then in 1997, they signed
the Agreement on General Cessation of Hostilities; and the following year, they
signed the General Framework of Agreement of Intent on August 27, 1998.
However, in 1999 and in the early of 2000, the MILF attacked a number of
municipalities in Central Mindanao. In March 2000, they took the hall of
Kauswagan, Lanao del Norte; hence, then Pres. Estrada declared an all-out war-
which tolled the peace negotiation. It was when then Pres. Arroyo assumed office,
when the negotiation regarding peace in Mindanao continued. MILF was hesitant;
however, this negotiation proceeded when the government of Malaysia interceded.
Formal peace talks resumed and MILF suspended all its military actions. The Tripoli
Agreement in 2001 lead to the ceasefire between the parties. After the death of
MILF Chairman Hashim and Iqbal took over his position, the crafting of MOA-AD
in its final form was born.

 MOA-AD Overview

This is an agreement to be signed by the GRP and the MILF. Used as reference in
the birth of this MOA-AD are the Tripoli Agreement, organic act of ARMM, IPRA
Law, international laws such as ILO Convention 169, the UN Charter etc., and the
principle of Islam i.e compact right entrenchment (law of compact, treaty and
order). The body is divided into concepts and principles, territory, resources, and
governance.

Embodied in concepts and principles, is the definition of Bangsamoro as all


indigenous peoples of Mindanao and its adjacent islands. These people have the
right to self- governance of their Bangsamoro homeland to which they have
exclusive ownership by virtue of their prior rights of occupation in the land. The
MOA-AD goes on to describe the Bangsamoro people as "the ‘First Nation' with
defined territory and with a system of government having entered into treaties of
amity and commerce with foreign nations." It then mentions for the first time the
"Bangsamoro Juridical Entity" (BJE) to which it grants the authority and
jurisdiction over the Ancestral Domain and Ancestral Lands of the Bangsamoro.

As defined in the territory of the MOA-AD, the BJE shall embrace the Mindanao-
Sulu-Palawan geographic region, involving the present ARMM, parts of which are
those which voted in the inclusion to ARMM in a plebiscite. The territory is divided
into two categories, “A” which will be subject to plebiscite not later than 12 mos.
after the signing and “B” which will be subject to plebiscite 25 years from the
signing of another separate agreement. Embodied in the MOA-AD that the BJE
shall have jurisdiction over the internal waters-15kms from the coastline of the
BJE territory; they shall also have "territorial waters," which shall stretch beyond
the BJE internal waters up to the baselines of the Republic of the Philippines (RP)
south east and south west of mainland Mindanao; and that within these territorial
waters, the BJE and the government shall exercise joint jurisdiction, authority and
management over all natural resources. There will also be sharing of minerals in the
territorial waters; but no provision on the internal waters.

Included in the resources is the stipulation that the BJE is free to enter into any
economic cooperation and trade relations with foreign countries and shall have the
option to establish trade missions in those countries, as well as environmental
cooperation agreements, but not to include aggression in the GRP. The external
defense of the BJE is to remain the duty and obligation of the government. The
BJE shall have participation in international meetings and events" like those of the
ASEAN and the specialized agencies of the UN. They are to be entitled to
participate in Philippine official missions and delegations for the negotiation of
border agreements or protocols for environmental protection and equitable sharing
of incomes and revenues involving the bodies of water adjacent to or between the
islands forming part of the ancestral domain. The BJE shall also have the right to
explore its resources and that the sharing between the Central Government and
the BJE of total production pertaining to natural resources is to be 75:25 in favor
of the BJE. And they shall have the right to cancel or modify concessions and
TLAs.

And lastly in the governance, the MOA-AD claims that the relationship between
the GRP and MILF is associative i.e. characterized by shared authority and
responsibility. This structure of governance shall be further discussed in the
Comprehensive Compact, a stipulation which was highly contested before the court.
The BJE shall also be given the right to build, develop and maintain its own
institutions, the details of which shall be discussed in the comprehensive compact
as well.

Issues:

1. WON the petitions have complied with the procedural requirements for the
exercise of judicial review

2. WON respondents violate constitutional and statutory provisions on public


consultation and the right to information when they negotiated and later initialed
the MOA-AD; and

3. WON the contents of the MOA-AD violated the Constitution and the laws

Ruling:

The SC declared the MOA-AD contrary to law and the Constitution.

 On the Procedural Issue

1st issue: As regards the procedural issue, SC upheld that there is indeed a need
for the exercise of judicial review.

The power of judicial review is limited to actual cases or controversy, that is the
court will decline on issues that are hypothetical, feigned problems or mere
academic questions. Related to the requirement of an actual case or controversy is
the requirement of ripeness. The contention of the SolGen is that there is no issue
ripe for adjudication since the MOA-AD is only a proposal and does not
automatically create legally demandable rights and obligations. Such was denied.

The SC emphasized that the petitions are alleging acts made in violation of their
duty or in grave abuse of discretion. Well-settled jurisprudence states that acts
made by authority which exceed their authority, by violating their duties under
E.O. No. 3 and the provisions of the Constitution and statutes, the petitions make a
prima facie case for Certiorari, Prohibition, and Mandamus, and an actual case or
controversy ripe for adjudication exists. When an act of a branch of government is
seriously alleged to have infringed the Constitution, it becomes not only the right
but in fact the duty of the judiciary to settle the dispute. This is aside from the
fact that concrete acts made under the MOA-AD are not necessary to render the
present controversy ripe and that the law or act in question as not yet effective
does not negate ripeness.

With regards to the locus standi, the court upheld the personalities of the
Province of Cotabato, Province of Zamboanga del norte, City of Iligan, City of
Zamboanga, petitioners in intervention Province of Sultan Kudarat, City of Isabela
and Municipality of Linnamon to have locus standi since it is their LGUs which will
be affected in whole or in part if include within the BJE. Intervenors Franklin
Drilon and Adel Tamano, in alleging their standing as taxpayers, assert that
government funds would be expended for the conduct of an illegal and
unconstitutional plebiscite to delineate the BJE territory. On that score alone,
they can be given legal standing. Senator Mar Roxas is also given a standing as an
intervenor. And lastly, the Intervening respondents Muslim Multi-Sectoral
Movement for Peace and Development, an advocacy group for justice and the
attainment of peace and prosperity in Muslim Mindanao; and Muslim Legal
Assistance Foundation Inc., a non-government organization of Muslim lawyers since
they stand to be benefited or prejudiced in the resolution of the petitions
regarding the MOA-AD.

On the contention of mootness of the issue considering the signing of the MOA-AD
has already been suspended and that the President has already disbanded the GRP,
the SC disagrees. The court reiterates that the moot and academic principle is a
general rule only, the exceptions, provided in David v. Macapagal-Arroyo, that it
will decide cases, otherwise moot and academic, if it finds that (a) there is a grave
violation of the Constitution; (b) the situation is of exceptional character and
paramount public interest is involved; (c) the constitutional issue raised requires
formulation of controlling principles to guide the bench, the bar, and the public;
and (d) the case is capable of repetition yet evading review; and that where there
is a voluntary cessation of the activity complained of by the defendant or doer, it
does not divest the court the power to hear and try the case especially when the
plaintiff is seeking for damages or injunctive relief.

Clearly, the suspension of the signing of the MOA-AD and the disbandment of the
GRP did not render the petitions moot and academic. The MOA-AD is subject to
further legal enactments including possible Constitutional amendments more than
ever provides impetus for the Court to formulate controlling principles to guide the
bench, the bar, the public and, in this case, the government and its negotiating
entity.

At all events, the Court has jurisdiction over most if not the rest of the petitions.
There is a reasonable expectation that petitioners will again be subjected to the
same problem in the future as respondents' actions are capable of repetition, in
another or any form. But with respect to the prayer of Mandamus to the signing of
the MOA-AD, such has become moot and academic considering that parties have
already complied thereat.

 On the Substantive Issue

2nd Issue: The SC ruled that the MOA-AD is a matter of public concern, involving
as it does the sovereignty and territorial integrity of the State, which directly
affects the lives of the public at large.

As enshrined in the Constitution, the right to information guarantees the right of


the people to demand information, and integrated therein is the recognition of the
duty of the officialdom to give information even if nobody demands. The policy of
public disclosure establishes a concrete ethical principle for the conduct of public
affairs in a genuinely open democracy, with the people's right to know as the
centerpiece. It is a mandate of the State to be accountable by following such
policy. These provisions are vital to the exercise of the freedom of expression and
essential to hold public officials at all times accountable to the people.

Also, it was held that such stipulation in the Constitution is self-executory with
reasonable safeguards —the effectivity of which need not await the passing of a
statute. Hence, it is essential to keep open a continuing dialogue or process of
communication between the government and the people. It is in the interest of the
State that the channels for free political discussion be maintained to the end that
the government may perceive and be responsive to the people's will.
The idea of a feedback mechanism was also sought for since it is corollary to the
twin rights to information and disclosure. And feedback means not only the
conduct of the plebiscite as per the contention of the respondents. Clearly, what
the law states is the right of the petitioners to be consulted in the peace agenda
as corollary to the constitutional right to information and disclosure. As such,
respondent Esperon committed grave abuse of discretion for failing to carry out
the furtive process by which the MOA-AD was designed and crafted runs contrary
to and in excess of the legal authority, and amounts to a whimsical, capricious,
oppressive, arbitrary and despotic exercise thereto. Moreover, he cannot invoke of
executive privilege because he already waived it when he complied with the Court’s
order to the unqualified disclosure of the official copies of the final draft of the
MOA-AD.

In addition, the LGU petitioners has the right to be involved in matters related to
such peace talks as enshrined in the State policy. The MOA-AD is one peculiar
program that unequivocally and unilaterally vests ownership of a vast territory to
the Bangsamoro people, which could pervasively and drastically result to the
diaspora or displacement of a great number of inhabitants from their total
environment.

With respect to the ICC/IPPs they also have the right to participate fully at all
levels on decisions that would clearly affect their lives, rights and destinies. The
MOA-AD is an instrument recognizing ancestral domain, hence it should have
observed the free and prior informed consent to the ICC/IPPs; but it failed to do
so. More specially noted by the court is the excess in authority exercised by the
respondent—since they allowed delineation and recognition of ancestral domain
claim by mere agreement and compromise; such power cannot be found in IPRA or
in any law to the effect.

3rd issue: With regard to the provisions of the MOA-AD, there can be no question
that they cannot be all accommodated under the present Constitution and laws.
Not only its specific provisions but the very concept underlying them:

 On matters of the Constitution.


Association as the type of relationship governing between the parties. The
parties manifested that in crafting the MOA-AD, the term association was
adapted from the international law. In international law, association happens when
two states of equal power voluntarily establish durable links i.e. the one state, the
associate, delegates certain responsibilities to the other, principal, while
maintaining its international status as state; free association is a middle ground
between integration and independence. The MOA-AD contains many provisions that
are consistent with the international definition of association which fairly would
deduced that the agreement vest into the BJE a status of an associated state, or
at any rate, a status closely approximating it. The court vehemently objects
because the principle of association is not recognized under the present
Constitution.

 On the recognition of the BJE entity as a state. The concept implies power
beyond what the Constitution can grant to a local government; even the
ARMM do not have such recognition; and the fact is such concept implies
recognition of the associated entity as a state. There is nothing in the law
that contemplate any state within the jurisdiction other than the Philippine
State, much less does it provide for a transitory status that aims to prepare
any part of Philippine territory for independence. The court disagrees with
the respondent that the MOA-AD merely expands the ARMM. BJE is a state
in all but name as it meets the criteria of a state laid down in the
Montevideo Convention, namely, a permanent population, a defined territory,
a government, and a capacity to enter into relations with other states. As
such the MOA-AD clearly runs counter to the national sovereignty and
territorial integrity of the Republic.

 On the expansion of the territory of the BJE. The territory included in the
BJE includes those areas who voted in the plebiscite for them to become
part of the ARMM. The stipulation of the respondents in the MOA-AD that
these areas need not participate in the plebiscite is in contrary to the
express provision of the Constitution. The law states that that "[t]he
creation of the autonomous region shall be effective when approved by a
majority of the votes cast by the constituent units in a plebiscite called for
the purpose, provided that only provinces, cities, and geographic areas voting
favorably in such plebiscite shall be included in the autonomous region."
Clearly, assuming that the BJE is just an expansion of the ARMM, it would
still run afoul the wordings of the law since those included in its territory
are areas which voted in its inclusion to the ARMM and not to the BJE.

 On the powers vested in the BJE as an entity. The respondents contend that
the powers vested to the BJE in the MOA-AD shall be within sub-paragraph
9 of sec 20, art. 10 of the constitution and that a mere passage of a law is
necessary in order to vest in the BJE powers included in the agreement. The
Court was not persuaded. SC ruled that such conferment calls for
amendment of the Constitution; otherwise new legislation will not concur
with the Constitution. Take for instance the treaty making power vested to
the BJE in the MOA-AD. The Constitution is clear that only the President
has the sole organ and is the country’s sole representative with foreign
nation. Should the BJE be granted with the authority to negotiate with
other states, the former provision must be amended consequently. Section
22 must also be amended—the provision of the law that promotes national
unity and development. Because clearly, associative arrangement of the
MOA-AD does not epitomize national unity but rather, of semblance of
unity. The associative ties between the BJE and the national government,
the act of placing a portion of Philippine territory in a status which, in
international practice, has generally been a preparation for independence, is
certainly not conducive to national unity.

 On matters of domestic statutes.

o Provisions contrary to the organic act of ARMM. RA 9054 is a bar to the


adoption of the definition of Bangsamoro people used in the MOA-AD. Said law
specifically distinguishes between the Bangsamoro people and the Tribal peoples
that is contrary with the definition of the MOA-AD which includes all indigenous
people of Mindanao.

o Provisions contrary to the IPRA law. Also, the delineation and recognition of the
ancestral domain is a clear departure from the procedure embodied in the IPRA
law which ironically is the term of reference of the MOA-AD.

 On matters of international law.

The Philippines adopts the generally accepted principle of international law as part
of the law of the land. In international law, the right to self-determination has long
been recognized which states that people can freely determine their political
status and freely pursue their economic, social, and cultural development. There
are the internal and external self-determination—internal, meaning the self-
pursuit of man and the external which takes the form of the assertion of the right
to unilateral secession. This principle of self-determination is viewed with respect
accorded to the territorial integrity of existing states. External self-
determination is only afforded in exceptional cases when there is an actual block in
the meaningful exercise of the right to internal self-determination. International
law, as a general rule, subject only to limited and exceptional cases, recognizes
that the right of disposing national territory is essentially an attribute of the
sovereignty of every state.

On matters relative to indigenous people, international law states that indigenous


peoples situated within states do not have a general right to independence or
secession from those states under international law, but they do have rights
amounting to what was discussed above as the right to internal self-determination;
have the right to autonomy or self-government in matters relating to their internal
and local affairs, as well as ways and means for financing their autonomous
functions; have the right to the lands, territories and resources which they have
traditionally owned, occupied or otherwise used or acquired.

Clearly, there is nothing in the law that required the State to guarantee the
indigenous people their own police and security force; but rather, it shall be the
State, through police officers, that will provide for the protection of the people.
With regards to the autonomy of the indigenous people, the law does not obligate
States to grant indigenous peoples the near-independent status of a state; since it
would impair the territorial integrity or political unity of sovereign and independent
states.

 On the basis of the suspensive clause.

o It was contented by the respondents that grave abuse of discretion cannot be


had, since the provisions assailed as unconstitutional shall not take effect until the
necessary changes to the legal framework are effected.

The Court is not persuaded. This suspensive clause runs contrary to Memorandum
of Instructions from the President stating that negotiations shall be conducted in
accordance to the territorial integrity of the country—such was negated by the
provision on association incorporated in the MOA-AD. Apart from this, the
suspensive clause was also held invalid because of the delegated power to the GRP
Peace panel to advance peace talks even if it will require new legislation or even
constitutional amendments. The legality of the suspensive clause hence hinges on
the query whether the President can exercise such power as delegated by EO No.3
to the GRP Peace Panel. Well settled is the rule that the President cannot delegate
a power that she herself does not possess. The power of the President to conduct
peace negotiations is not explicitly mentioned in the Constitution but is rather
implied from her powers as Chief Executive and Commander-in-chief. As Chief
Executive, the President has the general responsibility to promote public peace,
and as Commander-in-Chief, she has the more specific duty to prevent and
suppress rebellion and lawless violence.

As such, the President is given the leeway to explore, in the course of peace
negotiations, solutions that may require changes to the Constitution for their
implementation. At all event, the president may not, of course, unilaterally
implement the solutions that she considers viable; but she may not be prevented
from submitting them as recommendations to Congress, which could then, if it is
minded, act upon them pursuant to the legal procedures for constitutional
amendment and revision.

While the President does not possess constituent powers - as those powers may be
exercised only by Congress, a Constitutional Convention, or the people through
initiative and referendum - she may submit proposals for constitutional change to
Congress in a manner that does not involve the arrogation of constituent powers.
Clearly, the principle may be inferred that the President - in the course of
conducting peace negotiations - may validly consider implementing even those
policies that require changes to the Constitution, but she may not unilaterally
implement them without the intervention of Congress, or act in any way as if the
assent of that body were assumed as a certainty. The President’s power is limited
only to the preservation and defense of the Constitution but not changing the same
but simply recommending proposed amendments or revisions.

o The Court ruled that the suspensive clause is not a suspensive condition but is a
term because it is not a question of whether the necessary changes to the legal
framework will take effect; but, when. Hence, the stipulation is mandatory for the
GRP to effect the changes to the legal framework –which changes would include
constitutional amendments. Simply put, the suspensive clause is inconsistent with
the limits of the President's authority to propose constitutional amendments, it
being a virtual guarantee that the Constitution and the laws of the Republic of the
Philippines will certainly be adjusted to conform to all the "consensus points" found
in the MOA-AD. Hence, it must be struck down as unconstitutional.

 On the concept underlying the MOA-AD.

While the MOA-AD would not amount to an international agreement or unilateral


declaration binding on the Philippines under international law, respondents' act of
guaranteeing amendments is, by itself, already a constitutional violation that
renders the MOA-AD fatally defective. The MOA-AD not being a document that
can bind the Philippines under international law notwithstanding, respondents'
almost consummated act of guaranteeing amendments to the legal framework is, by
itself, sufficient to constitute grave abuse of discretion. The grave abuse lies not
in the fact that they considered, as a solution to the Moro Problem, the creation
of a state within a state, but in their brazen willingness to guarantee that
Congress and the sovereign Filipino people would give their imprimatur to their
solution.Upholding such an act would amount to authorizing a usurpation of the
constituent powers vested only in Congress, a Constitutional Convention, or the
people themselves through the process of initiative, for the only way that the
Executive can ensure the outcome of the amendment process is through an undue
influence or interference with that process.
CASE DIGEST: THE PROVINCE OF NORTH COTABATO, ET AL . V . THE
GOVERNMENT OF THE REPUBLIC OF THE PHILIPPINES, ET AL .
Published by arce on September 7, 2013 | Leave a response

THE PROVINCE OF NORTH COTABATO, et al . v . THE GOVERNMENT OF


THE REPUBLIC OF THE PHILIPPINES, et al .

President Gloria Macapagal-Arroyo, in line with the government‘s policy of pursuing


peace negotiations with the Moro Islamic Liberation Front (MILF), asked Prime
Minister Mahathir Mohammad to convince the MILF to continue negotiating with the
government. MILF, thereafter, convened its Central Committee and decided to meet
with the Government of the Republic of the Philippines (GRP). Formal peace talks were
held in Libya which resulted to the crafting of the GRP-MILF Tripoli Agreement on
Peace (Tripoli Agreement 2001) which consists of three (3) aspects: a.) security aspect;
b.) rehabilitation aspect; and c.) ancestral domain aspect. Various negotiations were
held which led to the finalization of the Memorandum of Agreement on the Ancestral
Domain (MOA-AD). The said memorandum was set to be signed last August 5, 2008. In
its body, it grants ―the authority and jurisdiction over the Ancestral Domain and
Ancestral Lands of the Bangsamoro to the Bangsamoro Juridical Entity (BJE). The
latter, in addition, has the freedom to enter into any economic cooperation and trade
relation with foreign countries. ―The sharing between the Central Government and the
BJE of total production pertaining to natural resources is to be 75:25 in favor of the
BJE. The MOA-AD further provides for the extent of the territory of the Bangsamoro. It
describes it as ―the land mass as well as the maritime, terrestrial, fluvial
and alluvial domains, including the aerial domain and the atmospheric space above it,
embracing the Mindanao-Sulu-Palawan geographic region. With regard to governance,
on the other hand, a shared responsibility and authority between the Central
Government and BJE was provided. The relationship was described as ―associative.
With the formulation of the MOA-AD, petitioners aver that the negotiation and
finalization of the MOA-AD violates constitutional and statutory provisions on public
consultation, as mandated by Executive Order No. 3, and right to information. They
further contend that it violates the Constitution and laws. Hence, the filing of the
petition.

ISSUES:

1) Whether or not the MOA-AD violates constitutional and statutory provisions on


public consultation and right to information 2) Whether or not the MOA-AD violates the
Constitution and the laws.

HELD:

The MOA-AD subject of the present cases is of public concern, involving as it does the
sovereignty and territorial integrity of the State, which directly affects the lives of the
public at large. Intended as a ―splendid symmetry to the right to information under
the Bill of Rights is the policy of public disclosure under Section 28, Article II of the
Constitution which provides that subject to reasonable conditions prescribed by law, the
State adopts and implements a policy of full public disclosure of all its transactions
involving public interest. Moreover, the policy of full public disclosure enunciated in
above-quoted Section 28 complements the right of access to information on matters of
public concern found in the Bill of Rights. The right to information guarantees the right
of the people to demand information, while Section 28 recognizes the duty of
officialdom to give information even if nobody demands. The policy of public disclosure
establishes a concrete ethical principle for the conduct of public affairs in a genuinely
open democracy, with the people‘s right to know as the centerpiece. It is a mandate of
the State to be accountable by following such policy. These provisions are vital to the
exercise of the freedom of expression and essential to hold public officials at all times
accountable to the people. Indubitably, the effectivity of the policy of public disclosure
need not await the passing of a statute. As Congress cannot revoke this principle, it is
merely directed to provide for ―reasonable safeguards.‖ The complete and effective
exercise of the right to information necessitates that its complementary provision on
public disclosure derive the same self-executory nature. Since both provisions go hand-
in-hand, it is absurd to say that the broader right to information on matters of public
concern is already enforceable while the correlative duty of the State to disclose its
transactions involving public interest is not enforceable until there is an enabling law.
Respondents cannot thus point to the absence of an implementing legislation as an
excuse in not effecting such policy. An essential element of these freedoms is to keep
open a continuing dialogue or process of communication between the government and
the people. It is in the interest of the State that the channels for free political discussion
be maintained to the end that the government may perceive and be responsive to the
people‘s will. Envisioned to be corollary to the twin rights to information and disclosure
is the design for feedback mechanisms. The imperative of a public consultation, as a
species of the right to information, is evident in the ―marching orders‖ to respondents.
The mechanics for the duty to disclose information and to conduct public consultation
regarding the peace agenda and process is manifestly provided by E.O. No. 3. The
preambulatory clause of E.O. No. 3 declares that there is a need to further enhance
the contribution of civil society to the comprehensive peace process by institutionalizing
the people‘s participation. One of the three underlying principles of the comprehensive
peace process is that it ―should be community-based, reflecting the sentiments, values
and principles important to all Filipinos and ―shall be defined not by the government
alone, nor by the different contending groups only, but by all Filipinos as one
community. Included as a component of the comprehensive peace process is consensus-
building and empowerment for peace, which includes ―continuing consultations on
both national and local levels to build consensus for a peace agenda and process, and the
mobilization and facilitation of people‘s participation in the peace process.Clearly, E.O.
No. 3 contemplates not just the conduct of a plebiscite to effectuate “continuing”
consultations, contrary to respondents’ position that plebiscite is “more than sufficient
consultation.Further, E.O. No. 3 enumerates the functions and responsibilities of the
PAPP, one of which is to ―conduct regular dialogues with the National Peace Forum
(NPF) and other peace partners to seek relevant information, comments,
recommendations as well as to render appropriate and timely reports on the progress of
the comprehensive peace process. E.O. No. 3 mandates the establishment of the NPF to
be ―the principal forum for the Presidential Adviser on Peace Progress (PAPP) to
consult with and seek advi[c]e from the peace advocates, peace partners and concerned
sectors of society on both national and local levels, on the implementation of the
comprehensive peace process, as well as for government[-]civil society dialogue and
consensus-building on peace agenda and initiatives. In fine, E.O. No. 3 establishes
petitioners’ right to be consulted on the peace agenda, as a corollary to the constitutional
right to information and disclosure. In general, the objections against the MOA-AD
center on the extent of the powers conceded therein to the BJE. Petitioners assert that
the powers granted to the BJE exceed those granted to any local government under
present laws, and even go beyond those of the present ARMM. Before assessing some of
the specific powers that would have been vested in the BJE, however, it would be useful
to turn first to a general idea that serves as a unifying link to the different provisions of
the MOA-AD, namely, the international law concept of association. Significantly, the
MOA-AD explicitly alludes to this concept, indicating that the Parties actually framed its
provisions with it in mind. Association is referred to in paragraph 3 on TERRITORY,
paragraph 11 on RESOURCES, and paragraph 4 on GOVERNANCE. It is in the last
mentioned provision, however, that the MOA-AD most clearly uses it to describe the
envisioned relationship between the BJE and the Central Government.

4. The relationship between the Central Government and the Bangsamoro juridical
entity shall be associative characterized by shared authority and responsibility with a
structure of governance based on executive, legislative, judicial and administrative
institutions with defined powers and functions in the comprehensive compact. A period
of transition shall be established in a comprehensive peace compact specifying the
relationship between the Central Government and the BJE. The nature of the
―associative relationship may have been intended to be defined more precisely in the
still to be forged Comprehensive Compact. Nonetheless, given that there is a concept of
―association in international law, and the MOA-AD – by its inclusion of international
law instruments in its TOR– placed itself in an international legal context, that concept
of association may be brought to bear in understanding the use of the term ―associative
in the MOA-AD. The MOA-AD contains many provisions which are consistent with the
international legal concept of association, specifically the following: the BJE‘s capacity
to enter into economic and trade relations with foreign countries, the commitment of
the Central Government to ensure the BJE‘s participation in meetings and events in the
ASEAN and the specialized UN agencies, and the continuing responsibility of the
Central Government over external defense. Moreover, the BJE‘s right to participate in
Philippine official missions bearing on negotiation of border agreements, environmental
protection, and sharing of revenues pertaining to the bodies of water adjacent to or
between the islands forming part of the ancestral domain, resembles the right of the
governments of FSM and the Marshall Islands to be consulted by the U.S. government
on any foreign affairs matter affecting them. These provisions of the MOA indicate,
among other things, that the Parties aimed to vest in the BJE the status of an associated
state or, at any rate, a status closely approximating it. The concept of association is not
recognized under the present Constitution. No province, city, or municipality, not even
the ARMM, is recognized under our laws as having an ―associative‖ relationship with
the national government. Indeed, the concept implies powers that go beyond anything
ever granted by the Constitution to any local or regional government. It also implies the
recognition of the associated entity as a state. The Constitution, however, does not
contemplate any state in this jurisdiction other than the Philippine State, much less does
it provide for a transitory status that aims to prepare any part of Philippine territory for
independence.
Even the mere concept animating many of the MOA-AD‘s provisions, therefore, already
requires for its validity the amendment of constitutional provisions, specifically the
following provisions of Article X:
SECTION 1. The territorial and political subdivisions of the Republic of the Philippines
are the provinces, cities, municipalities, and barangays. There shall be autonomous
regions in Muslim Mindanao and the Cordilleras as hereinafter provided. SECTION 15.
There shall be created autonomous regions in Muslim Mindanao and in the Cordilleras
consisting of provinces, cities, municipalities, and geographical areas sharing common
and distinctive historical and cultural heritage, economic and social structures, and
other relevant characteristics within the framework of this Constitution and the national
sovereignty as well as territorial integrity of the Republic of the Philippines.
It is not merely an expanded version of the ARMM, the status of its relationship with the
national government being fundamentally different from that of the ARMM. Indeed,
BJE is a state in all but name as it meets the criteria of a state laid down in the
Montevideo Convention, namely, a permanent population, a defined territory, a
government, and a capacity to enter into relations with other states.
The defining concept underlying the relationship between the national government and
the BJE being itself contrary to the present Constitution, it is not surprising that many
of the specific provisions of the M OA-AD on the formation and powers of the BJE are in
conflict with the Constitution and the laws. Article X, Section 18 of the Constitution
provides that ―[t]he creation of the autonomous region shall be effective when
approved by a majority of the votes cast by the constituent units in a plebiscite called for
the purpose, provided that only provinces, cities, and geographic areas voting favorably
in such plebiscite shall be included in the autonomous region.
The BJE is more of a state than an autonomous region. But even assuming that it is
covered by the term ―autonomous region in the constitutional provision just quoted,
the MOA-AD would still be in conflict with it. Under paragraph 2(c) on TERRITORY in
relation to 2(d) and 2(e), the present geographic area of the ARMM and, in addition, the
municipalities of Lanao del Norte which voted for inclusion in the ARMM during the
2001 plebiscite – Baloi, Munai, Nunungan, Pantar, Tagoloan and Tangkal – are
automatically part of the BJE without need of another plebiscite, in contrast to the areas
under Categories A and B mentioned earlier in the overview. That the present
components of the ARMM and the above-mentioned municipalities voted for inclusion
therein in 2001, however, does not render another plebiscite unnecessary under the
Constitution, precisely because what these areas voted for then was their inclusion in
the ARMM, not the BJE.
Article II, Section 22 of the Constitution must also be amended if the scheme envisioned
in the MOA-AD is to be effected. That constitutional provision states: ―The State
recognizes and promotes the rights of indigenous cultural communities within the
framework of national unity and development. An associative arrangement does not
uphold national unity. While there may be a semblance of unity because of the
associative ties between the BJE and the national government, the act of placing a
portion of Philippine territory in a status which, in international practice, has generally
been a preparation for independence, is certainly not conducive to national unity.
The MOA-AD cannot be reconciled with the present Constitution and laws. Not only its
specific provisions but the very concept underlying them, namely, the associative
relationship envisioned between the GRP and the BJE, are unconstitutional, for the
concept presupposes that the associated entity is a state and implies that the same is on
its way to independence.
While there is a clause in the MOA-AD stating that the provisions thereof inconsistent
with the present legal framework will not be effective until that framework is amended,
the same does not cure its defect. The inclusion of provisions in the MOA-AD
establishing an associative relationship between the BJE and the Central Government is,
itself, a violation of the Memorandum of Instructions from the President dated March 1,
2001, addressed to the government peace panel. Moreover, as the clause is worded, it
virtually guarantees that the necessary amendments to the Constitution and the laws
will eventually be put in place. Neither the GRP Peace Panel nor the President herself is
authorized to make such a guarantee. Upholding such an act would amount to
authorizing a usurpation of the constituent powers vested only in Congress, a
Constitutional Convention, or the people themselves through the process of initiative,
for the only way that the Executive can ensure the outcome of the amendment process is
through an undue influence or interference with that process.
PRESS RELEASE
THE SOUTH CHINA SEA ARBITRATION
(THE REPUBLIC OF THE PHILIPPINES V . THE PEOPLE’ S REPUBLIC OF CHINA )

The Hague, 12 July 2016

The Tribunal Renders Its Award

A unanimous Award has been issued today by the Tribunal constituted under Annex VII to the
UnitedNations Convention on the Law of the Sea (the “Convention”) in the arbitration instituted by
the Republic of the Philippines against the People’s Republic of China.

This arbitration concerned the role of historic rights and the source of maritime entitlements in the
South China Sea, the status of certain maritime features and the maritime entitlements they are
capable of generating, and the lawfulness of certain actions by China that were alleged by the
Philippines to violate the Convention. In light of limitations on compulsory dispute settlement under
the Convention, the Tribunal has emphasized that it does not rule on any question of sovereignty over
land territory and does not delimit any boundary between the Parties.

China has repeatedly stated that “it will neither accept nor participate in the arbitration unilaterally
initiated by the Philippines.” Annex VII, however, provides that the “[a]bsence of a party or failure
of a party to defend its case shall not constitute a bar to the proceedings.” Annex VII also provides
that, in the event that a party does not participate in the proceedings, a tribunal “must satisfy itself not
only that it has jurisdiction over the dispute but also that the claim is well founded in fact and law.”
Accordingly, throughout these proceedings, the Tribunal has taken steps to test the accuracy of the
Philippines’ claims, including by requesting further written submissions from the Philippines, by
questioning the Philippines both prior to and during two hearings, by appointing independent experts
to report to the Tribunal on technical matters, and by obtaining historical evidence concerning
features in the South China Sea and providing it to the Parties for comment.

China has also made clear—through the publication of a Position Paper in December 2014 and in
other official statements—that, in its view, the Tribunal lacks jurisdiction in this matter. Article 288
of the Convention provides that: “In the event of a dispute as to whether a court or tribunal has
jurisdiction, the matter shall be settled by decision of that court or tribunal.” Accordingly, the
Tribunal convened a hearing on jurisdiction and admissibility in July 2015 and rendered an Award on
Jurisdiction and Admissibility on 29 October 2015, deciding some issues of jurisdiction and
deferring others for further consideration. The Tribunal then convened a hearing on the merits from
24 to 30 November 2015.

The Award of today’s date addresses the issues of jurisdiction not decided in the Award on
Jurisdiction and Admissibility and the merits of the Philippines’ claims over which the Tribunal has
jurisdiction. The Award is final and binding, as set out in Article 296 of the Convention and Article
11 of Annex VII.

Historic Rights and the ‘Nine-Dash Line’: The Tribunal found that it has jurisdiction to consider
the Parties’ dispute concerning historic rights and the source of maritime entitlements in the South
China Sea. On the merits, the Tribunal concluded that the Convention comprehensively allocates
rights to maritime areas and that protections for pre-existing rights to resources were considered, but
not adopted in the Convention. Accordingly, the Tribunal concluded that, to the extent China had
historic rights to resources in the waters of the South China Sea, such rights were extinguished to the
extent they were incompatible with the exclusive economic zones provided for in the Convention.
The Tribunal also noted that, although Chinese navigators and fishermen, as well as those of other
States, had historically made use of the islands in the South China Sea, there was no evidence that
China had historically exercised exclusive control over the waters or their resources. The Tribunal
concluded that there was no legal basis for China to claim historic rights to resources within the sea
areas falling within the ‘nine-dash line’.

Status of Features: The Tribunal next considered entitlements to maritime areas and the status of
features. The Tribunal first undertook an evaluation of whether certain reefs claimed by China are
above water at high tide. Features that are above water at high tide generate an entitlement to at least
a 12 nautical mile territorial sea, whereas features that are submerged at high tide do not. The
Tribunal noted that the reefs have been heavily modified by land reclamation and construction,
recalled that the Convention classifies features on their natural condition, and relied on historical
materials in evaluating the features. The Tribunal then considered whether any of the features
claimed by China could generate maritime zones beyond 12 nautical miles. Under the Convention,
islands generate an exclusive economic zone of 200 nautical miles and a continental shelf, but
“[r]ocks which cannot sustain human habitation or economic life of their own shall have no exclusive
economic zone or continental shelf.” The Tribunal concluded that this provision depends upon the
objective capacity of a feature, in its natural condition, to sustain either a stable community of people
or economic activity that is not dependent on outside resources or purely extractive in nature. The
Tribunal noted that the current presence of official personnel on many of the features is dependent on
outside support and not reflective of the capacity of the features. The Tribunal found historical
evidence to be more relevant and noted that the Spratly Islands were historically used by small
groups of fishermen and that several Japanese fishing and guano mining enterprises were attempted.
The Tribunal concluded that such transient use does not constitute inhabitation by a stable
community and that all of the historical economic activity had been extractive. Accordingly, the
Tribunal concluded that none of the Spratly Islands is capable of generating extended maritime
zones. The Tribunal also held that the Spratly Islands cannot generate maritime zones collectively as
a unit. Having found that none of the features claimed by China was capable of generating an
exclusive economic zone, the Tribunal found that it could—without delimiting a boundary—declare
that certain sea areas are within the exclusive economic zone of the Philippines, because those areas
are not overlapped by any possible entitlement of China.

Lawfulness of Chinese Actions: The Tribunal next considered the lawfulness of Chinese actions in
the
South China Sea. Having found that certain areas are within the exclusive economic zone of the
Philippines, the Tribunal found that China had violated the Philippines’ sovereign rights in its
exclusive economic zone by (a) interfering with Philippine fishing and petroleum exploration, (b)
constructing artificial islands and (c) failing to prevent Chinese fishermen from fishing in the zone.
The Tribunal also held that fishermen from the Philippines (like those from China) had traditional
fishing rights at Scarborough Shoal and that China had interfered with these rights in restricting
access. The Tribunal further held that Chinese law enforcement vessels had unlawfully created a
serious risk of collision when they physically obstructed Philippine vessels.

Harm to Marine Environment: The Tribunal considered the effect on the marine environment of
China’s recent large-scale land reclamation and construction of artificial islands at seven features in
the Spratly Islands and found that China had caused severe harm to the coral reef environment and
violated its obligation to preserve and protect fragile ecosystems and the habitat of depleted,
threatened, or endangered species. The Tribunal also found that Chinese authorities were aware that
Chinese fishermen have harvested endangered sea turtles, coral, and giant clams on a substantial
scale in the South China Sea (using methods that inflict severe damage on the coral reef
environment) and had not fulfilled their obligations to stop such activities.

Aggravation of Dispute: Finally, the Tribunal considered whether China’s actions since the
commencement of the arbitration had aggravated the dispute between the Parties. The Tribunal found
that it lacked jurisdiction to consider the implications of a stand-off between Philippine marines and
Chinese naval and law enforcement vessels at Second Thomas Shoal, holding that this dispute
involved military activities and was therefore excluded from compulsory settlement. The Tribunal
found, however, that China’s recent large-scale land reclamation and construction of artificial islands
was incompatible with the obligations on a State during dispute resolution proceedings, insofar as
China has inflicted irreparable harm to the marine environment, built a large artificial island in the
Philippines’ exclusive economic zone, and destroyed evidence of the natural condition of features in
the South China Sea that formed part of the Parties’ dispute.

An expanded summary of the Tribunal’s decisions is set out below.

The Tribunal was constituted on 21 June 2013 pursuant to the procedure set out in Annex VII of
the Convention to decide the dispute presented by the Philippines. The Tribunal is composed of
Judge Thomas A. Mensah of Ghana, Judge Jean-Pierre Cot of France, Judge Stanislaw Pawlak of
Poland, Professor Alfred H.A. Soons of the Netherlands, and Judge Rüdiger Wolfrum of Germany.
Judge Thomas A. Mensah serves as President of the Tribunal. The Permanent Court of Arbitration
acts as the Registry in the proceedings.

Further information about the case may be found at www.pcacases.com/web/view/7, including the
Award on Jurisdiction and Admissibility, the Rules of Procedure, earlier Press Releases, hearing
transcripts, and photographs. Procedural Orders, submissions by the Philippines, and reports by the
Tribunal’s experts will be made available in due course, as will unofficial Chinese translations of the
Tribunal’s Awards.

Background to the Permanent Court of Arbitration

The Permanent Court of Arbitration (PCA) is an intergovernmental organization established by the


1899 Hague Convention on the Pacific Settlement of International Disputes. The PCA has 121
Member States. Headquartered at the Peace Palace in The Hague, the Netherlands, the PCA
facilitates arbitration, conciliation, fact-finding, and other dispute resolution proceedings among
various combinations of States, State entities, intergovernmental organizations, and private parties.
The PCA’s International Bureau is currently administering 8 interstate disputes, 73 investor-State
arbitrations, and 34 cases arising under contracts involving a State or other public entity. The PCA
has administered 12 cases initiated by States under Annex VII to the United Nations Convention on
the Law of the Sea.

In July 2013, the Tribunal in the South China Sea Arbitration appointed the PCA to serve as Registry
for the proceedings. The Tribunal’s Rules of Procedure provide that the PCA shall “maintain an
archive of the arbitral proceedings and provide appropriate registry services as directed by the
Arbitral Tribunal.” Such services include assisting with the identification and appointment of experts;
publishing information about the arbitration and issuing press releases; organizing the hearings at the
Peace Palace in The Hague; and the financial management of the case, which involves holding a
deposit for expenses in the arbitration, such as to pay arbitrator fees, experts, technical support, court
reporters etc. The Registry also serves as the channel of communications amongst the Parties and the
Tribunal and observer States.

Photograph: Hearing in session, July 2015, Peace Palace, The Hague. Clockwise from top left: Registrar
and PCA Senior Legal Counsel Judith Levine; Judge Stanislaw Pawlak; Professor Alfred H. A. Soons; Judge
Thomas A. Mensah (Presiding Arbitrator); Judge Jean-Pierre Cot; Judge Rüdiger Wolfrum; PCA Senior Legal
Counsel Garth Schofield; former Secretary for Foreign Affairs of the Philippines, Mr. Albert F. Del Rosario;
former Solicitor General Mr. Florin T. Hilbay, Counsel for the Philippines; Mr. Paul S. Reichler; Professor
Philippe Sands; Professor Bernard H. Oxman; Professor Alan E. Boyle; Mr. Lawrence H. Martin.

SUMMARY OF THE TRIBUNAL’S DECISIONS ON ITS JURISDICTION


AND ON THE MERITS OF THE PHILIPPINES ’ CLAIMS

1. Background to the Arbitration

The South China Sea Arbitration between the Philippines and China concerned an application by the
Philippines for rulings in respect of four matters concerning the relationship between the Philippines
and China in the South China Sea. First, the Philippines sought a ruling on the source of the Parties’
rights and obligations in the South China Sea and the effect of the United Nations Convention on the
Law of the Sea (“Convention”) on China’s claims to historic rights within its so-called ‘nine-dash
line’. Second, the Philippines sought a ruling on whether certain maritime features claimed by both
China and the Philippines are properly characterized as islands, rocks, low-tide elevations or
submerged banks under the Convention. The status of these features under the Convention
determines the maritime zones they are capable of generating. Third, the Philippines sought rulings
on whether certain Chinese actions in the South China Sea have violated the Convention, by
interfering with the exercise of the Philippines’ sovereign rights and freedoms under the Convention
or through construction and fishing activities that have harmed the marine environment. Finally, the
Philippines sought a ruling that certain actions taken by China, in particular its large-scale land
reclamation and construction of artificial islands in the Spratly Islands since this arbitration was
commenced, have unlawfully aggravated and extended the Parties’ dispute.

The Chinese Government has adhered to the position of neither accepting nor participating in these
arbitral proceedings. It has reiterated this position in diplomatic notes, in the “Position Paper of the
Government of the People’s Republic of China on the Matter of Jurisdiction in the South China Sea
Arbitration Initiated by the Republic of the Philippines” dated 7 December 2014 (“China’s Position
Paper”), in letters to members of the Tribunal from the Chinese Ambassador to the Kingdom of the
Netherlands, and in many public statements. The Chinese Government has also made clear that these
statements and documents “shall by no means be interpreted as China’s participation in the arbitral
proceeding in any form.”

Two provisions of the Convention address the situation of a party that objects to the jurisdiction of a
tribunal and declines to participate in the proceedings:

(a) Article 288 of the Convention provides that: “In the event of a dispute as to whether a court or
tribunal has jurisdiction, the matter shall be settled by decision of that court or tribunal.”

(b) Article 9 of Annex VII to the Convention provides that:

If one of the parties to the dispute does not appear before the arbitral tribunal or fails to defend its
case, the other party may request the tribunal to continue the proceedings and to make its award.
Absence of a party or failure of a party to defend its case shall not constitute a bar to the proceedings.
Before making its award, the arbitral tribunal must satisfy itself not only that it has jurisdiction over
the dispute but also that the claim is well founded in fact and law.

Throughout these proceedings, the Tribunal has taken a number of steps to fulfil its duty to satisfy
itself as to whether it has jurisdiction and whether the Philippines’ claims are “well founded in fact
and law”. With respect to jurisdiction, the Tribunal decided to treat China’s informal
communications as equivalent to an objection to jurisdiction, convened a Hearing on Jurisdiction and
Admissibility on 7 to 13 July 2015, questioned the Philippines both before and during the hearing on
matters of jurisdiction, including potential issues not raised in China’s informal communications, and
issued an Award on Jurisdiction and Admissibility on 29 October 2015 (the “Award on
Jurisdiction”), deciding some issues of jurisdiction and deferring others for further consideration in
conjunction with the merits of the Philippines’ claims. With respect to the merits, the Tribunal sought
to test the accuracy of the Philippines’ claims by requesting further written submissions from the
Philippines, by convening a hearing on the merits from 24 to 30 November 2015, by questioning the
Philippines both before and during the hearing with respect to its claims, by appointing independent
experts to report to the Tribunal on technical matters, and by obtaining historical records and
hydrographic survey data for the South China Sea from the archives of the United Kingdom
Hydrographic Office, the National Library of France, and the French National Overseas Archives and
providing it to the Parties for comment, along with other relevant materials in the public domain.

2. The Parties’ Positions

The Philippines made 15 Submissions in these proceedings, requesting the Tribunal to find that:

(1) China’s maritime entitlements in the South China Sea, like those of the Philippines, may not
extend beyond those expressly permitted by the United Nations Convention on the Law of the Sea;

(2) China’s claims to sovereign rights jurisdiction, and to “historic rights”, with respect to the
maritime areas of the South China Sea encompassed by the so-called “nine-dash line” are contrary to
the Convention and without lawful effect to the extent that they exceed the geographic and
substantive limits of China’s maritime entitlements expressly permitted by UNCLOS;

(3) Scarborough Shoal generates no entitlement to an exclusive economic zone or continental shelf;

(4) Mischief Reef, Second Thomas Shoal, and Subi Reef are low-tide elevations that do not generate
entitlement to a territorial sea, exclusive economic zone or continental shelf, and are not features that
are capable of appropriation by occupation or otherwise;

(5) Mischief Reef and Second Thomas Shoal are part of the exclusive economic zone and continental
shelf of the Philippines;

(6) Gaven Reef and McKennan Reef (including Hughes Reef) are low-tide elevations that do not
generate entitlement to a territorial sea, exclusive economic zone or continental shelf, but their low-
water line may be used to determine the baseline from which the breadth of the territorial sea of
Namyit and Sin Cowe, respectively, is measured;

(7) Johnson Reef, Cuarteron Reef and Fiery Cross Reef generate no entitlement to an exclusive
economic zone or continental shelf;

(8) China has unlawfully interfered with the enjoyment and exercise of the sovereign rights of the
Philippines with respect to the living and non-living resources of its exclusive economic zone and
continental shelf;

(9) China has unlawfully failed to prevent its nationals and vessels from exploiting the living
resources in the exclusive economic zone of the Philippines;

(10) China has unlawfully prevented Philippine fishermen from pursuing their livelihoods by
interfering with traditional fishing activities at Scarborough Shoal;

(11) China has violated its obligations under the Convention to protect and preserve the marine
environment at Scarborough Shoal, Second Thomas Shoal, Cuarteron Reef, Fiery Cross Reef, Gaven
Reef, Johnson Reef, Hughes Reef and Subi Reef;

(12) China’s occupation of and construction activities on Mischief Reef

(a) violate the provisions of the Convention concerning artificial islands, installations and structures;
(b) violate China’s duties to protect and preserve the marine environment under the Convention; and

(c) constitute unlawful acts of attempted appropriation in violation of the Convention;

(13) China has breached its obligations under the Convention by operating its law enforcement
vessels in a dangerous manner, causing serious risk of collision to Philippine vessels navigating in
the vicinity of Scarborough Shoal;

(14) Since the commencement of this arbitration in January 2013, China has unlawfully aggravated
and extended the dispute by, among other things:

(a) interfering with the Philippines’ rights of navigation in the waters at, and adjacent to, Second
Thomas Shoal;

(b) preventing the rotation and resupply of Philippine personnel stationed at Second Thomas Shoal;

(c) endangering the health and well-being of Philippine personnel stationed at Second Thomas
Shoal; and

(d) conducting dredging, artificial island-building and construction activities at Mischief Reef,
Cuarteron Reef, Fiery Cross Reef, Gaven Reef, Johnson Reef, Hughes Reef and Subi Reef; and

(15) China shall respect the rights and freedoms of the Philippines under the Convention, shall
comply with its duties under the Convention, including those relevant to the protection and
preservation of the marine environment in the South China Sea, and shall exercise its rights and
freedoms in the South China Sea with due regard to those of the Philippines under the Convention.

With respect to jurisdiction, the Philippines has asked the Tribunal to declare that the Philippines’
claims “are entirely within its jurisdiction and are fully admissible.”

China does not accept and is not participating in this arbitration but stated its position that the
Tribunal “does not have jurisdiction over this case.” In its Position Paper, China advanced the
following arguments:

 The essence of the subject-matter of the arbitration is the territorial sovereignty over several
maritime features in the South China Sea, which is beyond the scope of the Convention and
does not concern the interpretation or application of the Convention;
 China and the Philippines have agreed, through bilateral instruments and the Declaration on
the Conduct of Parties in the South China Sea, to settle their relevant disputes through
negotiations. By unilaterally initiating the present arbitration, the Philippines has breached its
obligation under international law;
 Even assuming, arguendo, that the subject-matter of the arbitration were concerned with the
interpretation or application of the Convention, that subject-matter would constitute an
integral part of maritime delimitation between the two countries, thus falling within the scope
of the declaration filed by China in 2006 in accordance with the Convention, which excludes,
inter alia, disputes concerning maritime delimitation from compulsory arbitration and other
compulsory dispute settlement procedures;
Although China has not made equivalent public statements with respect to the merits of the majority
of the Philippines’ claims, the Tribunal has sought throughout the proceedings to ascertain China’s
position on the basis of its contemporaneous public statements and diplomatic correspondence.

3. The Tribunal’s Decisions on the Scope of its Jurisdiction

The Tribunal has addressed the scope of its jurisdiction to consider the Philippines’ claims both in its
Award on Jurisdiction, to the extent that issues of jurisdiction could be decided as a preliminary
matter, and in its Award of 12 July 2016, to the extent that issues of jurisdiction were intertwined
with the merits of the Philippines’ claims. The Tribunal’s Award of 12 July 2016 also incorporates
and reaffirms the decisions on jurisdiction taken in the Award on Jurisdiction.

For completeness, the Tribunal’s decisions on jurisdiction in both awards are summarized here
together.

a. Preliminary Matters

In its Award on Jurisdiction, the Tribunal considered a number of preliminary matters with respect to
its jurisdiction. The Tribunal noted that both the Philippines and China are parties to the Convention
and that the Convention does not permit a State to except itself generally from the mechanism for the
resolution of disputes set out in the Convention. The Tribunal held that China’s non-participation
does not deprive the Tribunal of jurisdiction and that the Tribunal had been properly constituted
pursuant to the provisions of Annex VII to the Convention, which include a procedure to form a
tribunal even in the absence of one party. Finally, the Tribunal rejected an argument set out in
China’s Position Paper and held that the mere act of unilaterally initiating an arbitration cannot
constitute an abuse of the Convention.

b. Existence of a Dispute Concerning Interpretation and Application of the Convention

In its Award on Jurisdiction, the Tribunal considered whether the Parties’ disputes concerned the
interpretation or application of the Convention, which is a requirement for resort to the dispute
settlement mechanisms of the Convention.

The Tribunal rejected the argument set out in China’s Position Paper that the Parties’ dispute is
actually about territorial sovereignty and therefore not a matter concerning the Convention. The
Tribunal accepted that there is a dispute between the Parties concerning sovereignty over islands in
the South China Sea, but held that the matters submitted to arbitration by the Philippines do not
concern sovereignty. The Tribunal considered that it would not need to implicitly decide sovereignty
to address the Philippines’ Submissions and that doing so would not advance the sovereignty claims
of either Party to islands in the South China Sea.

The Tribunal also rejected the argument set out in China’s Position Paper that the Parties’ dispute is
actually about maritime boundary delimitation and therefore excluded from dispute settlement by
Article 298 of the Convention and a declaration that China made on 25 August 2006 pursuant to that
Article. The Tribunal noted that a dispute concerning whether a State has an entitlement to a
maritime zone is a distinct matter from the delimitation of maritime zones in an area in which they
overlap. The Tribunal noted that entitlements, together with a wide variety of other issues, are
commonly considered in a boundary delimitation, but can also arise in other contexts. The Tribunal
held that it does not follow that a dispute over each of these issues is necessarily a dispute over
boundary delimitation.
Finally, the Tribunal held that each of the Philippines’ Submissions reflected a dispute concerning
the Convention. In doing so, the Tribunal emphasized (a) that a dispute concerning the interaction
between the Convention and other rights (including any Chinese “historic rights”) is a dispute
concerning the Convention and (b) that where China has not clearly stated its position, the existence
of a dispute may be inferred from the conduct of a State or from silence and is a matter to be
determined objectively.

c. Involvement of Indispensable Third-Parties

In its Award on Jurisdiction, the Tribunal considered whether the absence from this arbitration of
other States that have made claims to the islands of the South China Sea would be a bar to the
Tribunal’s jurisdiction. The Tribunal noted that the rights of other States would not form “the very
subject-matter of the decision,” the standard for a third-party to be indispensable. The Tribunal
further noted that in December 2014, Viet Nam had submitted a statement to the Tribunal, in which
Viet Nam asserted that it has “no doubt that the Tribunal has jurisdiction in these proceedings.” The
Tribunal also noted that Viet Nam, Malaysia, and Indonesia had attended the hearing on jurisdiction
as observers, without any State raising the argument that its participation was indispensable.

In its Award of 12 July 2016, the Tribunal noted that it had received a communication from Malaysia
on 23 June 2016, recalling Malaysia’s claims in the South China Sea. The Tribunal compared its
decisions on the merits of the Philippines’ Submissions with the rights claimed by Malaysia and
reaffirmed its decision that Malaysia is not an indispensable party and that Malaysia’s interests in the
South China Sea do not prevent the Tribunal from addressing the Philippines’ Submissions.

d. Preconditions to Jurisdiction

In its Award on Jurisdiction, the Tribunal considered the applicability of Articles 281 and 282 of the
Convention, which may prevent a State from making use of the mechanisms under the Convention if
they have already agreed to another means of dispute resolution.

The Tribunal rejected the argument set out in China’s Position Paper that the 2002 China–ASEAN
Declaration on the Conduct of Parties in the South China Sea prevented the Philippines from
initiating arbitration. The Tribunal held that the Declaration is a political agreement and not legally
binding, does not provide a mechanism for binding settlement, does not exclude other means of
dispute settlement, and therefore does not restrict the Tribunal’s jurisdiction under Articles 281 or
282. The Tribunal also considered the Treaty of Amity and Cooperation in Southeast Asia, and the
Convention on Biological Diversity, and a series of joint statements issued by the Philippines and
China referring to the resolution of disputes through negotiations and concluded that none of these
instruments constitute an agreement that would prevent the Philippines from bringing its claims to
arbitration.

The Tribunal further held that the Parties had exchanged views regarding the settlement of their
disputes, as required by Article 283 of the Convention, before the Philippines initiated the arbitration.
The Tribunal concluded that this requirement was met in the record of diplomatic communications
between the Philippines and China, in which the Philippines expressed a clear preference for
multilateral negotiations involving the other States surrounding the South China Sea, while China
insisted that only bilateral talks could be considered.

e. Exceptions and Limitations to Jurisdiction


In its Award of 12 July 2016, the Tribunal considered whether the Philippines’ Submissions
concerning Chinese historic rights and the ‘nine-dash line’ were affected by the exception from
jurisdiction for disputes concerning “historic title” in Article 298 of the Convention. The Tribunal
reviewed the meaning of “historic title” in the law of the sea and held that this refers to claims of
historic sovereignty over bays and other near-shore waters. Reviewing China’s claims and conduct in
the South China Sea, the Tribunal concluded that China claims historic rights to resources within the
‘nine-dash line’, but does not claim historic title over the waters of the South China Sea.
Accordingly, the Tribunal concluded that it had jurisdiction to consider the Philippines’ claims
concerning historic rights and, as between the Philippines and China, the ‘nine-dash line’.

In its Award of 12 July 2016, the Tribunal also considered whether the Philippines’ Submissions
were affected by the exception from jurisdiction in Article 298 for disputes concerning sea boundary
delimitation. The Tribunal had already found in its Award on Jurisdiction that the Philippines’
Submissions do not concern boundary delimitation as such, but noted that several of the Philippines’
Submissions were dependent on certain areas forming part of the Philippines’ exclusive economic
zone. The Tribunal held that it could only address such submissions if there was no possibility that
China could have an entitlement to an exclusive economic zone overlapping that of the Philippines
and deferred a final decision on its jurisdiction. In its Award of 12 July 2016, the Tribunal reviewed
evidence about the reefs and islands claimed by China in the South China Sea and concluded that
none is capable of generating an entitlement to an exclusive economic zone. Because China has no
possible entitlement to an exclusive economic zone overlapping that of the Philippines in the Spratly
Islands, the Tribunal held that the Philippines’ submissions were not dependent on a prior
delimitation of a boundary.

In its Award of 12 July 2016, the Tribunal also considered whether the Philippines’ Submissions
were affected by the exception from jurisdiction in Article 298 for disputes concerning law
enforcement activities in the exclusive economic zone. The Tribunal recalled that the exception in
Article 298 would apply only if the Philippines’ Submissions related to law enforcement activities in
China’s exclusive economic zone. Because, however, the Philippines’ Submissions related to events
in the Philippines’ own exclusive economic zone or in the territorial sea, the Tribunal concluded that
Article 298 did not pose an obstacle to its jurisdiction.

Lastly, in its Award of 12 July 2016, the Tribunal considered whether the Philippines’ submissions
were affected by the exception from jurisdiction in Article 298 for disputes concerning military
activities. The Tribunal considered that the stand-off between Philippine marines on Second Thomas
Shoal and Chinese naval and law enforcement vessels constituted military activities and concluded
that it lacked jurisdiction over the Philippines’ Submission No. 14(a)-(c). The Tribunal also
considered whether China’s land reclamation and construction of artificial islands at seven features in
the Spratly Islands constituted military activities, but noted that China had repeatedly emphasized the
non-military nature of its actions and had stated at the highest level that it would not militarize its
presence in the Spratlys. The Tribunal decided that it would not deem activities to be military in
nature when China itself had repeatedly affirmed the opposite. Accordingly, the Tribunal concluded
that Article 298 did not pose an obstacle to its jurisdiction.

4. The Tribunal’s Decisions on the Merits of the Philippines’ Claims

a. The ‘Nine-Dash Line’ and China’s Claim to Historic Rights in the Maritime Areas of the
South China Sea
In its Award of 12 July 2016, the Tribunal considered the implications of China’s ‘nine-dash line’
and whether China has historic rights to resources in the South China Sea beyond the limits of the
maritime zones that it is entitled to pursuant to the Convention.

The Tribunal examined the history of the Convention and its provisions concerning maritime zones
and concluded that the Convention was intended to comprehensively allocate the rights of States to
maritime areas. The Tribunal noted that the question of pre-existing rights to resources (in particular
fishing resources) was carefully considered during the negotiations on the creation of the exclusive
economic zone and that a number of States wished to preserve historic fishing rights in the new zone.
This position was rejected, however, and the final text of the Convention gives other States only a
limited right of access to fisheries in the exclusive economic zone (in the event the coastal State
cannot harvest the full allowable catch) and no rights to petroleum or mineral resources. The
Tribunal found that China’s claim to historic rights to resources was incompatible with the detailed
allocation of rights and maritime zones in the Convention and concluded that, to the extent China had
historic rights to resources in the waters of the South China Sea, such rights were extinguished by the
entry into force of the Convention to the extent they were incompatible with the Convention’s system
of maritime zones.

The Tribunal also examined the historical record to determine whether China actually had historic
rights to resources in the South China Sea prior to the entry into force of the Convention. The
Tribunal noted that there is evidence that Chinese navigators and fishermen, as well as those of other
States, had historically made use of the islands in the South China Sea, although the Tribunal
emphasized that it was not empowered to decide the question of sovereignty over the islands.
However, the Tribunal considered that prior to the Convention, the waters of the South China Sea
beyond the territorial sea were legally part of the high seas, in which vessels from any State could
freely navigate and fish. Accordingly, the Tribunal concluded that historical navigation and fishing
by China in the waters of the South China Sea represented the exercise of high seas freedoms, rather
than a historic right, and that there was no evidence that China had historically exercised exclusive
control over the waters of the South China Sea or prevented other States from exploiting their
resources.

Accordingly, the Tribunal concluded that, as between the Philippines and China, there was no legal
basis for China to claim historic rights to resources, in excess of the rights provided for by the
Convention, within the sea areas falling within the ‘nine-dash line’.

b. The Status of Features in the South China Sea

In its Award of 12 July 2016, the Tribunal considered the status of features in the South China Sea
and the entitlements to maritime areas that China could potentially claim pursuant to the Convention.

The Tribunal first undertook a technical evaluation as to whether certain coral reefs claimed by China
are or are not above water at high tide. Under Articles 13 and 121 of the Convention, features that are
above water at high tide generate an entitlement to at least a 12 nautical mile territorial sea, whereas
features that are submerged at high tide generate no entitlement to maritime zones. The Tribunal
noted that many of the reefs in the South China Sea have been heavily modified by recent land
reclamation and construction and recalled that the Convention classifies features on the basis of their
natural condition. The Tribunal appointed an expert hydrographer to assist it in evaluating the
Philippines’ technical evidence and relied heavily on archival materials and historical hydrographic
surveys in evaluating the features. The Tribunal agreed with the Philippines that Scarborough Shoal,
Johnson Reef, Cuarteron Reef, and Fiery Cross Reef are high-tide features and that Subi Reef,
Hughes Reef, Mischief Reef, and Second Thomas Shoal were submerged at high tide in their natural
condition. However, the Tribunal disagreed with the Philippines regarding the status of Gaven Reef
(North) and McKennan Reef and concluded that both are high tide features.

The Tribunal then considered whether any of the features claimed by China could generate an
entitlement to maritime zones beyond 12 nautical miles. Under Article 121 of the Convention,
islands generate an entitlement to an exclusive economic zone of 200 nautical miles and to a
continental shelf, but “[r]ocks which cannot sustain human habitation or economic life of their own
shall have no exclusive economic zone or continental shelf.” The Tribunal noted that this provision
was closely linked to the expansion of coastal State jurisdiction with the creation of the exclusive
economic zone and was intended to prevent insignificant features from generating large entitlements
to maritime zones that would infringe on the entitlements of inhabited territory or on the high seas
and the area of the seabed reserved for the common heritage of mankind. The Tribunal interpreted
Article 121 and concluded that the entitlements of a feature depend on (a) the objective capacity of a
feature, (b) in its natural condition, to sustain either (c) a stable community of people or (d) economic
activity that is neither dependent on outside resources nor purely extractive in nature.

The Tribunal noted that many of the features in the Spratly Islands are currently controlled by one or
another of the littoral States, which have constructed installations and maintain personnel there. The
Tribunal considered these modern presences to be dependent on outside resources and support and
noted that many of the features have been modified to improve their habitability, including through
land reclamation and the construction of infrastructure such as desalination plants. The Tribunal
concluded that the current presence of official personnel on many of the features does not establish
their capacity, in their natural condition, to sustain a stable community of people and considered that
historical evidence of habitation or economic life was more relevant to the objective capacity of the
features. Examining the historical record, the Tribunal noted that the Spratly Islands were historically
used by small groups of fishermen from China, as well as other States, and that several Japanese
fishing and guano mining enterprises were attempted in the 1920s and 1930s. The Tribunal
concluded that temporary use of the features by fishermen did not amount to inhabitation by a stable
community and that all of the historical economic activity had been extractive in nature.
Accordingly, the Tribunal concluded that all of the high-tide features in the Spratly Islands
(including, for example, Itu Aba, Thitu, West York Island, Spratly Island, North-East Cay, South-
West Cay) are legally “rocks” that do not generate an exclusive economic zone or continental shelf.

The Tribunal also held that the Convention does not provide for a group of islands such as the
Spratly Islands to generate maritime zones collectively as a unit.

c. Chinese Activities in the South China Sea

In its Award of 12 July 2016, the Tribunal considered the lawfulness under the Convention of various
Chinese actions in the South China Sea.

Having found that Mischief Reef, Second Thomas Shoal and Reed Bank are submerged at high tide,
form part of the exclusive economic zone and continental shelf of the Philippines, and are not
overlapped by any possible entitlement of China, the Tribunal concluded that the Convention is clear
in allocating sovereign rights to the Philippines with respect to sea areas in its exclusive economic
zone. The Tribunal found as a matter of fact that China had (a) interfered with Philippine petroleum
exploration at Reed Bank, (b) purported to prohibit fishing by Philippine vessels within the
Philippines’ exclusive economic zone, (c) protected and failed to prevent Chinese fishermen from
fishing within the Philippines’ exclusive economic zone at Mischief Reef and Second Thomas Shoal,
and (d) constructed installations and artificial islands at Mischief Reef without the authorization of
the Philippines. The Tribunal therefore concluded that China had violated the Philippines’ sovereign
rights with respect to its exclusive economic zone and continental shelf.

The Tribunal next examined traditional fishing at Scarborough Shoal and concluded that fishermen
from the Philippines, as well as fishermen from China and other countries, had long fished at the
Shoal and had traditional fishing rights in the area. Because Scarborough Shoal is above water at
high tide, it generates an entitlement to a territorial sea, its surrounding waters do not form part of the
exclusive economic zone, and traditional fishing rights were not extinguished by the Convention.
Although the Tribunal emphasized that it was not deciding sovereignty over Scarborough Shoal, it
found that China had violated its duty to respect to the traditional fishing rights of Philippine
fishermen by halting access to the Shoal after May 2012. The Tribunal noted, however, that it would
reach the same conclusion with respect to the traditional fishing rights of Chinese fishermen if the
Philippines were to prevent fishing by Chinese nationals at Scarborough Shoal.

The Tribunal also considered the effect of China’s actions on the marine environment. In doing so,
the Tribunal was assisted by three independent experts on coral reef biology who were appointed to
assist it in evaluating the available scientific evidence and the Philippines’ expert reports. The
Tribunal found that China’s recent large scale land reclamation and construction of artificial islands
at seven features in the Spratly Islands has caused severe harm to the coral reef environment and that
China has violated its obligation under Articles 192 and 194 of the Convention to preserve and
protect the marine environment with respect to fragile ecosystems and the habitat of depleted,
threatened, or endangered species. The Tribunal also found that Chinese fishermen have engaged in
the harvesting of endangered sea turtles, coral, and giant clams on a substantial scale in the South
China Sea, using methods that inflict severe damage on the coral reef environment. The Tribunal
found that Chinese authorities were aware of these activities and failed to fulfill their due diligence
obligations under the Convention to stop them.

Finally, the Tribunal considered the lawfulness of the conduct of Chinese law enforcement vessels at
Scarborough Shoal on two occasions in April and May 2012 when Chinese vessels had sought to
physically obstruct Philippine vessels from approaching or gaining entrance to the Shoal. In doing so,
the Tribunal was assisted by an independent expert on navigational safety who was appointed to
assist it in reviewing the written reports provided by the officers of the Philippine vessels and the
expert evidence on navigational safety provided by the Philippines. The Tribunal found that Chinese
law enforcement vessels had repeatedly approached the Philippine vessels at high speed and sought
to cross ahead of them at close distances, creating serious risk of collision and danger to Philippine
ships and personnel. The Tribunal concluded that China had breached its obligations under the
Convention on the International Regulations for Preventing Collisions at Sea, 1972, and Article 94
the Convention concerning maritime safety.

d. Aggravation of the Dispute between the Parties

In its Award of 12 July 2016, the Tribunal considered whether China’s recent large-scale land
reclamation and construction of artificial islands at seven features in the Spratly Islands since the
commencement of the arbitration had aggravated the dispute between the Parties. The Tribunal
recalled that there exists a duty on parties engaged in a dispute settlement procedure to refrain from
aggravating or extending the dispute or disputes at issue during the pendency of the settlement
process. The Tribunal noted that China has (a) built a large artificial island on Mischief Reef, a low-
tide elevation located in the exclusive economic zone of the Philippines; (b) caused permanent,
irreparable harm to the coral reef ecosystem and (c) permanently destroyed evidence of the natural
condition of the features in question. The Tribunal concluded that China had violated its obligations
to refrain from aggravating or extending the Parties’ disputes during the pendency of the settlement
process.

e. Future Conduct of the Parties

Finally, the Tribunal considered the Philippines’ request for a declaration that, going forward, China
shall respect the rights and freedoms of the Philippines and comply with its duties under the
Convention. In this respect, the Tribunal noted that both the Philippines and China have repeatedly
accepted that the Convention and general obligations of good faith define and regulate their conduct.
The Tribunal considered that the root of the disputes at issue in this arbitration lies not in any
intention on the part of China or the Philippines to infringe on the legal rights of the other, but rather
in fundamentally different understandings of their respective rights under the Convention in the
waters of the South China Sea. The Tribunal recalled that it is a fundamental principle of
international law that bad faith is not presumed and noted that Article 11 of Annex VII provides that
the “award . . . shall be complied with by the parties to the dispute.” The Tribunal therefore
considered that no further declaration was necessary.
Francisco Chavez vs Public
Estates Authority (July 2002)
February 27, 2012
Share this...

4 0 0 0

ADVERTISEMENTS

384 SCRA 152 – Civil Law – Land Titles and Deeds – Lands of the Public Domain
The Public Estates Authority (PEA) is the central implementing agency tasked to undertake
reclamation projects nationwide. It took over the leasing and selling functions of the DENR
(Department of Environmental and Natural Resources) insofar as reclaimed or about to be
reclaimed foreshore lands are concerned.
PEA sought the transfer to the Amari Coastal Bay and Development Corporation, a private
corporation, of the ownership of 77.34 hectares of the Freedom Islands. PEA also sought to
have 290.156 hectares of submerged areas of Manila Bay to Amari.
ISSUE: Whether or not the transfer is valid.
HELD: No. To allow vast areas of reclaimed lands of the public domain to be transferred to
Amari as private lands will sanction a gross violation of the constitutional ban on private
corporations from acquiring any kind of alienable land of the public domain.
The Supreme Court affirmed that the 157.84 hectares of reclaimed lands comprising the
Freedom Islands, now covered by certificates of title in the name of PEA, are alienable
lands of the public domain. The 592.15 hectares of submerged areas of Manila Bay remain
inalienable natural resources of the public domain. The transfer (as embodied in a joint
venture agreement) to AMARI, a private corporation, ownership of 77.34 hectares of the
Freedom Islands, is void for being contrary to Section 3, Article XII of the 1987 Constitution
which prohibits private corporations from acquiring any kind of alienable land of the public
domain. Furthermore, since the Amended JVA also seeks to transfer to Amari ownership of
290.156 hectares of still submerged areas of Manila Bay, such transfer is void for being
contrary to Section 2, Article XII of the 1987 Constitution which prohibits the alienation of
natural resources other than agricultural lands of the public domain.
Chavez v. Pea and Amari

Fact:
In 1973, the Comissioner on Public Highways entered into a contract to reclaim areas of Manila Bay
with the Construction and Development Corportion of the Philippines (CDCP).

PEA (Public Estates Authority) was created by President Marcos under P.D. 1084, tasked with
developing and leasing reclaimed lands. These lands were transferred to the care of PEA under P.D.
1085 as part of the Manila Cavite Road and Reclamation Project (MCRRP). CDCP and PEA entered
into an agreement that all future projects under the MCRRP would be funded and owned by PEA.

By 1988, President Aquino issued Special Patent No. 3517 transferring lands to PEA. It was followed
by the transfer of three Titles (7309, 7311 and 7312) by the Register of Deeds of Paranaque to PEA
covering the three reclaimed islands known as the FREEDOM ISLANDS.

Subsquently, PEA entered into a joint venture agreement (JVA) with AMARI, a Thai-Philippine
corporation to develop the Freedom Islands. Along with another 250 hectares, PEA and AMARI
entered the JVA which would later transfer said lands to AMARI. This caused a stir especially when
Sen. Maceda assailed the agreement, claiming that such lands were part of public domain (famously
known as the “mother of all scams”).

Peitioner Frank J. Chavez filed case as a taxpayer praying for mandamus, a writ of preliminary
injunction and a TRO against the sale of reclaimed lands by PEA to AMARI and from implementing
the JVA. Following these events, under President Estrada’s admin, PEA and AMARI entered into an
Amended JVA and Mr. Chaves claim that the contract is null and void.

Issue:
w/n: the transfer to AMARI lands reclaimed or to be reclaimed as part of the stipulations in the
(Amended) JVA between AMARI and PEA violate Sec. 3 Art. XII of the 1987 Constitution
w/n: the court is the proper forum for raising the issue of whether the amended joint venture
agreement is grossly disadvantageous to the government.

Held:
On the issue of Amended JVA as violating the constitution:
1. The 157.84 hectares of reclaimed lands comprising the Freedom Islands, now covered by
certificates of title in the name of PEA, are alienable lands of the public domain. PEA may lease these
lands to private corporations but may not sell or transfer ownership of these lands to private
corporations. PEA may only sell these lands to Philippine citizens, subject to the ownership
limitations in the 1987 Constitution and existing laws.

2. The 592.15 hectares of submerged areas of Manila Bay remain inalienable natural resources of the
public domain until classified as alienable or disposable lands open to disposition and declared no
longer needed for public service. The government can make such classification and declaration only
after PEA has reclaimed these submerged areas. Only then can these lands qualify as agricultural
lands of the public domain, which are the only natural resources the government can alienate. In
their present state, the 592.15 hectares of submerged areas are inalienable and outside the commerce
of man.

3. Since the Amended JVA seeks to transfer to AMARI, a private corporation, ownership of 77.34
hectares110 of the Freedom Islands, such transfer is void for being contrary to Section 3, Article XII
of the 1987 Constitution which prohibits private corporations from acquiring any kind of alienable
land of the public domain.
4. Since the Amended JVA also seeks to transfer to AMARI ownership of 290.156 hectares111 of still
submerged areas of Manila Bay, such transfer is void for being contrary to Section 2, Article XII of
the 1987 Constitution which prohibits the alienation of natural resources other than agricultural
lands of the public domain.

PEA may reclaim these submerged areas. Thereafter, the government can classify the reclaimed
lands as alienable or disposable, and further declare them no longer needed for public service. Still,
the transfer of such reclaimed alienable lands of the public domain to AMARI will be void in view of
Section 3, Article XII of the 1987Constitution which prohibits private corporations from acquiring
any kind of alienable land of the public domain
G.R. No. 201031

TOMAS R. LEONIDAS, Petitioner


vs.
TANCREDO VARGAS and REPUBLIC OF THE PIDLIPPINES, Respondents

DECISION

DEL CASTILLO, J.:

Assailed in this Petition for Review on Certiorari1 are the August 13, 2009 Decision2 and February 22,
2012 Resolution3 of the Court of Appeals (CA) in CA-G.R. CV No. 02296, which affirmed with
modification the March 19, 2007 Decision4 of the Regional Trial Court (RTC) of Barotac Viejo, Iloilo,
Branch 66, in LRC Case No. 02-195.

Factual Antecedents

On February 2, 2002, Tomas R. Leonidas (herein petitioner) filed an application for land
registration5 (Application) covering Lot 566 and Lot 1677 which are both situated in Concepcion,
Iloilo (collectively, subject lots).

Petitioner alleged that he inherited the subject lots from his parents, Ponciano Leonidas, Jr.
(Ponciano) and Asuncion Roxas de Leonidas (Asuncion); that as evidenced by the 1-fay 17, 193 7
Certificate of Sale issued by the Provincial Treasurer of Iloilo, the subject lots, then covered by Tax
Declaration (TD) No. 722, were purchased by Asuncion when auctioned due to delinquency in the
payment of real property taxes by the original owners, the heirs of Inis Luching; that Asuncion
immediately took possession of the subject lots and exercised dominical rights thereover notoriously,
continuously, and exclusively; that upon Asuncion’s death in 1986, Ponciano succeeded to the
ownership and possession of the subject lots; that after Ponciano’s death in l 991, the subject lots
became his (petitioner’s) own exclusive property; that he permitted and tolerated the occupation of
some portions of the subject lots by Juanito Tisolan, Pancing Guevarra, Carmencita Guevarra, Delia
.Aspera-Ecleo, Victorino Mosqueda, Nora Binas, Crisanto Amangas (Amangas),6 Rosana Vasquez,
Henry Asturias, Ronnie Astorias, Antonio Asturias, and Jacob Narciso; that as far as known to him
(petitioner), the following are the owners of all adjoining properties, i.e.the owners of Lot 564, Lot
565 Lot 1578, and Lot 1677, Mansueto Sicad, Francisco Aspero, Brigido Celestial, and Eugenio
Bondoc, Jr. who are all from Poblacion, Concepcion, Iloilo, and Carmen Paoli of unknown address;
that Lot 566 is bounded on the west by the provincial road and the (petitioner) does not claim any
portion thereof; that the latest assessed value of the subject lots is ₱52, 660.00 as certified by the
Provincial Treasurer of Iloilo; that to the best of his knowledge and belief, there is no mortgage or
encumbrance of any kind whatsoever affecting the subject lots except for taxes due thereon; that a
certain Tomas Vargas (Tomas), however, had declared a portion of the subject lots in his name for
taxation purposes; but that Tomas died shortly after the end of the Second World War, and the
whereabouts of his heirs, if any, are unknown, despite his diligent search to locate them in
Concepcion, Iloilo, and elsewhere.

Petitioner also alleged that he was 77 years old, Filipino, a resident of No. 55 Chestnut St., West
Fairview, Quezon City, and married to Ofelia Gustilo Leonidas (Ofelia); that attached to his
Application were original Survey Plans with photographic copies each, the Tracing Cloth Plan land
Management Service, Department of Environment and Natural Resources (DENR), Region VI, Iloilo
City, in lieu of the surveyor’s Certificate, Technical Description with three photographic copies, the
Certificate in quadruplicate of the Provincial Treasurer showing the latest assessed value of the
subject lots, and a copy of the muniment of title to prove ownership of the subject lots, with the
original to be presented at the trial.

Petitioner thus prayed that the subject lots be brought under the operation of the Property
Registration Decree7 (PD 1529) and that the titles thereto be registered and confirmed in his name.

The Republic of the Philippines (Republic), represented by the Office of the Solicitor General (OSG),
opposed the said Application. The Republic claimed that neither the petitioner nor his predecessors-
in-interest had been in continuous, exclusive, and notorious possession and occupation of the
subject lots since June 12, 1945, or prior thereto, as required by Section 48 of Commonwealth Act
(CA) No. 141, as amended by PD 1073; that the petitioner’s muniment/s of title, tax declarations,
and tax payment receipts did not constitute competent and sufficient evidence of either a bona
fide acquisition of the subject lots, and neither did the petitioner's bare claim of open, continuous,
exclusive, and notorious possession and occupation thereof in the concept of owner since June 12,
1945, or prior thereto, amount to convincing proof of his claim of possession and ownership over the
subject lots; that, although the petitioner's muniments of title might appear genuine, the tax
declarations and/or tax payments showing the pretended possession were, in fact, of recent vintage;
that the claim of ownership in fee simple on the basis of a Spanish title or grant could no longer be
availed of by petitioner who had failed to file an appropriate application therefor within the period of
six months from Feb1uary 16, 1976, as required by PD 892; and that the subject lots are portions of
the public domain belonging to the Republic which are not subject to private appropriation. Thus, the
Republic prayed that the petitioner's Application be denied and that the subject lots be declared part
of the public domain.

On March 11, 2003, Tancredo Vargas (Tancredo) also filed an Opposition8 to the Application.
Tancredo averred that he is Tomas' legitimate son and compulsory heir; that during Tomas's lifetime,
the latter was the absolute and exclusive owner of a certain parcel of land located at Loong,
Concepcion, Iloilo, which parcel of land is bounded on the north by the seashore, on the south by
Severino Asturias (Asturias),9 on the east by the seashore, and on the west by Asturias and Braulio
Celestial; that this parcel of land had an area of 36,237 square meters and was covered by TD No.
3549 in Tomas's name; that the petitioner does not exclusively own Lot 1677 since it had been split
into two, viz. Lot 1677-A and Lot 1677-B; that he (Tancredo) is the owner of Lot 1677-A; that Lot 566
was also not exclusively owned by the petitioner, as this Lot 566 had also been divided into two
lots, viz. Lot 566-A and Lot 566-B; that he (Tancredo) is the owner of Lot 566-A as shown in the
RPTA Tax Mapping project in the Municipal of Concepcion, Iloilo; that the petitioner's allegation that
the owners of the property covered by TD 772 became delinquent in the payment of the tax due
thereon, for which reason the Provincial Treasurer of Iloilo allegedly sold the same to Asuncion, was
not at all true; that the property covered by TD 772 was not sold at public auction because the
forfeiture was lifted prior to the public auction sale; and that the fact that the Office of the Provincial
Treasurer of Iloilo did not have a copy of the Certificate of Sale dated May 17, 1937 bolstered the
argument that petitioner’s allegation is questionable. Tancredo thus prayed that the petitioner's
Application be denied insofar as the portions covered by the TDs in the name of Tomas (disputed
portions) are concerned.

On March 21, 2003, another Opposition10 to the Application was filed by Moncerat A. Sicad-De
Julian, Gil A. Sicad, represented by his wife, Elizabeth Sicad, Teresita A. Sicad-Bayuran, Villaluz
Sicad-Zarriz, Eden A. Sicad, and Melchor Sicad, represented by his wife, Elena D. Sicad, (Elena;
collectively, the Sicads) all represented by their attorney-in-fact, Elena.11 These oppositors claimed
that they are the heirs of the late Mansueto Sicad (Mansueto) who was the owner of a portion of the
subject lots (Sicads’s contested portion); that the Sicads's contested portion was bought by
Mansueto from Asturias as evidenced by the Deed of Definite Sale of a Parcel of Land described as
Doc. No. 75, Page No. 35, Book No. 1, Series of 1950 of the notarial register of notary public Crespo
Celestial; that the Sicads's contested portion had been in the possession of Mansueto during the
latter's lifetime; that they had been in possession of the Sicads’s contested portion since Mansueto's
death; that part of the Sicads’s contested portion had already been registered under Original
Certificate of Title (OCT) No. F-36795; and that the petitioner had never been in possession of the
lots subject of his Application. The Sicads thus prayed that the petitioner's Application be dismissed,
insofar as it concerned the Sicads’s contested portion as set forth in the aforesaid Deed of Definite
Sale; and that the Sicads’s contested portion be registered instead in their names.

At the trial, the petitioner presented himself and Geronimo C. Pefiaflorida (Peñaflorida), Land
Management Inspector, DENR, Community Environment and Natural Resources Office (CENRO), at
Sara, lloilo as witnesses.12On the other hand, Catalino Guinez, Emeliana Isturias Matulac, and Elena
testified for the Sicads.13 For his part, Tancredo presented himself and a forn1er overseer or tenant
of the Vargas family,14 Jose Etchona (Etchona).15 Then on August 8, 2003, the petitioner filed his
Formal Offer

of Evidence16 wherein he submitted the Certificate of Sale dated May 17, 1937, TD 014134 for the
year 1976 in Asuncion's name and covering Cadastral Lot Nos. 1, 2, and 3 PSU-216090, TD 0037
for the year 1994 in the names of Asuncion and Ponciano and covering Cadastral Lot No. 1677, TD
0036 for the year 1994 in the names of Asuncion and Ponciano and covering Cadastral Lot No. 566,
TD 0114_ for the year 2003 in the names of Asuncion and Ponciano and covering CadastraJ Lot No.
1677-A, TD 0118 for the year 2003 in the names of Asuncion and Ponciano and covering Cadastral
Lot No. 1677-B, TD 0116 for the year 2003 in the names of Asuncion and Ponciano and covering
Cadastral Lot No. 566-A; and TD 0117 for the year 2003 in the names of Asuncion and Ponciano
and covering Cadastral Lot No. 566-B,17 tax receipts for the years 1986, 1987, 1988, 1989, 1990,
1991, 1994, 2002 and 2003, statement of the assessed value issued by the Provincial Assessor of
Iloilo on March 26, 1996, Lot No. 566's Blue Print Survey Plan with technical description, Lot l 677’s
Blue Print Survey Plan with technical description, Certificate of Unavailability of Surveyor's
Certificate of Survey for Lots 566 and 1677, and Survey Inspection Report dated August 28, 1997 for
Lot Nos. 566 and 1677 issued by Peñaflorida,18 i.e. CENRO Report dated August 28, 1997, to the
effect that the subject lots are free from liens and encumbrances, and are moreover within the
alienable and disposable area. Pursuant to the RTC's directive, petitioner also offered as additional
evidence the originally-approved subdivision plan covering Lot No. 1677, Csd-06-008798 to prove
the identity and location of the easement for public use;19 and a certification by Joel B. Diaz, CENRO
at Sara, Iloilo, to the effect that Lot No. 1677, Pls 1099, situated in Brgy; Loong, Concepcion, Iloilo,
with an area of 8,062 square meters was issued Patent No. 063015-92-846 dated May 28, 1992 in
the name ofFlordeluz Sedigo, but that Lot No. 1677 has doubled with the lot situated at Poblacion,
Concepcion, Iloilo in the name of the Heirs of Ponciano and that this latter lot is not covered by any
public land application filed with the CENRO in Sara, Iloilo, which explained why no patent has been
issued therefor, hence indicating that this other Lot No. 1677, Pls 1099, which is situated in Brgy.
Aglusong, Concepcion, Iloilo is entirely different from Lot No. 1677, which is situated in Sitio Loong,
Poblacion, Concepcion, Iloilo.20

The petitioner likewise submitted in evidence an Ocular Inspection Report covering an ocular
inspection earlier ordered by the RTC.21

Ruling of the Regional Trial Court

In its Decision dated March 19, 2007, the RTC disposed of this case in this wise:

WHEREFORE, general default having been declared and the [A]pplication supported by evidence,
the adjudication and registration of portion of Lot No. 566 with an area of 3.1161 hectares and
portion of Lot 1677 with an area of 3. 7255 hectares, all of Concepcion Cadastre, together with all
the improvements thereon are hereby ordered in favor of applicant [petitioner], of legal age, married
to [Ofelia], Filipino, and resident of Fairview, Quezon City, Philippines. Portions of Lot [No.] 1677
with an area of 2.3642 hectares and portion of Lot [No.] 566 with an area of 1.1782 hectares are
hereby adjudicated in favor of [Tancredo ], of legal age, single, Filipino, and resident of Lawa-an
Village, Balantang, Jaro, Iloilo City, Philippines which portions shall be segregated in a proper
subdivision survey and to follow the description of the plan of Municipal Assessor of Concepcion,
Iloilo commensurate to Lot 1677-A under [T.D.] No. 054822 and 566-A under [T.D.] No. 0550.

The easement of right of way of the lots, highways, streets, alleys, shorelines and other portion[s] of
land not specified as lots located within the borders of the land covered by this case are declared to
be the properties of the [Republic].

The Clerk of Court is directed to forward copies of this decision to all government agencies
concerned.

And finally, the Administrator, Land Registration Authority, is hereby directed, after this decision shall
have become final for which he shall be duly advised by specific order of this Court, to issue [a]
decree of registration and title in accordance with the amended plan on file in the record.

SO ORDERED.23

The RTC held that petitioner had sufficiently established that his predecessors-in-interest had
possessed and owned a parcel of land in Barangay Loong, Concepcion, Iloilo to the extent not
covered by Tancredo’s Opposition; that while petitioner and his predecessors-in-interest might not
have been in actual possession of the subject lots at all time, they nonetheless had been
consistently visiting the same; and that petitioner’s claim of possession and ownership is supported
by documents consisting of the Certificate of Sale issued by the Provincial Treasurer of Iloilo on May
17, 1937, the tax declarations in Asuncion's name for the years 1976, 1994, and 2003, the official
receipts showing payments of real estate taxes thereon, and the statement of the assessed value
issued by the Provincial Assessor of Iloilo on May 26, 1996. The RTC stressed that the period of
possession by petitioner and his predecessors-in-interest sufficed to confer a registrable title upon
petitioner.

The RTC likewise ruled that Tancredo was also able to establish a superior claim with respect to his
disputed portions; that all of the tax declarations in Asuncion's name continuously bore the
annotation acknowledging Tomas's adverse claim relative to Tancredo's disputed portions; that
Tomas's open and continuous possession for more than the required number of years was
sufficiently shown by a tax declaration issued as early as the year 1945; that the overseers and other
persons authorized to manage Tancredo's disputed portions were never driven out by petitioner; and
that Tancredo had visited the disputed portions more frequently than petitioner who, as the evidence
shows, has his permanent residence in Quezon City, Metro Manila.

With regard to the claim of the Sicads, the RTC held that Mansueto and his successors-in-interest
had no more interest in the Sicads' contested portion because what was shown to have been sold by
Asturias to Mansueto pertained to a lot measuring only two hectares, 52 acres, and 92 ares, a parcel
of land at par with the land covered by the aforementioned free patent issued to Mansueto.

The RTC emphasized that it is well-entrenched in jurisprudence that alienable public land openly,
continuously, and exclusively possessed by a person personally or through his predecessors-in-
interest for at least 30 years becomes ipsojure private property by mere lapse of time, or by
completion of said period pursuant to Section 48(b) of CA 141, as amended by RA 1942 and RA
3872.

Ruling of the Court of Appeals

Only the petitioner and the Republic filed their respective Notices of Appeal24 which were given due
course by the RTC in its Order of May 25, 2007.25 These notices of appeal were consolidated and
docketed as CA-G.R. CV No. 02296. In a Decision dated August 13, 2009, the CA disposed as
follows:

WHEREFORE, the Decision dated March 19, 2007 is modified, as follows: 1.) the portion pertaining
to the award of [Lot No.] 566 with an area of3.1161 hectares and [Lot No.] 1677 with an area of
3.7255 hectares to [petitioner], is REVERSED and SET ASIDE; and 2.) the portion pertaining to the
award of [Lot No.] 1677 with an area of 2.3642 hectares and [Lot No.] 566 with an area of 1.1782
hectares in favor of [Tancredo] is AFFIRMED.

SO ORDERED.26

The CA held that, contrary to the Republic's stance, the records showed that there had been
compliance with the jurisdictional requirements of publication, posting, and notice; that petitioner had
properly identified the subject lots; that the subject lots had already been classified as alienable and
disposable at the time that petitioner filed the Application in 2002, pursuant to the CENRO Report
dated August 28, 1997 issued by Peñaflorida; that it has been held that "[a] certification by the
CENRO of the DENR stating that the subject lots are found to be within the alienable and disposable
site per land classification project map is sufficient evidence to show the real character of the land
subject of the application;"27 that these notwithstanding, petitioner failed to prove with the requisite
evidence the kind of possession and the length of time required by law for the registration of the
subject lots in his name, because his lone testimony did not suffice to establish his and his
predecessors-in-interest’s alleged open, continuous, exclusive, and notorious possession over the
subject lots since June 12, 1945, or earlier; that petitioner's alleged acts of swimming in, and planting
trees on the subject lots, his having finished high school at the Victorino Salcedo High School in the
neighboring town of Sara, Iloilo, and his having left the subject lots when he attended college - all
these neither added up nor supported his assertion of dominion or ownership· over the subject lots;
that his allegation that his childhood memories regarding the subject lots all came back to him after
the death of his father Ponciano was indicative of the fact that he was really unaware of the
existence of the subject lots; that his Application was even opposed by Tancredo and by the Sicads
who claimed exclusive possession over certain portions of the subject lots; that petitioner's failure to
explain why he or his predecessors-in-interest declared the subject lots for taxation purposes only in
1976, was inconsistent with his claim of possession thereover since 1937; and that it is an axiom of
the law that the burden of proof in a land registration case rests upon the applicant who must
present clear, positive, and convincing evidence establishing the alleged possession and occupation
in good faith, and for the period required by law.

On the other hand, the CA ruled that Tancredo had sufficiently proven his open, continuous,
exclusive, and notorious possession and occupation for the period required by law, over the portions
of the subject lots he was claiming in the concept of an owner; that Tomas's adverse claims were
annotated on the TDs issued in Asuncion’s name covering the disputed portions, i.e. TD 014134,
0114, and 0117 ;28 that Tomas declared the disputed portions for taxation purposes in his name as
early as 1945; that Tancredo himself testified that Tomas first used the disputed portions as rice land
and converted the same into coconut land in the 1960s; that Tancredo’s witness, Etchona, likewise
testified that Tomas employed him and Domingo Celestial not only to cultivate, but also to guard the
disputed portions, and that Tomas himself appropriated the harvest from the disputed portions and
introduced improvements thereon; and that even petitioner himself admitted in his Application that
Tomas had declared the disputed portions in his (Tomas’) name for taxation purposes.

Petitioner moved for reconsideration29 but was denied by the CA in its Resolution of February 22,
2012.30

Issue

Before this Court, petitioner now raises the following issue:

[Whether] the [CA] gravely abused its discretion in denying the registration of [his] already vested
title [over] Lot [Nos.] 566 and 1677 of the Concepcion, Iloilo Cadastre as his private property, and in
awarding some portions thereof in favor of [Tancredo] in this land registration proceeding.31

Petitioner’s arguments

Petitioner insists in his Petition,32 Consolidated Reply,33 and Memorandum34 that the CA erred in
finding that he failed to prove that he and his predecessors-in-interest had been in open, continuous,
exclusive, and notorious possession and occupation of the subject lots since June 12, 1945, or
earlier, and that there is indubitable evidence that the subject lots were in fact sold in a tax sale on
May 1 7, 1937 by the gove1nment through the Provincial Treasurer of Iloilo; that he filed the present
Application so that an OCT can be issued in his name as evidence of his vested title over the subject
lots; that assuming that the subject lots are still part of the public domain, he is nevertheless still
entitled to have the subject lots registered in his name by reason of his and his predecessors-in-
interest's exclusive possession and occupation thereof for more than 30 years, as compared to
Tancredo's possession which supposedly began only in 1945; that under the Land Registration Act,
as amended, the possessor is deemed to have acquired by operation of law the right to a
government grant upon compliance with the conditions therefor, which was just what he did in this
case; that the confirmation proceeding is a mere formality and the registration thereunder does not
confer title but merely recognizes a title that is already vested; that rejection of his vested title to the
questioned lots will occasion loss of confidence in the government's sales of forfeited property by
reason of tax delinquency; that the CA erred in finding that the TDs in Asuncion's name carried
Tomas's adverse claim, as the attached copies thereof did not bear any such annotations; that the
CA also erred in stating that petitioner did not present any TDs to support his claim of ownership
over the subject lots for the reason that the CA Decision itself mentioned that he submitted a TD for
the year 1976; that contrary to the CA's findings, he did testify that he had visited the subject lots
every so often to plant trees after he and his parents left Concepcion in 1945, and that such
improvements were reflected in his exhibits; that the CA likewise erred in holding that he only came
to know about the subject lots after the death of his father, Ponciano, for the fact is that he did testify
that he and his cousins used to swim in the sea near the subject lots, as early as when he was 12
years old; that the CA moreover erred in concluding that Tancredo had successfully established his
claims over the disputed portions of the subject lots because the TDs in Asuncion's name are all
annotated with Tomas’s adverse claim, and that Tomas had declared said disputed portions in his
name as early as 1945; that the tax declarations supposedly in Tomas's name were neither
presented nor offered in evidence; that Tancredo admitted during his cross-examination that
Tomas’s 1945 tax declaration was procured notwithstanding the fact that the subject lots had already
been declared in Asuncion’s name; that Tancredo did not comply with the pertinent provisions of the
Land Registration Act, as amended, because he did not present evidence to prove the specific date
in 1945 when Tomas acquired the disputed portions, or how Tomas in fact acquired the same; that
besides these, Tancredo could not identify the disputed portions that he was claiming; that if
Tancredo wanted to vindicate his claims of ownership over the disputed portions, then Tancredo
should institute the proper action before a court of general jurisdiction, and not in the land
registration court, as the subject lots were no longer part of the public domain; that the issue of
whether the sale by the government to Asuncion on May 17, 1937 changed the classification of the
subject lots from public to private is of first impression and should be resolved by the Supreme
Court EnBanc; and that the circumstances obtaining in this case are exceptions to the rule that only
questions of law are allowed in a petition filed pursuant to Rule 45 of the Revised Rules of Court;
and that to deny his Application, or to render judgment ordering the reversion to public ownership of
the subject lots would amount to grave abuse on the part of the judiciary.

The Republic’s Arguments

In its Comment35 and Memorandum,36 the Republic counters that the instant Petition merely raises
questions of fact which are proscribed under Rule 45 of the Revised Rules of Court; that this Court is
not a trier of facts; that petitioner's case does not fall under any of the exceptions to the rule that
factual findings of the CA are invariably binding upon the Supreme Court; and that the assailed CA
Decision should not be disturbed because the CA had amply justified the reversal of the RTC
Decision which was erected upon the petitioner's failure to substantiate his claim of ownership over
the subject lots.

Tancredo’s Arguments

In his Comment37 and Memorandum,38 Tancredo maintains that the disputed portions had been in the
absolute possession and dominion of Tomas; that the findings of the RTC and the CA regarding
petitioner's ineligibility to obtain title to the disputed portions due to non-compliance with the
requirements of the law, and for insufficiency of evidence, should not be disturbed; that the CA’s
finding that petitioner's TDs bore the annotated claims of Tomas on the subject lots is a factual
finding and should not be disturbed; that petitioner's possession is not the possession required by
law for purposes of land registration because petitioner failed to present evidence that would prove
actual, notorious, continuous, and exclusive possession and occupation of the subject lots; that the
evidence adduced by petitioner is self-serving, hence undeserving of any weight; that the origin of
the disputed portions as pointed out by the RTC is Assessor's Lot No. 337, which is individually
identified after the Cadastral Survey as Lot Nos. 1676- A, 1677-A, and 566-A, all of the Concepcion
(Iloilo) Cadastre; that petitioner is barred or estopped from questioning the identity of the disputed
portions that had been adjudicated to him (Tancredo ), as the lack of sufficient identification
pertained to the subject lots that petitioner himself was trying to register; and that the issues raised
by petitioner were factual in nature, and the same is proscribed under Rule 45 of the Revised Rules
of Court.

The fundamental issues to be resolved in this case are: (1) Whether the petitioner is entitled to
obtain a title over the subject lots; and (2) Wether Tancredo has established, by his own evidence,
that he was qualified to acquire title over the disputed portions claimed by him.

The Court’s Ruling

The Petition is denied.

Requisites for the confirmation and


registration of an imperfect and
incomplete title under CA 141 and
PD 1529

"The Regalian doctrine, embodied in Section 2, Article XII of the 1987 Constitution, provides that all
lands of the public domain belong to the State, which is the source of any asserted right to
ownership of land."39 "[Commonwealth Act No. 141, in turn,] goven1s the classification and
disposition of lands of the public domain. Section 11 [thereof] provides, as one of the modes of
disposing public lands that are suitable for agriculture, the 'confirmation of imperfect or incomplete
titles.’ Section 48 [thereof], on the other hand, enumerates those who are considered to have
acquired an imperfect or incomplete title over public lands and, therefore, entitled to confirmation
and registration under the Land Registration Act [now PD 1529]."40 The latter law then "specifies who
are qualified to apply for registration of land."41 Taken together, all the foregoing provide for the
requisites for the confirmation and registration of an imperfect and incomplete title, thus -

x x x In particular, Section 14 (l) [of PD 1529] in relation to Section 48 (b) of[CA] 141, as amended by
Section4 of P.D. No. 1073, states:

SEC. 14. Who may apply. -The following persons may file in the proper Court of First Instance [now
Regional Trial Court] an application for registration of title to land, whether personally or through their
duly authorized representatives:

(1) Those who by themselves or through their predecessors-in-interest have been in open,
continuous, exclusive and notorious possession and occupation of alienable and disposable lands of
the public domain under a bonafide claim of ownership since June 12, 1945, or earlier.

xxx xxx xxx

Section 48. The following described citizens of the Philippines, occupying lands of the public domain
or claiming to own any such lands or an interest therein, but whose titles have not been perfected or
completed, may apply to the Court of First Instance [now Regional Trial Court] of the province where
the land is located for confirmation of their claims and the issuance of a certificate of title therefor,
under [PD 1529], to wit:

xxx xxx xxx

(b) Those who by themselves or through their predecessors-in-interest have been in open,
continuous, exclusive and notorious possession and occupation of [alienable and disposable lands]
of the public domain, under a bonafide claim of acquisition of ownership, since June 12, 1945, or
earlier, immediately preceding the filing of the application for confirmation of title except when
prevented by war or force majeure. These shall be conclusively presumed to have performed all the
conditions essential to a Government grant and shall be entitled to a certificate of title under the
provisions of this chapter.

Based on these legal parameters, applicants for registration of title under Section 14 (1) must
sufficiently establish: (1) that the subject land forms part of the disposable and alienable lands of the
public domain; (2) that the applicant and his predecessors-in-interest have been in open, continuous,
exclusive and notorious possession and occupation of the same; and (3) that his possession has
been under a bonafide claim of ownership since June 12, 1945, or earlier.

These triple requirements of alienability and possession and occupation since June 12, 1945 or
earlier under Section 14 (1) are indispensable prerequisites to a favorable registration of title to the
property. Each element must necessarily be proven by no less than clear, positive and convincing
evidence; otherwise, the application for registration should be denied.42

Petitioner did not cite the specific provision of CA 141 upon which he based his Application.
Neve1theless, the allegations therein seem to establish the fact that his claim is one of imperfect title
under the above- quoted Section 48(b) of CA 141 in relation to Section 14(1) of PD 1529
The subject lots are considered
alienable and disposable lands of the
public domain

The first· requirement is complied with in the case at bench. Notwithstanding that only a CENRO
certification covering the subject lots was presented in the instant case, the subject lots are
considered alienable and disposable lands of the public domain because of this Court’s ruling that
an application for land registration may be granted despite the absence of the DENR Secretary's
certification, provided that the same was pending at the time Republic v. Vega43 was promulgated on
January 17, 2011. In Republic v. Alora,44 this Court expressly clarified this matter in this wise:

x x x [I]n Republic v. T.A.N Properties, Inc., which was promulgated on 26 June 2008 x x x we held
that applicants for land registration must present a copy of the original classification approved by the
DENR Secretary and certified as true copy by the legal custodian of the official records. x x x

x x x In Republic v. Serrano [(decided on 24 February 2010)], we allowed the approval of a land


registration application even without the submission of the certification from the DENR Secretary. As
this ruling presented an apparent contradiction with our earlier pronouncement in Republic v. T.A.N
Properties, Inc., we sought to harmonize our previous rulings in Republic v. vega [(decided on 17
January 201 l)]. We then said that the applications for land registration may be granted even without
the DENR Secretary's certification provided that the application was currently pending at the
time Republic v. vega was promulgated. x x x45

It is worth stressing, however, that the foregoing ruling is the exception, not the rule. As explicitly
elucidated in Republic v. Vega:46

It must be emphasized that the present ruling on substantial compliance applies pro hac vice. It does
not in any way detract from our rulings in Republic v T.A.N Properties, Inc., and similar cases which
impose a strict requirement to prove that the public land is alienable and disposable, especially in
this case when the Decisions of the lower court and the [CA] were rendered prior to these rulings. To
establish that the land subject of the application is alienable and disposable public land, the general
rule remains: all applications for original registration under [PD 1529] must include both (1) a
CENRO or PENRO certification and (2) a certified true of the original classification made by the
DENR Secretary.

As an exception, however, the courts - in their sound discretion and based solely on the evidence
presented on record - may approve the application, pro hac vice, on the ground of substantial
compliance showing that there has been a positive act of government to show the nature and
character of the land and an absence of effective opposition from the government. This exception
shall only apply to applications for registration currently pending before the trial court prior to this
Decision and shall be inapplicable to all future applications. (Underscoring and emphases in the
original)47

That said, we hold that both the petitioner and Tancredo failed to establish clearly and convincingly
their respective rights to registration of imperfect titles under CA 141 and PD 1529, as will be
discussed below.

Petitioner failed to prove possession


of the subject lots in the manner and
for the period required by law
First off, petitioner failed to establish bonafide possession and ownership over the subject lots since
June 12, 1945 or earlier. His contention that his predecessors-in-interest became the owners of the
subject lots pursuant to the May 17, 1937 Certificate of Sale48 of the Forfeited Real Property issued
by the Provincial Treasurer of Iloilo appears to be consistent with the fact that TD 3549 in Tomas’s
name which was found by the CA as issued in 1945 bears an annotation stating that such is "[ c
]ontested by [Asuncion]".49 Even then, the Certificate of Public Sale indicated that the balance of
the purchase price in the amount of ₱29.44, was yet to be paid on or before December 31,
1937.50

No incontrovertible proof was, however, presented to establish the fact that this balance of the
purchase price in the said amount of ₱29 .44 had indeed been paid on or before December 31, 193
7. In addition, the CA also correctly pointed out that even as petitioner was able to submit TDs and
evidence of tax payments only for a few years, he nevertheless failed to explain why he or his
predecessors-in-interest declared the subject lots for taxation purposes only in 1976, this despite his
claim that his predecessors-in-interest had been in possession and occupation of the subject lots
since 1937, as allegedly shown in the Provincial Treasurer’s Certificate of Sale. It is settled that
intermittent and irregular tax payments run counter to a claim of ownership or possession.51

Second, even assuming for argument's sake that petitioner’s predecessors-in-interest had paid the
balance of the delinquent tax payment, petitioner nonetheless failed to prove his and his
predecessors-in-interests actual, notorious, exclusive and continuous possession of the subject lots
for the length of time required by law.

To be sure, petitioner's failure to explain what happened after his family supposedly left the subject
lots in 1941, when the war broke out, vis-a-vis his failure to prove that he had indeed introduced
valuable improvements in the subject lots during the time that he and his parents had been allegedly
in actual possession and occupation thereof, cast doubts upon his claim of actual possession and
occupation thereof. Withal, petitioner's testimony of having swum near the subject lots, of having
planted trees thereon, and his having finished high school at the Victorino Salcedo High School in
the neighboring town of Sara can hardly be considered as acts of dominion or ownership over the
subject lots. Besides, petitioner did not present clear and convincing evidence that the subject lots
had indeed been cultivated by him or by his predecessors-in-interest for the period of time required
by law. Needless to say, all these failings weaken his claim that he has been a bonafide possessor
and occupant of the subject lots in the manner and for the period prescribed by law, to wit:

The possession contemplated by Section 48 (b) of [CA] 141 is actual, not fictional or constructive.
In Carlos v. Republic of the Philippines, the Court explained the character of the required
possession, as follows:

The law speaks of possession and occupation. Since these words are separated by the conjunction
and, the clear intention of the law is not to make one synonymous with the other. Possession is
broader than occupation because it includes constructive possession. When, therefore, the law adds
the word occupation, it seeks to delimit the all-encompassing effect of constructive
possession, Taken together with the words open, continuous, exclusive and notorious, the
word occupation serves to highlight the fact ti1at for an applicant to qualify, his possession
must not be a mere fiction. Actual possession of a land consists in the manifestation of acts
of dominion over it of such a nature as a party would naturally exercise over his own
property.52 (Emphases in the original)

Oddly enough, while in its Decision the RTC appeared to have granted petitioner’s Application, said
Decision seemed to have indulged in a bit of non-sequitur when it said that "[petitioner] and his
predecessors were not in actual possession of the [subject Jots] all the time" x x x.53 Simply said, the
-A effectively ruled that since petitioner failed to prove that he or his predecessors-in-interest had
indeed performed the required acts of possession and occupation, or specific acts of dominion over
the subject lots, it stands to reason that registration thereof in his name cannot be allowed.

Tancredo also failed to establish


possession and occupation over the
disputed portions in the manner and
for the period required by law

At this juncture, we shall revisit the uniform finding by both the RTC and the CA, which in effect
upheld Tancredo's right to register the disputed portions in his name (as an exception to the settled
rule that questions of fact are proscribed in a Rule 45 petition since a correct evaluation of the facts
will yield a different conclusion).54

First off, Tancredo failed to show that his or his predecessor-ininterest’s possession and occupation
over the disputed portions had been under a bonafide claim of ownership since June 12, 1945, or
earlier. We are inclined to agree with petitioner's posture that Tancredo failed to adduce clear and
convincing evidence which established the origin or antecedents of Tomas's straightforward
possession and occupation, or claim of ownership, over the disputed portions. Consider the following
exchange/s between/among Tancredo, the petitioner, and the Court -

[Petitioner]: (to the witness[, Tancredo])

Q: When did your father acquire this property?

A: In 1945.

Q: From whom?

A: I have no idea.

xxxx

Q: Did you not ask your father from whom he acquired this property?

A: No, I did not.

Q: As a matter of fact[,] until the death of your father[,] you have not ask[ed] him from whom did he
acquire the property?

A: No, Sir.

xxxx

COURT: (to the witness[, Tancredo])

Q: Your father died in 1995 [,] why did you not [cause] the transfer of tax declaration in your name or
to the heirs?
A: Because the plan of the heirs is, if the property [is registered] in my father[']s name [then] the title
should be transferred in my name.

xxxx

Q: Your tax receipts correspond only [to] the year 2003, how about other tax receipts?

A: I [will just [try] to find out if the Provincial Treasurer’s Office still has the copy.

Q: Even just a certification stating that you [continued] in paying realty tax from 1946 up to 2003?

A: Yes, I can ask the provincial treasurer for that matter.

Q: When you secure[d] the tax declaration[,] you [knew] that the lot was also declared in the name of
[Asuncion], is it not?

A: Yes, Your Honor.

Q: That was in the office of the Municipal Assessor?

A: Yes, Your Honor.

Q: Did you verify if they were paying taxes also?

A: No, Your Honor.

Q: You did not?

A: I [did] not[,] Your Honor.

Q: If that is the case[,] why did you [say] a while ago that you [knew]. only [about] the case of
[petitioner] when this case was filed because the tax declaration itself [stated] that the lot was also
declared in the name of [Asuncion]?

A: Although I have already seen the notation on the tax declaration that they also [secured a] tax
declaration [over] the [disputed Portions]. I did not mind it Your Honor because they did not openly
claim ownership over the [disputed portions]. And in the same manner[.] Your Honor[,] in their tax
declaration it is also indicated that the [disputed portions] is also declare[d] in the name of [Tomas].55

More than this, Tancredo did not present clear, convincing evidence to support his claim that the
disputed portions were in fact transferred to him by his father, Tomas. Tancredo merely testified that
the disputed portions were given to him solely by Tomas, an act that was allegedly consented to by
his siblings. Thus-

[Petitioner]: (to the witness, Tancredo)

Q: You have siblings, meaning brothers and sisters?

A: Yes, Sir.
Q: You said a while ago that you succeeded to the ownership of the [subject lots] when your father
died in 1985, how about your siblings[?] [Did they] not succeed to the [ownership of the subject lots?]

A: They sign[ed] a deed of adjudication in favor of me[.] I have a copy and it was notarized.

xxxx

Q: In your [O]pposition you said that you were authorized?

A: Yes, Sir.

Q: By whom?

A: By my brothers and sisters.

Q: Where is your authority?

A: I can produce it. I can pass [sic] it anytime.

Q: You did not [s]tate in your [O]pposition that you have your siblings with you?

A: Because the property was given to me by my father. 56

Nonetheless, there is nothing in the records to support or confirm Tancredo’s claim that the property
was in fact deeded over to him by his father, Tomas.

In Buenaventura v. Pascual,57 this Court affirmed the lower courts' dismissal of the claims for
registration of imperfect titles because, among others, both the applicant and oppositors failed to
adduce evidence as to how they acquired the subject property from their respective predecessors-in-
interest, i.e., whether by succession or by donation or by some other mode. Furthermore, we
stressed therein that the applicant failed to prove the manner by which her predecessors-in-interest
possessed the subject property.

Then, again, Tancredo also failed to establish that he and his predecessors-in-interest had/have
been in open, continuous, exclusive and notorious possession and occupation of the disputed
portions since June 12, 1945, or prior thereto.

If anything, the records showed that Tancredo merely submitted photocopies of four tax declarations
which were attached as annexes to his Opposition. These included the 1945 TD 3549 as adverted
1âw phi 1

to by the CA in the records58pertaining to a 3.6237-hectare lot in an unstated cadastral lot, TD 0548


covering an 813-hectare lot in Cadastral Lot No. 1676-A,59 TD 0549 for a 2.3642-hectare lot in
Cadastral Lot No. 1677-A,60 and TD 0550 concen1ing a 1.1782-hectare lot in Cadastral Lot No. 566-
A.61 All four TDs are in Tomas's name, without copies of the dorsal portions thereof, and bearing
annotations stating either "[c]ontested by [Asuncion]" or "[a]lso declared in the name of [Asuncion] or
[Ponciano]".

It would thus appear that Tancredo had erected his opposition/claim to the lots in question upon the
said photocopies of four tax declarations whose authenticity or genuineness is open to the most
serious doubts. And, even on the assumption that the said tax declarations are in fact authentic and
genuine, still it is settled that tax declarations are not conclusive proof of ownership. If anything, tax
declarations are merely corroborative of a person's claim of possession. More than that, as
elsewhere indicated, intermittent and irregular tax payments, as in this case, do not really provide
strong support for a claim of ownership or possession.62

It is axiomatic of course that "[i]t is the policy of the State to encourage and promote the distribution
of alienable public lands as a spur to economic growth and in line with the social justice ideal
enshrined in the Constitution. At the same time, the law imposes stringent safeguards upon the
grant of such resources lest they fall into the wrong hands to the prejudice of the national
patrimony."63 This ruling controls the present case.

As a final note: All of the foregoing discussion showed that the issues raised in this case have all
been previously resolved and determined by settled jurisprudence; hence, there is no reason to
grant petitioner's prayer for this case to be referred to or heard by the Court EnBanc, as this is not a
case of first impression at all.

WHEREFORE, the Petition is hereby DENIED. We AFFIRMwithMODIFICATION the August 13,


2009 Decision and the February 22, 2012 Resolution of the Court of Appeals in CA-G.R. CV No.
02296 in that the award by the Regional Trial Court of Barotac Viejo, Iloilo, Branch 66 in LRC Case
No. 02~ 195 of Lot No. 1677 with an area of 2.3642 hectares and Lot No. 566 with an area of 1.1
782 hectares, both in favor of respondent Tancredo Vargas, is OVERTURNED and NULLIFIED

SO ORDERED.
Cruz vs Secretary of DENR
Natural Resources and Environmental Law; Constitutional Law; IPRA; Regalian
Doctrine

GR. No. 135385, Dec. 6, 2000

FACTS:

Petitioners Isagani Cruz and Cesar Europa filed a suit for prohibition and mandamus as
citizens and taxpayers, assailing the constitutionality of certain provisions of Republic Act
No. 8371, otherwise known as the Indigenous People’s Rights Act of 1997 (IPRA) and its
implementing rules and regulations (IRR). The petitioners assail certain provisions of the
IPRA and its IRR on the ground that these amount to an unlawful deprivation of the State’s
ownership over lands of the public domain as well as minerals and other natural resources
therein, in violation of the regalian doctrine embodied in section 2, Article XII of the
Constitution.

ISSUE:

Do the provisions of IPRA contravene the Constitution?

HELD:

No, the provisions of IPRA do not contravene the Constitution. Examining the IPRA, there
is nothing in the law that grants to the ICCs/IPs ownership over the natural resources
within their ancestral domain. Ownership over the natural resources in the ancestral
domains remains with the State and the rights granted by the IPRA to the ICCs/IPs over the
natural resources in their ancestral domains merely gives them, as owners and occupants of
the land on which the resources are found, the right to the small scale utilization of these
resources, and at the same time, a priority in their large scale development and exploitation.

Additionally, ancestral lands and ancestral domains are not part of the lands of the public
domain. They are private lands and belong to the ICCs/IPs by native title, which is a concept
of private land title that existed irrespective of any royal grant from the State. However, the
right of ownership and possession by the ICCs/IPs of their ancestral domains is a limited
form of ownership and does not include the right to alienate the same.
Land Titles And Deeds Case Digest: Oh Cho
V. Director Of Lands (1946)
G.R. No. L-48321. August 31, 1946

Lessons Applicable: (Land Titles and Deeds)


Sec. 2 Art. XII, 1987 Constitution

FACTS:

 Oh Cho is appealing from the rejection of his application based on disqualification as alien (Chinese) from
acquiring lands of the public domain.
 He had open, continuous, exclusive and notorious possession of the lot from 1880 to filing of the
application for registration on January 17, 1940
ISSUE: W/N Oh Cho entitled to decree or registration of the lot.

HELD: NO.

 GR: All lands that were not acquired from the Government, either by purchase or by grant below to the
public domain
 Exception: in the possession of an occupant and of his predecessors in interest since time immemorial,
for such possession would justify the presumption that the land had never been part of the public
domain or that it had been a private property even before the Spanish conquest.
(Cariño v. Insular Government) - not applicable since only from 1880
 His immediate possesor failed to comply with the condition precedent to apply for the registration of the
land of which they had been in possession at least since July 26, 1894 so what was transferred to Oh Cho
is merely possesory right which cannot ripen to ownership by prescription (aliens disqualified to own by
prescription)
G.R. No. 48321, August 31, 1946

o GR: All lands are acquired from the Government, either by purchase or by grant.
o EXCEPTION: Lands under private ownership since time immemorial.
o Application for decree of registration is a condition precedent to acquisition of title. Non-
compliance gives rise to mere possessory right.
o An alien cannot acquire title to lands of the public domain by prescription.

FACTS:

Oh Cho, a Chinese citizen, purchased from the Lagdameos a parcel of land in Tayabas,
which they openly, continuously and adversely possessed since 1880. On January 17,
1940, Oh Cho applied for registration of this land. The Solicitor General opposed on the
ground that Oh Cho lacked title to said land and also because he was an alien.

ISSUEs:

o Whether or not Oh Cho had title


o Whether or not Oh Cho is entitled to a decree of registration

HELD:

Oh Cho failed to show that he has title to the lot, which may be confirmed under the
Land Registration Act.

All lands that were not acquired from the Government, either by purchase or by grant,
belong to the public domain. An exception to the rule would be any land that should
have been in the possession of an occupant and of his predecessors in interest since
time immemorial, for such possession would justify the presumption that the land had
never been part of the public domain or that it had been a private property even before
the Spanish conquest.

The applicant does not come under the exception, for the earliest possession of the lot
by his first predecessor in interest began in 1880.

Under the Public Land Act, Oh Cho is not entitled to a decree of registration of the lot,
because he is an alien disqualified from acquiring lands of the public domain.

Oh Cho's predecessors in interest would have been entitled toa decree of registration
had they applied for the same. The application for the registration of the land was a
condition precedent, which was not complied with by the Lagmeos. Hence, the most
they had was mere possessory right, not title. This possessory right was what was
transferred to Oh Cho, but since the latter is an alien, the possessory right could never
ripen to ownership by prescription. As an alien, Oh Cho is disqualified from acquiring
title over public land by prescription.
La Bugal-B’Laan v. Ramos
Chester Cabalza recommends his visitors to please read the original & full text of the case cited. Xie xie!

La Bugal-B’Laan v. Ramos
G.R. No. 127882.
December 1, 2004

Facts:

The Petition for Prohibition and Mandamus before the Court challenges the constitutionality of (1) Republic Act 7942
(The Philippine Mining Act of 1995); (2) its Implementing Rules and Regulations (DENR Administrative Order [DAO]
96-40); and (3) the Financial and Technical Assistance Agreement (FTAA) dated 30 March 1995, executed by the
government with Western Mining Corporation (Philippines), Inc. (WMCP).

On 27 January 2004, the Court en banc promulgated its Decision, granting the Petition and declaring the
unconstitutionality of certain provisions of RA 7942, DAO 96-40, as well as of the entire FTAA executed between the
government and WMCP, mainly on the finding that FTAAs are service contracts prohibited by the 1987 Constitution.
The Decision struck down the subject FTAA for being similar to service contracts,[9] which, though permitted under
the 1973 Constitution, were subsequently denounced for being antithetical to the principle of sovereignty over our
natural resources, because they allowed foreign control over the exploitation of our natural resources, to the prejudice
of the Filipino nation.

The Decision quoted several legal scholars and authors who had criticized service contracts for, inter alia, vesting in
the foreign contractor exclusive management and control of the enterprise, including operation of the field in the event
petroleum was discovered; control of production, expansion and development; nearly unfettered control over the
disposition and sale of the products discovered/extracted; effective ownership of the natural resource at the point of
extraction; and beneficial ownership of our economic resources. According to the Decision, the 1987 Constitution
(Section 2 of Article XII) effectively banned such service contracts. Subsequently, Victor O. Ramos (Secretary,
Department of Environment and Natural Resources [DENR]), Horacio Ramos (Director, Mines and Geosciences
Bureau [MGB-DENR]), Ruben Torres (Executive Secretary), and the WMC (Philippines) Inc. filed separate Motions
for Reconsideration.

Issue:

Whether or not the Court has a role in the exercise of the power of control over the EDU of our natural resources?

Held:

The Chief Executive is the official constitutionally mandated to “enter into agreements with foreign owned
corporations.” On the other hand, Congress may review the action of the President once it is notified of “every
contract entered into in accordance with this [constitutional] provision within thirty days from its execution.” In contrast
to this express mandate of the President and Congress in the exploration, development and utilization (EDU) of
natural resources, Article XII of the Constitution is silent on the role of the judiciary. However, should the President
and/or Congress gravely abuse their discretion in this regard, the courts may -- in a proper case -- exercise their
residual duty under Article VIII. Clearly then, the judiciary should not inordinately interfere in the exercise of this
presidential power of control over the EDU of our natural resources.

Under the doctrine of separation of powers and due respect for co-equal and coordinate branches of government, the
Court must restrain itself from intruding into policy matters and must allow the President and Congress maximum
discretion in using the resources of our country and in securing the assistance of foreign groups to eradicate the
grinding poverty of our people and answer their cry for viable employment opportunities in the country. “The judiciary
is loath to interfere with the due exercise by coequal branches of government of their official functions.” As aptly
spelled out seven decades ago by Justice George Malcolm, “Just as the Supreme Court, as the guardian of
constitutional rights, should not sanction usurpations by any other department of government, so should it as strictly
confine its own sphere of influence to the powers expressly or by implication conferred on it by the Organic Act.” Let
the development of the mining industry be the responsibility of the political branches of government. And let not the
Court interfere inordinately and unnecessarily. The Constitution of the Philippines is the supreme law of the land. It is
the repository of all the aspirations and hopes of all the people.

The Constitution should be read in broad, life-giving strokes. It should not be used to strangulate economic growth or
to serve narrow, parochial interests. Rather, it should be construed to grant the President and Congress sufficient
discretion and reasonable leeway to enable them to attract foreign investments and expertise, as well as to secure for
our people and our posterity the blessings of prosperity and peace. The Court fully sympathize with the plight of La
Bugal B’laan and other tribal groups, and commend their efforts to uplift their communities. However, the Court
cannot justify the invalidation of an otherwise constitutional statute along with its implementing rules, or the
nullification of an otherwise legal and binding FTAA contract. The Court believes that it is not unconstitutional to allow
a wide degree of discretion to the Chief Executive, given the nature and complexity of such agreements, the
humongous amounts of capital and financing required for large-scale mining operations, the complicated technology
needed, and the intricacies of international trade, coupled with the State’s need to maintain flexibility in its dealings, in
order to preserve and enhance our country’s competitiveness in world markets. On the basis of this control standard,
the Court upholds the constitutionality of the Philippine Mining Law, its Implementing Rules and Regulations - insofar
as they relate to financial and technical agreements - as well as the subject Financial and Technical Assistance
Agreement (FTAA).

La Bugal-B’laan Tribal Association, Inc. Vs Ramos


Natural Resources and Environmental Laws

G.R. No. 127882; January 27, 2004

FACTS:
This petition for prohibition and mandamus challenges the constitutionality of Republic Act
No. 7942 (The Philippine Mining Act of 1995), its implementing rules and regulations and
the Financial and Technical Assistance Agreement (FTAA) dated March 30, 1995 by the
government with Western Mining Corporation(Philippines) Inc. (WMCP).
Accordingly, the FTAA violated the 1987 Constitution in that it is a service contract and is
antithetical to the principle of sovereignty over our natural resources, because they allowed
foreign control over the exploitation of our natural resources, to the prejudice of the Filipino
nation.

ISSUE:
What is the proper interpretation of the phrase “Agreements involving Either Technical or
Financial Assistance” contained in paragraph 4, Section 2, Article XII of the Constitution.

HELD:
The Supreme Court upheld the constitutionality of the Philippine Mining Law, its
implementing rules and regulations – insofar as they relate to financial and technical
agreements as well as the subject Financial and Technical Assistance Agreement.
Full control is not anathematic to day-to-day management by the contractor, provided that
the State retains the power to direct overall strategy; and to set aside, reverse or modify
plans and actions of the contractor. The idea of full control is similar to that which is
exercised by the board of directors of a private corporation, the performance of managerial,
operational, financial, marketing and other functions may be delegated to subordinate
officers or given to contractual entities, but the board retains full residual control of the
business.

LA BUGAL B’LAAN TRIBAL ASSOCIATION INC., et. al. v. V. O. RAMOS,


Secretary Department of Environment and Natural Resources; H. RAMOS,
Director, Mines and Geosciences Bureau (MGB-DENR); R. TORRES,
Executive Secretary; and WMC (PHILIPPINES) INC.

The constitutional provision allowing the President to enter into FTAA is a exception to
the rule that participation in the nation’s natural resources is reserved exclusively to
Filipinos. Provision must be construed strictly against their enjoyment by non-Filipinos.
RA 7942 (The Philippine Mining Act) took effect on April 9, 1995. Before the effectivity
of RA 7942, or on March 30, 1995, the President signed a Financial and Technical
Assistance Agreement (FTAA) with WMCP, a corporation organized under Philippine
laws, covering close to 100,000 hectares of land in South Cotabato, Sultan Kudarat,
Davao del Sur and North Cotabato. On August 15, 1995, the Environment Secretary
Victor Ramos issued DENR Administrative Order 95-23, which was later repealed by
DENR Administrative Order 96-40, adopted on December 20, 1996.
Petitioners prayed that RA 7942, its implementing rules, and the FTAA between the
government and WMCP be declared unconstitutional on ground that they allow fully
foreign owned corporations like WMCP to exploit, explore and develop Philippine
mineral resources in contravention of Article XII Section 2 paragraphs 2 and 4 of the
Charter.
In January 2001, WMC – a publicly listed Australian mining and exploration company –
sold its whole stake in WMCP to Sagittarius Mines, 60% of which is owned by Filipinos
while 40% of which is owned by Indophil Resources, an Australian company. DENR
approved the transfer and registration of the FTAA in Sagittarius‘ name but Lepanto
Consolidated assailed the same. The latter case is still pending before the Court of
Appeals.
EO 279, issued by former President Aquino on July 25, 1987, authorizes the DENR to
accept, consider and evaluate proposals from foreign owned corporations or foreign
investors for contracts or agreements involving wither technical or financial assistance
for large scale exploration, development and utilization of minerals which upon
appropriate recommendation of the (DENR) Secretary, the President may execute with
the foreign proponent. WMCP likewise contended that the annulment of the FTAA
would violate a treaty between the Philippines and Australia which provides for the
protection of Australian investments.

ISSUES:

1. Whether or not the Philippine Mining Act is unconstitutional for allowing fully
foreign-owned corporations to exploit the Philippine mineral resources. 2. Whether or
not the FTAA between the government and WMCP is a ―service contract that permits
fully foreign owned companies to exploit the Philippine mineral resources.

HELD:

First Issue: RA 7942 is Unconstitutional


RA 7942 or the Philippine Mining Act of 1995 is unconstitutional for permitting fully
foreign owned corporations to exploit the Philippine natural resources.
Article XII Section 2 of the 1987 Constitution retained the Regalian Doctrine which
states that ―All lands of the public domain, waters, minerals, coal, petroleum, and other
minerals, coal, petroleum, and other mineral oils, all forces of potential energy,
fisheries, forests or timber, wildlife, flora and fauna, and other natural resources are
owned by the State. The same section also states that, ―the exploration and
development and utilization of natural resources shall be under the full control and
supervision of the State.
Conspicuously absent in Section 2 is the provision in the 1935 and 1973 Constitution
authorizing the State to grant licenses, concessions, or leases for the exploration,
exploitation, development, or utilization of natural resources. By such omission, the
utilization of inalienable lands of the public domain through license, concession or lease
is no longer allowed under the 1987 Constitution.
Under the concession system, the concessionaire makes a direct equity investment for
the purpose of exploiting a particular natural resource within a given area. The
concession amounts to complete control by the concessionaireover the country‘s natural
resource, for it is given exclusive and plenary rights to exploit a particular resource at
the point of extraction.
The 1987 Constitution, moreover, has deleted the phrase ―management or other forms
of assistance in the 1973 Charter. The present Constitution now allows only ―technical
and financial assistance. The management and the operation of the mining activities by
foreign contractors, the primary feature of the service contracts was precisely the evil
the drafters of the 1987 Constitution sought to avoid.
The constitutional provision allowing the President to enter into FTAAs is an exception
to the rule that participation in the nation‘s natural resources is reserved exclusively to
Filipinos. Accordingly, such provision must be construed strictly against their
enjoyment by non-Filipinos. Therefore, RA 7942 is invalid insofar as the said act
authorizes service contracts. Although the statute employs the phrase ―financial and
technical agreements in accordance with the 1987 Constitution, its pertinent provisions
actually treat these agreements as service contracts that grant beneficial ownership to
foreign contractors contrary to the fundamental law.
The underlying assumption in the provisions of the law is that the foreign contractor
manages the mineral resources just like the foreign contractor in a service contract. By
allowing foreign contractors to manage or operate all the aspects of the mining
operation, RA 7942 has, in effect, conveyed beneficial ownership over the nation‘s
mineral resources to these contractors, leaving the State with nothing but bare title
thereto.
The same provisions, whether by design or inadvertence, permit a circumvention of the
constitutionally ordained 60-40% capitalizationrequirement for corporations or
associations engaged in the exploitation, development and utilization of Philippine
natural resources.
When parts of a statute are so mutually dependent and connected as conditions,
considerations, inducements or compensations for each other as to warrant a belief that
the legislature intended them as a whole, then if some parts are unconstitutional, all
provisions that are thus dependent, conditional or connected, must fail with them.
Under Article XII Section 2 of the 1987 Charter, foreign owned corporations are limited
only to merely technical or financial assistance to the State for large scale exploration,
development and utilization of minerals, petroleum and other mineral oils.
Second Issue: RP Government-WMCP FTAA is a Service Contract
The FTAA between he WMCP and the Philippine government is likewise
unconstitutional since the agreement itself is a service contract.
Section 1.3 of the FTAA grants WMCP a fully foreign owned corporation, the exclusive
right to explore, exploit, utilize and dispose of all minerals and by-products that may be
produced from the contract area. Section 1.2 of the same agreement provides that EMCP
shall provide all financing, technology, management, and personnel necessary for the
Mining Operations.
These contractual stipulations and related provisions in the FTAA taken together, grant
WMCP beneficial ownership over natural resources that properly belong to the State
and are intended for the benefit of its citizens. These stipulations are abhorrent to the
1987 Constitution. They are precisely the vices that the fundamental law seeks to avoid,
the evils that it aims to suppress. Consequently, the contract from which they spring
must be struck down.
JG Summit Holdings Inc. vs. CA
on 7:00 AM in Case Digests, Commercial Law
0

G.R. No. 124293, November 20, 2000

FACTS:

The National Investment and Development Corporation (NIDC), a government corporation,


entered into a Joint Venture Agreement (JVA) with Kawasaki Heavy Industries, Ltd. for the
construction, operation and management of the Subic National Shipyard, Inc., later became the
Philippine Shipyard and Engineering Corporation (PHILSECO). Under the JVA, NIDC and
Kawasaki would maintain a shareholding proportion of 60%-40% and that the parties have the
right of first refusal in case of a sale.

Through a series of transfers, NIDC’s rights, title and interest in PHILSECO eventually went to
the National Government. In the interest of national economy, it was decided that PHILSECO
should be privatized by selling 87.67% of its total outstanding capital stock to private entities.
After negotiations, it was agreed that Kawasaki’s right of first refusal under the JVA be
“exchanged” for the right to top by five percent the highest bid for said shares. Kawasaki that
Philyards Holdings, Inc. (PHI), in which it was a stockholder, would exercise this right in its
stead.

During bidding, Kawasaki/PHI Consortium is the losing bidder. Even so, because of the right to
top by 5% percent the highest bid, it was able to top JG Summit’s bid. JG Summit protested,
contending that PHILSECO, as a shipyard is a public utility and, hence, must observe the 60%-
40% Filipino-foreign capitalization. By buying 87.67% of PHILSECO’s capital stock at bidding,
Kawasaki/PHI in effect now owns more than 40% of the stock.

ISSUE:

o Whether or not PHILSECO is a public utility


o Whether or not Kawasaki/PHI can purchase beyond 40% of PHILSECO’s stocks

HELD:

In arguing that PHILSECO, as a shipyard, was a public utility, JG Summit relied on sec. 13, CA
No. 146. On the other hand, Kawasaki/PHI argued that PD No. 666 explicitly stated that a
“shipyard” was not a “public utility.” But the SC stated that sec. 1 of PD No. 666 was expressly
repealed by sec. 20, BP Blg. 391 and when BP Blg. 391 was subsequently repealed by EO 226,
the latter law did not revive sec. 1 of PD No. 666. Therefore, the law that states that a shipyard
is a public utility still stands.

A shipyard such as PHILSECO being a public utility as provided by law is therefore required to
comply with the 60%-40% capitalization under the Constitution. Likewise, the JVA between
NIDC and Kawasaki manifests an intention of the parties to abide by this constitutional
mandate. Thus, under the JVA, should the NIDC opt to sell its shares of stock to a third party,
Kawasaki could only exercise its right of first refusal to the extent that its total shares of stock
would not exceed 40% of the entire shares of stock. The NIDC, on the other hand, may
purchase even beyond 60% of the total shares. As a government corporation and necessarily a
100% Filipino-owned corporation, there is nothing to prevent its purchase of stocks even
beyond 60% of the capitalization as the Constitution clearly limits only foreign capitalization.

Kawasaki was bound by its contractual obligation under the JVA that limits its right of first
refusal to 40% of the total capitalization of PHILSECO. Thus, Kawasaki cannot purchase
beyond 40% of the capitalization of the joint venture on account of both constitutional and
contractual proscriptions.

JG SUMMIT HOLDINGS, INC., vs. COURT OF APPEALS, COMMITTEE ON PRIVATIZATION, ASSET PRIVATIZATION
TRUST and PHILYARDS HOLDINGS G.R. No. 124293. November 20, 2000

FACTS:

National Investment and Development Corporation (NIDC) and Kawasaki Heavy Industries entered into a Joint
Venture Agreement in a shipyard business named PHILSECO, with a shareholding of 60-40 respectively. NIDC’s
interest was later transferred to the National Government.

Pursuant to President Aquino’s Proclamation No.5, which established the Committee on Privatization (COP) and Asset
Privatization Trust (APT), and allowed for the disposition of the government’s non-performing assets, the latter
allowed Kawasaki Heavy Industries to choose a company to which it has stockholdings, to top the winning bid of JG
Summit Holdings over PHILSECO. JG Summit protested alleging that such act would effectively increase Kawasaki’s
interest in PHILSECO—a shipyard is a public utility–and thus violative of the Constitution.

ISSUE:
Whether or not respondents’ act is valid.

HELD:
No.
A shipyard such as PHILSECO being a public utility as provided by law, the following provision of the Article XII of the
Constitution applies:
“Sec. 11. No franchise, certificate, or any other form of authorization for the operation of a public utility shall be granted except
to citizens of the Philippines or to corporations or associations organized under the laws of the Philippines at least sixty per
centum of whose capital is owned by such citizens, nor shall such franchise, certificate, or authorization be exclusive in character
or for a longer period than fifty years. Neither shall any such franchise or right be granted except under the condition that it shall
be subject to amendment, alteration, or repeal by the Congress when the common good so requires. The State shall encourage
equity participation in public utilities by the general public. The participation of foreign investors in the governing body of any
public utility enterprise shall be limited to their proportionate share in its capital, and all the executive and managing officers of
such corporation or association shall be citizens of the Philippines.”

Notably, paragraph 1.4 of the JVA accorded the parties the right of first refusal “under the same terms.” This phrase
implies that when either party exercises the right of first refusal under paragraph 1.4, they can only do so to the
extent allowed them by paragraphs 1.2 and 1.3 of the JVA or under the proportion of 60%-40% of the shares of
stock. Thus, should the NIDC opt to sell its shares of stock to a third party, Kawasaki could only exercise its right of
first refusal to the extent that its total shares of stock would not exceed 40% of the entire shares of stock of SNS or
PHILSECO. The NIDC, on the other hand, may purchase even beyond 60% of the total shares. As a government
corporation and necessarily a 100% Filipino-owned corporation, there is nothing to prevent its purchase of stocks
even beyond 60% of the capitalization as the Constitution clearly limits only foreign capitalization.

JG Summit Holdings vs CA Case Digest


JG Summit Holdings Inc. vs. Court of Appeals

[GR 124293, 20 November 2000]

Facts: On 27 January 1977, the National Investment and Development Corporation (NIDC), a
government corporation, entered into a Joint Venture Agreement (JVA) with Kawasaki Heavy
Industries, Ltd. of Kobe, Japan (Kawasaki) for the construction, operation, and management of the
Subic National Shipyard, Inc. (SNS), which subsequently became the Philippine Shipyard and
Engineering Corporation (PHILSECO). Under the JVA, NIDC and Kawasaki would maintain a
shareholding proportion of 60% - 40%, respectively. One of the provisions of the JVA accorded the
parties the right of first refusal should either party sell, assign or transfer its interest in the joint
venture. On 25 November 1986, NIDC transferred all its rights, title and interest in PHILSECO to the
Philippine National Bank (PNB).

More than two months later or on 3 February 1987, by virtue of Administrative Order 14, PNB's
interest in PHILSECO was transferred to the National Government. Meanwhile, on 8 December
1986, President Corazon C. Aquino issued Proclamation 50 establishing the Committee on
Privatization (COP) and the Asset Privatization Trust (APT) to take title to and possession of,
conserve, manage and dispose of non-performing assets of the National Government. On 27
February 1987, a trust agreement was entered into between the National Government and the APT
by virtue of which the latter was named the trustee of the National Government's share in
PHILSECO. In 1989, as a result of a quasi-reorganization of PHILSECO to settle its huge obligations
to PNB, the National Government's shareholdings in PHILSECO increased to 97.41% thereby
reducing Kawasaki's shareholdings to 2.59%. Exercising their discretion, the COP and the APT
deemed it in the best interest of the national economy and the government to privatize PHILSECO
by selling 87.67% of its total outstanding capital stock to private entities.
After a series of negotiations between the APT and Kasawaki, they agreed that the latter's right of
first refusal under the JVA be "exchanged" for the right to top by 5% the highest bid for said shares.
They further agreed that Kawasaki would be entitled to name a company in which it was a
stockholder, which could exercise the right to top. On 7 September 1990, Kawasaki informed APT
that Philyards Holdings, Inc. (PHI) would exercise its right to top by 5%. At the pre-bidding
conference held on 28 September 1993, interested bidders were given copies of the JVA between
NIDC and Kawasaki, and of the Asset Specific Bidding Rules (ASBR) drafted for the 87.67% equity
(sic) in PHILSECO of the National Government. The provisions of the ASBR were explained to the
interested bidders who were notified that bidding would be held on 2 December 1993. At the public
bidding on said date, the consortium composed of JG Summit Holdings, Inc. (JGSMI), Sembawang
Shipyard Ltd. of Singapore (Sembawang), and Jurong Shipyard Limited of Malaysia (Jurong), was
declared the highest bidder at P2.03 billion. The following day, the COP approved the sale of
87.67% National Government shares of stock in PHILSECO to said consortium. It notified JGSMI of
said approval "subject to the right of Kawasaki Heavy Industries, Inc./Philyards Holdings, Inc. to top
JGSMI's bid by 5% as specified in the bidding rules."

On 29 December 1993, JGSMI informed the APT that it was protesting the offer of PHI to top its bid
on the grounds that: (a) the Kawasaki/PHI consortium composed of Kawasaki, Philyards, Mitsui,
Keppel, SM Group, ICTSI and Insular Life violated the ASBR because the last four (4) companies
were the losing bidders (for P1.528 billion) thereby circumventing the law and prejudicing the weak
winning bidder; (b) only Kawasaki could exercise the right to top; (c) giving the same option to top to
PHI constituted unwarranted benefit to a third party; (d) no right of first refusal can be exercised in a
public bidding or auction sale, and (e) the JG Summit Consortium was not estopped from
questioning the proceedings. On 2 February 1994, JGSMI was notified that PHI had fully paid the
balance of the purchase price of the subject bidding. On 7 February 1994, the APT notified JGSMI
that PHI had exercised its option to top the highest bid and that the COP had approved the same on
6 January 1994. On 24 February 1994, the APT and PHI executed a Stock Purchase Agreement.
Consequently, JGSMI filed with the Supreme Court a petition for mandamus under GR 114057. On
11 May 1994, said petition was referred to the Court of Appeals. On 18 July 1995, the Court of
Appeals "denied" for lack of merit the petition for mandamus. JGSMI filed a motion for the
reconsideration of said Decision which was denied on 15 March 1996. JGSMI filed the petition for
review on certiorari.

Issue: Whether PHILSECO, as a shipyard, is a public utility and, hence, could be operated only by a
corporation at least 60% of whose capital is owned by Filipino citizens, in accordance with Article XII,
Section 10 of the Constitution.

Held: A shipyard such as PHILSECO being a public utility as provided by law, Section 11 of the
Article XII of the Constitution applies. The provision states that "No franchise, certificate, or any other
form of authorization for the operation of a public utility shall be granted except to citizens of the
Philippines or to corporations or associations organized under the laws of the Philippines at least
sixty per centum of whose capital is owned by such citizens, nor shall such franchise, certificate, or
authorization be exclusive in character or for a longer period than fifty years. Neither shall any such
franchise or right be granted except under the condition that it shall be subject to amendment,
alteration, or repeal by the Congress when the common good so requires. The State shall
encourage equity participation in public utilities by the general public. The participation of foreign
investors in the governing body of any public utility enterprise shall be limited to their proportionate
share in its capital, and all the executive and managing officers of such corporation or association
shall be citizens of the Philippines." The progenitor of this constitutional provision, Article XIV,
Section 5 of the 1973 Constitution, required the same proportion of 60% - 40% capitalization. The
JVA between NIDC and Kawasaki entered into on 27 January 1977 manifests the intention of the
parties to abide by the constitutional mandate on capitalization of public utilities. The joint venture
created between NIDC and Kawasaki falls within the purview of an "association" pursuant to Section
5 of Article XIV of the 1973 Constitution and Section 11 of Article XII of the 1987 Constitution.
Consequently, a joint venture that would engage in the business of operating a public utility, such as
a shipyard, must observe the proportion of 60%-40% Filipino-foreign capitalization. Further,
paragraph 1.4 of the JVA accorded the parties the right of first refusal "under the same terms." This
phrase implies that when either party exercises the right of first refusal under paragraph 1.4, they
can only do so to the extent allowed them by paragraphs 1.2 and 1.3 of the JVA or under the
proportion of 60%-40% of the shares of stock. Thus, should the NIDC opt to sell its shares of stock
to a third party, Kawasaki could only exercise its right of first refusal to the extent that its total shares
of stock would not exceed 40% of the entire shares of stock of SNS or PHILSECO. The NIDC, on
the other hand, may purchase even beyond 60% of the total shares. As a government corporation
and necessarily a 100% Filipino-owned corporation, there is nothing to prevent its purchase of
stocks even beyond 60% of the capitalization as the Constitution clearly limits only foreign
capitalization. Kawasaki was bound by its contractual obligation under the JVA that limits its right of
first refusal to 40% of the total capitalization of PHILSECO. Thus, Kawasaki cannot purchase
beyond 40% of the capitalization of the joint venture on account of both constitutional and
contractual proscriptions. From the facts on record, it appears that at the outset, the APT and
Kawasaki respected the 60%-40% capitalization proportion in PHILSECO. However, APT
subsequently encouraged Kawasaki to participate in the public bidding of the National Government's
shareholdings of 87.67% of the total PHILSECO shares, definitely over and above the 40% limit of
its shareholdings. In so doing, the APT went beyond the ambit of its authority.
Francisco Tatad vs Jesus Garcia,
Jr.
November 27, 2012
Share this...

1 0 0 0

ADVERTISEMENTS

243 SCRA 436 – Business Organization – Corporation Law – Corporate Nationality – Public
Utility – Nationality Requirement in Nationalized Areas of Activity
In 1989, the government planned to build a railway transit line along EDSA. No bidding was
made but certain corporations were invited to prequalify. The only corporation to qualify was
the EDSA LRT Consortium which was obviously formed for this particular undertaking. An
agreement was then made between the government, through the Department of
Transportation and Communication (DOTC), and EDSA LRT Consortium. The agreement
was based on the Build-Operate-Transfer scheme provided for by law (RA 6957, amended
by RA 7718). Under the agreement, EDSA LRT Consortium shall build the facilities, i.e.,
railways, and shall supply the train cabs. Every phase that is completed shall be turned over
to the DOTC and the latter shall pay rent for the same for 25 years. By the end of 25 years,
it was projected that the government shall have fully paid EDSA LRT Consortium.
Thereafter, EDSA LRT Consortium shall sell the facilities to the government for $1.00.
However, Senators Francisco Tatad, John Osmeña, and Rodolfo Biazon opposed the
implementation of said agreement as they averred that EDSA LRT Consortium is a foreign
corporation as it was organized under Hongkong laws; that as such, it cannot own a public
utility such as the EDSA railway transit because this falls under the nationalized areas of
activities. The petition was filed against Jesus Garcia, Jr. in his capacity as DOTC
Secretary.
ISSUE: Whether or not the petition shall prosper.
HELD: No. The Supreme Court made a clarification. The SC ruled that EDSA LRT
Consortium, under the agreement, does not and will not become the owner of a public utility
hence, the question of its nationality is misplaced. It is true that a foreign corporation cannot
own a public utility but in this case what EDSA LRT Consortium will be owning are the
facilities that it will be building for the EDSA railway project. There is no prohibition against a
foreign corporation to own facilities used for a public utility. Further, it cannot be said that
EDSA LRT Consortium will be the one operating the public utility for it will be DOTC that will
operate the railway transit. DOTC will be the one exacting fees from the people for the use
of the railway and from the proceeds, it shall be paying the rent due to EDSA LRT
Consortium. All that EDSA LRT Consortium has to do is to build the facilities and receive
rent from the use thereof by the government for 25 years – it will not operate the railway
transit. Although EDSA LRT Consortium is a corporation formed for the purpose of building
a public utility it does not automatically mean that it is operating a public utility. The moment
for determining the requisite Filipino nationality is when the entity applies for a franchise,
certificate or any other form of authorization for that purpose.

TATAD V. GARCIA, JR. (G.R. NO. 114222)


Facts:
DOTC planned to construct a light railway transit line along EDSA (EDSA
LRT III) to provide a mass transit system and alleviate the congestion and
growing transportation problem in the metropolis. RA 6957 was enacted
allowing for the financing, construction and operation of government projects
through private initiative and investment. Accordingly, prequalification and
bidding was made and EDSA LRT Corporation (organized under HK laws)
was recommended to be awarded with the contract. The President approved
the awarding of the contract. Petitioners are senators praying for the
prohibition of respondents from further implementing and enforcing the
contract.
Issue:
Whether or not the EDSA LRT III, a public utility, can be owned by a foreign
corporation.
Ruling: YES.
The Constitution, in no uncertain terms, requires a franchise for the operation
of a public utility. However, it does not require a franchise before one can
own the facilities needed to operate a public utility so long as it does not
operate them to serve the public.
In law, there is a clear distinction between the “operation” of a public utility
and the ownership of the facilities and equipment used to serve the public.
Ownership is defined as a relation in law by virtue of which a thing
pertaining to one person is completely subjected to his will in everything not
prohibited by law or the concurrence with the rights of another. The exercise
of the rights encompassed in ownership is limited by law so that a property
cannot be operated and used to serve the public as a public utility unless the
operator has a franchise. The operation of a rail system as a public utility
includes the transportation of passengers from one point to another point,
their loading and unloading at designated places and the movement of the
trains at pre-scheduled times.
In sum, private respondent will not run the light rail vehicles and collect fees
from the riding public. It will have no dealings with the public and the public
will have no right to demand any services from it. Even the mere formation
of a public utility corporation does not ipso facto characterize the corporation
as one operating a public utility. The moment for determining the requisite
Filipino nationality is when the entity applies for a franchise, certificate or
any other form of authorization for that purpose.
JACOBUS BERNHARD HULST, Petitioner v. PR BUILDERS, INC., Respondent.

RESOLUTION

AUSTRIA-MARTINEZ, J.:

This resolves petitioner's Motion for Partial Reconsideration.

On September 3, 2007, the Court rendered a Decision1 in the present case, the dispositive portion of which
reads:

WHEREFORE, the instant petition is GRANTED. The Decision dated October 30, 2002 of the Court of Appeals
in CA-G.R. SP No. 60981 is REVERSED and SET ASIDE. The Order dated August 28, 2000 of HLURB Arbiter
Ma. Perpetua Y. Aquino and Director Belen G. Ceniza in HLRB Case No. IV6-071196-0618 is declared NULL
and VOID. HLURB Arbiter Aquino and Director Ceniza are directed to issue the corresponding certificates of
sale in favor of the winning bidder, Holly Properties Realty Corporation. Petitioner is ordered to return to
respondent the amount of P2,125,540.00, without interest, in excess of the proceeds of the
auction sale delivered to petitioner. After the finality of herein judgment, the amount of P2,125,540.00
shall earn 6% interest until fully paid.

SO ORDERED.2 (Emphasis supplied) cra lawlib rary

Petitioner filed the present Motion for Partial Reconsideration3 insofar as he was ordered to return to
respondent the amount of P2,125,540.00 in excess of the proceeds of the auction sale delivered to
petitioner. Petitioner contends that the Contract to Sell between petitioner and respondent involved a
condominium unit and did not violate the Constitutional proscription against ownership of land by aliens. He
argues that the contract to sell will not transfer to the buyer ownership of the land on which the unit is
situated; thus, the buyer will not get a transfer certificate of title but merely a Condominium Certificate of
Title as evidence of ownership; a perusal of the contract will show that what the buyer acquires is the
seller's title and rights to and interests in the unit and the common areas.

Despite receipt of this Court's Resolution dated February 6, 2008, respondent failed to file a comment on the
subject motion.

The Motion for Partial Reconsideration is impressed with merit.

The Contract to Sell between petitioner and respondent provides as follows:

Section 3. TITLE AND OWNERSHIP OF UNIT

A. Upon full payment by the BUYER of the purchase price stipulated in Section 2 hereof, x x x, the SELLER
shall deliver to the BUYER the Deed of Absolute Sale conveying its rights, interests and title to the
UNIT and to the common areas appurtenant to such UNIT, and the corresponding Condominium
Certificate of Title in the SELLER's name; x x x

b. The Seller shall register with the proper Registry of Deeds, the Master Deed with the Declaration of
Restrictions and other documents and shall immediately comply with all requirements of Republic Act No.
4726 (The Condominium Act) and Presidential Decree No. 957 (Regulating the Sale of Subdivision Lots
and Condominiums, Providing Penalties for Violations Thereof). It is hereby understood that all title, rights
and interest so conveyed shall be subject to the provisions of the Condominium Act, the Master
Deed with Declaration of Restrictions, the Articles of Incorporation and By-Laws and the Rules and
Regulations of the Condominium Corporation, zoning regulations and such other restrictions on the use of
the property as annotated on the title or may be imposed by any government agency or instrumentality
having jurisdiction thereon.4(Emphasis supplied) c ralawl ibra ry
Under Republic Act (R.A.) No. 4726, otherwise known as the Condominium Act, foreign nationals can own
Philippine real estate through the purchase of condominium units or townhouses constituted under the
Condominium principle with Condominium Certificates of Title. Section 5 of R.A. No. 4726 states:

SECTION 5. Any transfer or conveyance of a unit or an apartment, office or store or other space therein,
shall include the transfer or conveyance of the undivided interest in the common areas or, in a proper case,
the membership or shareholdings in the condominium corporation; Provided, however, That where the
common areas in the condominium project are held by the owners of separate units as co-owners thereof,
no condominium unit therein shall be conveyed or transferred to persons other than Filipino citizens or
corporations at least 60% of the capital stock of which belong to Filipino citizens, except in cases of
hereditary succession. Where the common areas in a condominium project are held by a
corporation, no transfer or conveyance of a unit shall be valid if the concomitant transfer of the
appurtenant membership or stockholding in the corporation will cause the alien interest in such
corporation to exceed the limits imposed by existing laws. (Emphasis supplied) cral awlib rary

The law provides that no condominium unit can be sold without at the same time selling the corresponding
amount of rights, shares or other interests in the condominium management body, the Condominium
Corporation; and no one can buy shares in a Condominium Corporation without at the same time buying a
condominium unit. It expressly allows foreigners to acquire condominium units and shares in condominium
corporations up to not more than 40% of the total and outstanding capital stock of a Filipino-owned or
controlled corporation. Under this set up, the ownership of the land is legally separated from the unit itself.
The land is owned by a Condominium Corporation and the unit owner is simply a member in this
Condominium Corporation.5 As long as 60% of the members of this Condominium Corporation are Filipino,
the remaining members can be foreigners.

Considering that the rights and liabilities of the parties under the Contract to Sell is covered by the
Condominium Act wherein petitioner as unit owner was simply a member of the Condominium Corporation
and the land remained owned by respondent, then the constitutional proscription against aliens owning real
property does not apply to the present case. There being no circumvention of the constitutional prohibition,
the Court's pronouncements on the invalidity of the Contract of Sale should be set aside.

WHEREFORE, the Motion for Partial Reconsideration is GRANTED. Accordingly, the Decision dated
September 3, 2007 of the Court is MODIFIED by deleting the order to petitioner to return to respondent
the amount of P2,125,540.00 in excess of the proceeds of the auction sale delivered to petitioner.

SO ORDERED.
Agan v PIATCO G.R. No. 155001. May 5, 2003

7/6/2010

0 Comments

Facts: In August 1989, the DOTC engaged the services of Aeroport de Paris (ADP) to conduct a comprehensive
study of the Ninoy Aquino International Airport (NAIA) and determine whether the present airport can cope with the
traffic development up to the year 2010.

On March 27, 1995, then DOTC Secretary Jose Garcia endorsed the proposal of Asia's Emerging Dragon
Corp. (unsolicited proposal dated Oct. 5, 1994) to the National Economic and Development Authority (NEDA). A
revised proposal, however, was forwarded by the DOTC to NEDA on December 13, 1995. On January 5, 1996, the
NEDA Investment Coordinating Council (NEDA ICC) — Technical Board favorably endorsed the project to the ICC —
Cabinet Committee which approved the same, subject to certain conditions, on January 19, 1996. On February 13,
1996, the NEDA passed Board Resolution No. 2 which approved the NAIA IPT III Project.

On August 29, 1996, the Second Pre-Bid Conference was held where certain clarifications were made. Upon the
request of prospective bidder People's Air Cargo & Warehousing Co., Inc (Paircargo), the PBAC warranted that
based on Sec. 11.6, Rule 11 of the Implementing Rules and Regulations of the BOT Law, only the proposed Annual
Guaranteed Payment submitted by the challengers would be revealed to AEDC, and that the challengers' technical
and financial proposals would remain confidential. The PBAC also clarified that the list of revenue sources contained
in Annex 4.2a of the Bid Documents was merely indicative and that other revenue sources may be included by the
proponent, subject to approval by DOTC/MIAA. Furthermore, the PBAC clarified that only those fees and charges
denominated as Public Utility Fees would be subject to regulation, and those charges which would be actually
deemed Public Utility Fees could still be revised, depending on the outcome of PBAC's query on the matter with the
Department of Justice.

On September 26, 1996, AEDC informed the PBAC in writing of its reservations as regards the Paircargo
Consortium, which include:
a. The lack of corporate approvals and financial capability of PAIRCARGO;
b. The lack of corporate approvals and financial capability of PAGS;
c. The prohibition imposed by RA 337, as amended (the General Banking Act) on the amount that Security Bank
could legally invest in the project;
d. The inclusion of Siemens as a contractor of the PAIRCARGO Joint Venture, for prequalification purposes; and
e. The appointment of Lufthansa as the facility operator, in view of the Philippine requirement in the operation of
a public utility.

The PBAC gave its reply on October 2, 1996, informing AEDC that it had considered the issues raised by the
latter, and that based on the documents submitted by Paircargo and the established prequalification criteria, the
PBAC had found that the challenger, Paircargo, had prequalified to undertake the project. The Secretary of the DOTC
approved the finding of the PBAC.

On October 16, 1996, the PBAC opened the third envelope submitted by AEDC and the Paircargo Consortium
containing their respective financial proposals. Both proponents offered to build the NAIA Passenger Terminal III for
at least $350 million at no cost to the government and to pay the government: 5% share in gross revenues for the first
five years of operation, 7.5% share in gross revenues for the next ten years of operation, and 10% share in gross
revenues for the last ten years of operation, in accordance with the Bid Documents.

As AEDC failed to match the proposal within the 30-day period, then DOTC Secretary Amado Lagdameo, on
December 11, 1996, issued a notice to Paircargo Consortium regarding AEDC's failure to match the proposal. AEDC
subsequently protested the alleged undue preference given to PIATCO and reiterated its objections as regards the
prequalification of PIATCO.

On July 12, 1997, the Government, through then DOTC Secretary Arturo T. Enrile, and PIATCO, through its
President, Henry T. Go, signed the "Concession Agreement for the Build-Operate-and-Transfer Arrangement of the
Ninoy Aquino International Airport Passenger Terminal III" (1997 Concession Agreement). The Government granted
PIATCO the franchise to operate and maintain the said terminal during the concession period and to collect the fees,
rentals and other charges in accordance with the rates or schedules stipulated in the 1997 Concession Agreement.
The Agreement provided that the concession period shall be for twenty-five (25) years commencing from the in-
service date, and may be renewed at the option of the Government for a period not exceeding twenty-five (25) years.
At the end of the concession period, PIATCO shall transfer the development facility to MIAA.

During the pendency of the case before this Court, President Gloria Macapagal Arroyo, on November 29, 2002, in
her speech at the 2002 Golden Shell Export Awards at Malacañang Palace, stated that she will not "honor (PIATCO)
contracts which the Executive Branch's legal offices have concluded (as) null and void."

Issue: Whether the petitioners and the petitioners-in-intervention have standing;


Whether this Court has jurisdiction; and
Whether the BOT and contracts therein are unconstitutional.

Held: YES.
Ratio: Messrs. Lopez et al. are employees of the MIAA. These petitioners (Messrs. Agan et al. and Messrs. Lopez
et al.) are confronted with the prospect of being laid off from their jobs and losing their means of livelihood when their
employer-companies are forced to shut down or otherwise retrench and cut back on manpower. Such development
would result from the imminent implementation of certain provisions in the contracts that tend toward the creation of a
monopoly in favor of Piatco, its subsidiaries and related companies.

Petitioners-in-intervention are service providers in the business of furnishing airport-related services to


international airlines and passengers in the NAIA and are therefore competitors of Piatco as far as that line of
business is concerned. On account of provisions in the Piatco contracts, petitioners-in-intervention have to enter into
a written contract with Piatco so as not to be shut out of NAIA Terminal III and barred from doing business there.
Since there is no provision to ensure or safeguard free and fair competition, they are literally at its mercy. They claim
injury on account of their deprivation of property (business) and of the liberty to contract, without due process of law.

By way of background, two monopolies were actually created by the Piatco contracts. The first and more obvious
one refers to the business of operating an international passenger terminal in Luzon, the business end of which
involves providing international airlines with parking space for their aircraft, and airline passengers with the use of
departure and arrival areas, check-in counters, information systems, conveyor systems, security equipment and
paraphernalia, immigrations and customs processing areas; and amenities such as comfort rooms, restaurants and
shops.

In furtherance of the first monopoly, the Piatco Contracts stipulate that the NAIA Terminal III will be the only facility
to be operated as an international passenger terminal; that NAIA Terminals I and II will no longer be operated as
such; and that no one (including the government) will be allowed to compete with Piatco in the operation of an
international passenger terminal in the NAIA Complex. Given that, at this time, the government and Piatco are the
only ones engaged in the business of operating an international passenger terminal, I am not acutely concerned with
this particular monopolistic situation.

There was however another monopoly within the NAIA created by the subject contracts for Piatco — in the
business of providing international airlines with the following: groundhandling, in-flight catering, cargo handling, and
aircraft repair and maintenance services. These are lines of business activity in which are engaged many service
providers (including the petitioners-in-intervention), who will be adversely affected upon full implementation of the
Piatco Contracts, particularly Sections 3.01(d) and (e) of both the ARCA and the CA.
Should government pay at all for reasonable expenses incurred in the construction of the Terminal? Indeed it
should, otherwise it will be unjustly enriching itself at the expense of Piatco and, in particular, its funders, contractors
and investors — both local and foreign. After all, there is no question that the State needs and will make use of
Terminal III, it being part and parcel of the critical infrastructure and transportation-related programs of government.

The rule on hierarchy of courts will not also prevent this Court from assuming jurisdiction over the cases at bar.
The said rule may be relaxed when the redress desired cannot be obtained in the appropriate courts or where
exceptional and compelling circumstances justify availment of a remedy within and calling for the exercise of this
Court's primary jurisdiction. Thus, considering the nature of the controversy before the Court, procedural bars may be
lowered to give way for the speedy disposition of the instant cases.

In sum, this Court rules that in view of the absence of the requisite financial capacity of the Paircargo Consortium,
predecessor of respondent PIATCO, the award by the PBAC of the contract for the construction, operation and
maintenance of the NAIA IPT III is null and void.

Is PIATCO a qualified bidder?


Public respondents argue that the Paircargo Consortium, PIATCO's predecessor, was not a duly pre-qualified
bidder on the unsolicited proposal submitted by AEDC as the Paircargo Consortium failed to meet the financial
capability required under the BOT Law and the Bid Documents. They allege that in computing the ability of the
Paircargo Consortium to meet the minimum equity requirements for the project, the entire net worth of Security Bank,
a member of the consortium, should not be considered. R.A. No. 337, as amended or the General Banking Act that a
commercial bank cannot invest in any single enterprise in an amount more than 15% of its net worth.

We agree with public respondents that with respect to Security Bank, the entire amount of its net worth could not
be invested in a single undertaking or enterprise, whether allied or non-allied in accordance with the provisions of
R.A. No. 337
The PBAC should not be allowed to speculate on the future financial ability of the bidder to undertake the project
on the basis of documents submitted. This would open doors to abuse and defeat the very purpose of a public
bidding.

Is the 1997 Concession Agreement valid?


Petitioners and public respondents contend that the 1997 Concession Agreement is invalid as it contains
provisions that substantially depart from the draft Concession Agreement included in the Bid Documents. They
maintain that a substantial departure from the draft Concession Agreement is a violation of public policy and renders
the 1997 Concession Agreement null and void.

If the winning bidder is allowed to later include or modify certain provisions in the contract awarded such that the
contract is altered in any material respect, then the essence of fair competition in the public bidding is destroyed. A
public bidding would indeed be a farce if after the contract is awarded; the winning bidder may modify the contract
and include provisions which are favorable to it that were not previously made available to the other bidders.

With respect to terminal fees that may be charged by PIATCO, as shown earlier, this was included within the
category of "Public Utility Revenues" under the 1997 Concession Agreement. This classification is significant
because under the 1997 Concession Agreement, "Public Utility Revenues" are subject to an "Interim Adjustment" of
fees upon the occurrence of certain extraordinary events specified in the agreement. However, under the draft
Concession Agreement, terminal fees are not included in the types of fees that may be subject to "Interim
Adjustment."

Finally, under the 1997 Concession Agreement, "Public Utility Revenues," except terminal fees, are denominated
in US Dollars while payments to the Government are in Philippine Pesos. In the draft Concession Agreement, no
such stipulation was included. By stipulating that "Public Utility Revenues" will be paid to PIATCO in US Dollars while
payments by PIATCO to the Government are in Philippine currency under the 1997 Concession Agreement, PIATCO
is able to enjoy the benefits of depreciations of the Philippine Peso, while being effectively insulated from the
detrimental effects of exchange rate fluctuations.

Under the draft Concession Agreement, default by PIATCO of any of its obligations to creditors who have
provided, loaned or advanced funds for the NAIA IPT III project does not result in the assumption by the Government
of these liabilities. In fact, nowhere in the said contract does default of PIATCO's loans figure in the agreement. Such
default does not directly result in any concomitant right or obligation in favor of the Government.

It is clear from the above-quoted provisions that Government, in the event that PIATCO defaults in its loan
obligations, is obligated to pay "all amounts recorded and from time to time outstanding from the books" of PIATCO
which the latter owes to its creditors. These amounts include "all interests, penalties, associated fees, charges,
surcharges, indemnities, reimbursements and other related expenses." This obligation of the Government to pay
PIATCO's creditors upon PIATCO's default would arise if the Government opts to take over NAIA IPT III. It should be
noted, however, that even if the Government chooses the second option, which is to allow PIATCO's unpaid creditors
operate NAIA IPT III, the Government is still at a risk of being liable to PIATCO's creditors should the latter be unable
to designate a qualified operator within the prescribed period. In effect, whatever option the Government chooses to
take in the event of PIATCO's failure to fulfill its loan obligations, the Government is still at a risk of assuming
PIATCO's outstanding loans.

As such the Government is virtually at the mercy of PIATCO (that it would not default on its loan obligations to its
Senior Lenders), the Senior Lenders (that they would appoint a qualified nominee or transferee or agree to some
other arrangement with the Government) and the existence of a qualified nominee or transferee who is able and
willing to take the place of PIATCO in NAIA IPT III.

In view of regulation of monopolies


The operation of an international passenger airport terminal is no doubt an undertaking imbued with public
interest. In entering into a Build-Operate-and-Transfer contract for the construction, operation and maintenance of
NAIA IPT III, the government has determined that public interest would be served better if private sector resources
were used in its construction and an exclusive right to operate be granted to the private entity undertaking the said
project, in this case PIATCO. Nonetheless, the privilege given to PIATCO is subject to reasonable regulation and
supervision by the Government through the MIAA, which is the government agency authorized to operate the NAIA
complex, as well as DOTC, the department to which MIAA is attached.

While it is the declared policy of the BOT Law to encourage private sector participation by "providing a climate of
minimum government regulations," the same does not mean that Government must completely surrender its
sovereign power to protect public interest in the operation of a public utility as a monopoly. The operation of said
public utility cannot be done in an arbitrary manner to the detriment of the public which it seeks to serve.

In contrast to the arrastre and stevedoring service providers in the case of Anglo-Fil Trading Corporation v. Lazaro
whose contracts consist of temporary hold-over permits, the affected service providers in the cases at bar, have a
valid and binding contract with the Government, through MIAA, whose period of effectivity, as well as the other terms
and conditions thereof cannot be violated.

Should the dispute be referred to arbitration prior to judicial recourse?


Respondent Piatco claims that Section 10.02 of the Amended and Restated Concession Agreement (ARCA)
provides for arbitration under the auspices of the International Chamber of Commerce to settle any dispute or
controversy or claim arising in connection with the Concession Agreement, its amendments and supplements. The
government disagrees; however, insisting that there can be no arbitration based on Section 10.02 of the ARCA, since
all the Piatco contracts are void ab initio.
G.R. No. L-27170 November 22, 1977

IN RE APPLICATION FOR THE REGISTRATION OF LAND. EUGENE MOSS, through his


Administrator, DR. TEODORICO H. JACELDO, applicant-appellant,
vs.
DIRECTOR OF LANDS, oppositor-appellee.

Estanislao L. Granados for appellant.

Solicitor General Estelito P. Mendoza, Assistant Solicitor General Eulogio Raquel-Santos and
Solicitor Salvador C. Jacob for appellee.

AQUINO, J.:

Eugene Moss appealed from the decision of the Court of First Instance of Leyte, Carigara Branch VI,
denying his application for the registration of a ten-hectare island on the ground that, being an
American citizen or an alien, he is disqualified to acquire lands under section 5, Article XIII of the
1935 Constitution, as held in Krivenko vs. Register of Deeds, 79 Phil. 461 (Land Registration Case
No. N-68, LRC Record No. N-27971).

The Solicitor General, disagreeing with the trial court, recommends in his brief for the Director of
Lands that the decision be reversed and that the application of Moss be granted.

On Carigara Bay there is an islet known as Calumpihan Island within the jurisdiction of Barrio
Calumpihan, Capoocan, Leyte. It is planted to coconuts more than sixty to eighty years old in 1966.
Fruit trees, corn and tobacco are also grown on the land.

That land was already declared for tax p in 1930. It was then owned by Vicente Pragas who had
possessed and cultivated it since 1894. On November 14, 1932 Pragas sold the island to Eduardo
Cecilio. On March 23, 1937 Cecilio sold it to Catalina Pabilion. Then, on December 23, 1944 the
spouses Catalina Pabilion and Guillermo Dadizon sold the land to Rufino M. Pascual who, in turn
sold it on January 20, 1945 for P 1,200 to Eugene Moss and Albert Boyd Cassidy, American
nationals.

In an action to quiet title filed by Moss against Cassidy, an absentee, Moss was ajudged the sole
owner of the and in a decision dated March 27, 1962 rendered by the Court of First Instance of Leyte
in Civil Case No. 645.

Moss declared the land for tax purposes on June 27, 1950. He paid the realty taxes on the said land
for the years 1945 to 1966. He had it surveyed on November 20, 1962.

On April 3, 1965 Moss, through Dr. Teodorico H. Jaceldo, his administrator and attorney-in-fact, filed
an application for the registration of the said land. Moss is a retired army colonel, an American
citizen, married to Eileen Moss and a former resident of 504 Glen Park Drive, Nashville, Tennessee
and now Los Fresnos Texas.

As already stated, the trial court denied the application for registration because Moss is an alien.
That holding is erroneous. It is erroneous because while aliens are disqualified to Acquire lands
under the 1935 Constitution (Levy Hermanos vs. Ledesma, 69 Phil. 49; Krivenko vs. Register of
Deeds, supra), citizens of the United States can acquire lands like Filipino citizens.

The trial court overlooked the Ordinance which was appended to the 1935 Constitution by
Resolution No. 39 of the National Assembly dated September 15, 1939. That resolution was
approved by the President of the United States on November 10, 1939. It provides as follows:

ORDINANCE APPENDED TO THE CONSTITUTION

SECTION 1. Notwithstanding the provisions of the foregoing Constitution, pending


the final and complete withdrawal of the sovereignty of the United States over the
Philippines —

xxx xxx xxx

(17) Citizens and corporations of the United States shall enjoy in the Commonwealth
of the Philippines all the civil rights of the citizens and corporations, respectively,
thereof. (1 Philippine Annotated Laws, pp. 31-34).

That Ordinance was made a part of the 1935 Constitution as directed in section 2 of the Tydings-
McDuffie Law or the Independence law. Subsection (17) quoted above is a reproduction of
subsection (16) of section 2(a) of the Independence Law.

Moss validly acquired the island in question under the provisions of subsection (17), section I of the
aforementioned Ordinance (See sec. 127 of Public Land Law; Republic vs. Quasha, L-30299,
August 17, 1972, 46 SCRA 160, 172-173. Note that the instant case is not covered by section 11,
Article XVII of the 1973 Constitution and by Presidential Decree No. 713 dated May 27, 1975, 71
O.G. 4115 re residential lands of American citizens).

The proclamation of Philippine independence on July 4, 1946, did not impair Moss' proprietary fights
over the said land because the 1935 Constitution provides that upon the proclamation of Philippine
independence 'all existing property rights of citizens or corporations of the United States shall be
acknowledged, respected, and safeguarded to the same extent as property right of citizens of the
Philippines" (Sec. 1[1], Article XVII).

That constitutional provision is implemented in Article VI of the Treaty of General Relations entered
into between the Republic of the Philippines and the United States on July 4, 1946 which provides
that all existing property rights of citizens and corporations of the United States in the Republic of the
Philippines shall be acknowledged, respected and safeguarded to the same extent as property rights
of citizens and corporation of the Republic of the Philippines (42 O.G. 1651. See Republic Act No.
76).

When Moss purchased the land, Leyte was already liberated as shown in the proclamation of
General Douglas MacArthur dated October 23, 1944. (Moss and Cassidy were allegedly members of
the American liberation forces that landed in Leyte). See 41 O.G. 147.

Since Moss and his predecessors in interest have been in possession en concepts de dueño of
Calumpihan Island for more than thirty years immediately preceding the firing of his application for
confirmation of his title, he is entitled to the registration of his title to the island (Sec. 48[b], Public
Land Law, as amended by Republic Act No. 1942).
WHEREFORE, the trial court's decision is reversed and set aside and the application of Eugene
Moss, of age, citizen of the United States, married to Eileen Moss, P. O. Box 184, Los Fresnos
Texas, U. S. A., for the confirmation and registration of his title to the land shown in the plan Psu-
198022, is hereby granted.

SO ORDERED.
BOARD OF MEDICINE, G.R. No. 166097
DR. RAUL FLORES
(now DR. JOSE S. RAMIREZ), Present:
in his capacity as Chairman of the
Board, PROFESSIONAL QUISUMBING,* J.,
REGULATION COMMISSION, YNARES-SANTIAGO,
through its Chairman, Chairperson,
HERMOGENES POBRE AUSTRIA-MARTINEZ,
(now DR. ALCESTIS M. GUIANG), NACHURA, and
Petitioners, REYES, JJ.

-versus-

Promulgated:
YASUYUKI OTA, July 14, 2008
Respondent.
x -- -- -- -- -- -- -- -- -- -- -- -- -- -- -- -- -- -- -- -- -- -- -- -- -- -- -- -- -- -- -- -- x

DECISION

AUSTRIA-MARTINEZ, J.:

Before the Court is a Petition for Review on Certiorari assailing the Decision[1] of
the Court of Appeals (CA) in CA-G.R. SP No. 84945[2] dated November 16,
2004 which affirmed the Decision[3] of the Regional Trial Court (RTC), Branch
22, Manila, dated October 19, 2003.[4]

The facts are as follows:

Yasuyuki Ota (respondent) is a Japanese national, married to a Filipina, who has


continuously resided in the Philippines for more than 10 years. He graduated
from BicolChristian College of Medicine on April 21, 1991 with a degree of
Doctor of Medicine.[5] After successfully completing a one-year post graduate
internship training at the JoseReyes Memorial Medical Center, he filed an
application to take the medical board examinations in order to obtain a medical
license. He was required by the Professional Regulation Commission (PRC) to
submit an affidavit of undertaking, stating among others that should he
successfully pass the same, he would not practice medicine until he submits proof
that reciprocity exists between Japan and the Philippines in admitting foreigners
into the practice of medicine.[6]

Respondent submitted a duly notarized English translation of the Medical


Practitioners Law of Japan duly authenticated by the Consul General of the
Philippine Embassy to Japan, Jesus I. Yabes;[7] thus, he was allowed to take the
Medical Board Examinations in August 1992, which he subsequently passed.[8]

In spite of all these, the Board of Medicine (Board) of the PRC, in a letter
dated March 8, 1993, denied respondent's request for a license to

practice medicine in the Philippines on the ground that the Board believes that no
genuine reciprocity can be found in the law of Japan as there is no Filipino or
foreigner who can possibly practice there.[9]

Respondent then filed a Petition for Certiorari and Mandamus against the Board
before the RTC of Manila on June 24, 1993, which petition was amended
on February 14, 1994to implead the PRC through its Chairman.[10]

In his petition before the RTC, respondent alleged that the Board and the PRC, in
refusing to issue in his favor a Certificate of Registration and/or license to practice
medicine, had acted arbitrarily, in clear contravention of the provision of Section
20 of Republic Act (R.A.) No. 2382 (The Medical Act of 1959), depriving him of
his legitimate right to practice his profession in the Philippines to his great damage
and prejudice.[11]

On October 19, 2003, the RTC rendered its Decision finding that respondent had
adequately proved that the medical laws of Japan allow foreigners like Filipinos to
be granted license and be admitted into the practice of medicine under the principle
of reciprocity; and that the Board had a ministerial duty of issuing the Certificate
of Registration and license to respondent, as it was shown that he had substantially
complied with the requirements under the law.[12] The RTC then ordered the Board
to issue in favor of respondent the corresponding Certificate of Registration and/or
license to practice medicine in the Philippines.[13]
The Board and the PRC (petitioners) appealed the case to the CA, stating that
while respondent submitted documents showing that foreigners are allowed to
practice medicine in Japan, it was not shown that the conditions for the practice of
medicine there are practical and attainable by a foreign applicant, hence,
reciprocity was not established; also, the power of the PRC and the Board to
regulate and control the practice of medicine is discretionary and not ministerial,
hence, not compellable by a writ of mandamus.[14]

The CA denied the appeal and affirmed the ruling of the RTC.[15]

Hence, herein petition raising the following issue:

WHETHER THE COURT OF APPEALS COMMITTED A REVERSIBLE


ERROR IN FINDING THAT RESPONDENT HAD ESTABLISHED THE
EXISTENCE OF RECIPROCITY IN THE PRACTICE OF MEDICINE
BETWEEN THE PHILIPPINES AND JAPAN.[16]
Petitioners claim that: respondent has not established by competent and conclusive
evidence that reciprocity in the practice of medicine exists between
the Philippines and Japan. While documents state that foreigners are allowed to
practice medicine in Japan, they do not similarly show that the conditions for the
practice of medicine in said country are practical and attainable by a foreign
applicant. There is no reciprocity in this case, as the requirements to practice
medicine in Japan are practically impossible for a Filipino to comply with. There
are also ambiguities in the Medical Practitioners Law of Japan, which were not
clarified by respondent, i.e., what are the provisions of the School Educations
Laws, what are the criteria of the Minister of Health and Welfare of Japan in
determining whether the academic and technical capability of foreign medical
graduates are the same or better than graduates of medical schools in Japan, and
who can actually qualify to take the preparatory test for the National Medical
Examination. Consul General Yabes also stated that there had not been a single
Filipino who was issued a license to practice medicine by the Japanese
Government. The publication showing that there were foreigners practicing
medicine in Japan, which respondent presented before the Court, also did not
specifically show that Filipinos were among those listed as practicing said
profession.[17]Furthermore, under Professional Regulation Commission v. De
Guzman,[18] the power of the PRC and the Board to regulate and control the
practice of medicine includes the power to regulate admission to the ranks of those
authorized to practice medicine, which power is discretionary and not ministerial,
hence, not compellable by a writ of mandamus.[19]
Petitioners pray that the CA Decision dated November 16, 2004 be reversed and
set aside, that a new one be rendered reinstating the Board Order dated March 8,
1993 which disallows respondent to practice medicine in the Philippines, and that
respondent's petition before the trial court be dismissed for lack of merit. [20]

In his Comment, respondent argues that: Articles 2 and 11 of the Medical


Practitioners Law of Japan and Section 9 of the Philippine Medical Act of 1959
show that reciprocity exists between the Philippines and Japan concerning the
practice of medicine. Said laws clearly state that both countries allow foreigners to
practice medicine in their respective jurisdictions as long as the applicant meets the
educational requirements, training or residency in hospitals and pass the licensure
examination given by either country. Consul General Yabes in his letter
dated January 28, 1992 stated that the Japanese Government allows a foreigner to
practice medicine in Japan after complying with the local requirements. The fact
that there is no reported Filipino who has successfully penetrated the medical
practice in Japan does not mean that there is no reciprocity between the two
countries, since it does not follow that no Filipino will ever be granted a medical
license by the Japanese Government. It is not the essence of reciprocity that before
a citizen of one of the contracting countries can demand its application, it is
necessary that the interested citizens country has previously granted the same
privilege to the citizens of the other contracting country.[21] Respondent further
argues that Section 20 of the Medical Act of 1959[22] indicates the mandatory
character of the statute and an imperative obligation on the part of the Board
inconsistent with the idea of discretion. Thus, a foreigner, just like a Filipino
citizen, who successfully passes the examination and has all the qualifications and
none of the disqualifications, is entitled as a matter of right to the issuance of a
certificate of registration or a physicians license, which right is enforceable
by mandamus.[23]

Petitioners filed a Reply[24] and both parties filed their respective


memoranda[25] reiterating their arguments.

The Court denies the petition for lack of merit.

There is no question that a license to practice medicine is a privilege or


franchise granted by the government.[26] It is a right that is earned through years of
education and training, and which requires that one must first secure a license from
the state through professional board examinations.[27]
Indeed,
[T]he regulation of the practice of medicine in all its branches has long been
recognized as a reasonable method of protecting the health and safety of the
public. That the power to regulate and control the practice of medicine includes
the power to regulate admission to the ranks of those authorized to practice
medicine, is also well recognized. Thus, legislation and administrative regulations
requiring those who wish to practice medicine first to take and pass medical
board examinations have long ago been recognized as valid exercises of
governmental power. Similarly, the establishment of minimum medical
educational requirements i.e., the completion of prescribed courses in a
recognized medical school for admission to the medical profession, has also been
sustained as a legitimate exercise of the regulatory authority of the state.[28]

It must be stressed however that the power to regulate the exercise of a


profession or pursuit of an occupation cannot be exercised by the State or its
agents in an arbitrary, despotic, or oppressive manner. A political body which
regulates the exercise of a particular privilege has the authority to both forbid and
grant such privilege in accordance with certain conditions. As the legislature
cannot validly bestow an arbitrary power to grant or refuse a license on a public
agency or officer, courts will generally strike down license legislation that vests in
public officials discretion to grant or refuse a license to carry on some ordinarily
lawful business, profession, or activity without prescribing definite rules and
conditions for the guidance of said officials in the exercise of their power.[29]

R.A. No. 2382 otherwise known as the Medical Act of 1959 states in Section 9
thereof that:

Section 9. Candidates for Board Examinations.- Candidates for Board


examinations shall have the following qualifications:

1. He shall be a citizen of the Philippines or a citizen of any foreign country who


has submitted competent and conclusive documentary evidence, confirmed by the
Department of Foreign Affairs, showing that his countrys existing laws permit
citizens of the Philippines to practice medicine under the same rules and
regulations governing citizens thereof;

xxxx

Presidential Decree (P.D.) No. 223[30] also provides in Section (j) thereof
that:
j) The [Professional Regulation] Commission may, upon the
recommendation of the Board concerned, approve the registration of and
authorize the issuance of a certificate of registration with or without examination
to a foreigner who is registered under the laws of his country: Provided, That the
requirement for the registration or licensing in said foreign state or country are
substantially the same as those required and contemplated by the laws of the
Philippines and that the laws of such foreign state or country allow the citizens of
the Philippines to practice the profession on the same basis and grant the same
privileges as the subject or citizens of such foreign state or country: Provided,
finally, That the applicant shall submit competent and conclusive documentary
evidence, confirmed by the Department of Foreign Affairs, showing that his
country's existing laws permit citizens of the Philippines to practice the profession
under the rules and regulations governing citizens thereof. The Commission is
also hereby authorized to prescribe additional requirements or grant certain
privileges to foreigners seeking registration in the Philippines if the same
privileges are granted to or some additional requirements are required of citizens
of the Philippines in acquiring the same certificates in his country;

xxxx

As required by the said laws, respondent submitted a copy of the Medical


Practitioners Law of Japan, duly authenticated by the Consul General of the
Embassy of the Philippines in Japan, which provides in Articles 2 and 11, thus:

Article 2. Anyone who wants to be medical practitioner must pass the


national examination for medical practitioner and get license from the Minister of
Health and Welfare.

xxxx

Article 11. No one can take the National Medical Examination except
persons who conform to one of the following items:

1. Persons who finished regular medical courses at a university based on


the School Education Laws (December 26, 1947) and graduated from
said university.

2. Persons who passed the preparatory test for the National Medical
Examination and practiced clinics and public sanitation more than one
year after passing the said test.

3. Persons who graduated from a foreign medical school or acquired


medical practitioner license in a foreign country, and also are
recognized to have the same or more academic ability and techniques
as persons stated in item 1 and item 2 of this article.[31]
Petitioners argue that while the Medical Practitioners Law of Japan allows
foreigners to practice medicine therein, said document does not show that
conditions for the practice of medicine in said country
are practical and attainable by a foreign applicant; and since the requirements
are practically impossible for a Filipino to comply with, there is no reciprocity
between the two countries, hence, respondent may not be granted license to
practice medicine in the Philippines.

The Court does not agree.

R.A. No. 2382, which provides who may be candidates for the medical
board examinations, merely requires a foreign citizen to submit competent and
conclusive documentary evidence, confirmed by the Department of Foreign
Affairs (DFA), showing that his countrys existing laws permit citizens of the
Philippines to practice medicine under the same rules and regulations governing
citizens thereof.

Section (j) of P.D. No. 223 also defines the extent of PRC's power to grant
licenses, i.e., it may, upon recommendation of the board, approve the registration
and authorize the issuance of a certificate of registration with or without
examination to a foreigner who is registered under the laws of his country,
provided the following conditions are met: (1) that the requirement for the
registration or licensing in said foreign state or country are substantially the same
as those required and contemplated by the laws of the Philippines; (2) that the laws
of such foreign state or country allow the citizens of the Philippines to practice the
profession on the same basis and grant the same privileges as the subject or
citizens of such foreign state or country; and (3) that the applicant shall submit
competent and conclusive documentary evidence, confirmed by the DFA, showing
that his country's existing laws permit citizens of the Philippines to practice the
profession under the rules and regulations governing citizens thereof.

The said provision further states that the PRC is authorized to prescribe
additional requirements or grant certain privileges to foreigners seeking
registration in the Philippines if the same privileges are granted to or some
additional requirements are required of citizens of the Philippines in acquiring the
same certificates in his country.
Nowhere in said statutes is it stated that the foreign applicant must show that
the conditions for the practice of medicine in said country
are practical and attainable by Filipinos. Neither is it stated that it must first be
proven that a Filipino has been granted license and allowed to practice his
profession in said country before a foreign applicant may be given license to
practice in the Philippines. Indeed, the phrase used in both R.A. No. 2382 and P.D.
No. 223 is that:
[T]he applicant shall submit] competent and conclusive documentary evidence,
confirmed by the Department of Foreign Affairs, showing that his country's
existing laws permit citizens of the Philippines to practice the profession [of
medicine] under the [same] rules and regulations governing citizens thereof.
x x x (Emphasis supplied)

It is enough that the laws in the foreign country permit a Filipino to get
license and practice therein. Requiring respondent to prove first that a Filipino has
already been granted license and is actually practicing therein unduly expands the
requirements provided for under R.A. No. 2382 and P.D. No. 223.

While it is true that respondent failed to give details as to the conditions


stated in the Medical Practitioners Law of Japan -- i.e., the provisions of the
School Educations Laws, the criteria of the Minister of Health and Welfare of
Japan in determining whether the academic and technical capability of foreign
medical graduates are the same as or better than that of graduates of medical
schools in Japan, and who can actually qualify to take the preparatory test for the
National Medical Examination respondent, however, presented proof that
foreigners are actually practicing in Japan and that Filipinos are not precluded
from getting a license to practice there.

Respondent presented before the trial court a Japanese Government


publication, Physician-Dentist-Pharmaceutist Survey, showing that there are a
number of foreign physicians practicing medicine in Japan.[32] He also presented a
letter dated January 28, 1992 from Consul General Yabes,[33] which states:

Sir:

With reference to your letter dated 12 January 1993, concerning your


request for a Certificate of Confirmation for the purpose of establishing a
reciprocity with Japan in the practice of medical profession relative to the case of
Mr. Yasuyuki Ota, a Japanese national, the Embassy wishes to inform you that
inquiries from the Japanese Ministry of Foreign Affairs, Ministry of Health and
Welfare as well as Bureau of Immigration yielded the following information:

1. They are not aware of a Filipino physician who was granted a license by
the Japanese Government to practice medicine in Japan;
2. However, the Japanese Government allows a foreigner to practice
medicine in Japan after complying with the local requirements such as
holding a valid visa for the purpose of taking the medical board exam,
checking the applicant's qualifications to take the examination, taking
the national board examination in Japanese and filing an application
for the issuance of the medical license.

Accordingly, the Embassy is not aware of a single Filipino physician


who was issued by the Japanese Government a license to practice medicine,
because it is extremely difficult to pass the medical board examination in the
Japanese language. Filipino doctors here are only allowed to work in Japanese
hospitals as trainees under the supervision of a Japanese doctor.On certain
occasions, they are allowed to show their medical skills during seminars for
demonstration purposes only. (Emphasis supplied)

Very truly yours,

Jesus I. Yabes
Minister Counsellor &
Consul General

From said letter, one can see that the Japanese Government allows
foreigners to practice medicine therein provided that the local requirements are
complied with, and that it is not the impossibility or the prohibition against
Filipinos that would account for the absence of Filipino physicians holding
licenses and practicing medicine in Japan, but the difficulty of passing the board
examination in the Japanese language. Granting that there is still no Filipino who
has been given license to practice medicine in Japan, it does not mean that no
Filipino will ever be able to be given one.

Petitioners next argue that as held in De Guzman, its power to issue licenses
is discretionary, hence, not compellable by mandamus.

The Court finds that the factual circumstances of De Guzman are different
from those of the case at bar; hence, the principle applied therein should be viewed
differently in this case. In De Guzman, there were doubts about the integrity and
validity of the test results of the examinees from a particular school which
garnered unusually high scores in the two most difficult subjects. Said doubts
called for serious inquiry concerning the applicants satisfactory compliance with
the Board requirements.[34] And as there was no definite showing that the
requirements and conditions to be granted license to practice medicine had been
satisfactorily met, the Court held that the writ of mandamus may not be granted to
secure said privilege without thwarting the legislative will.[35]

Indeed, to be granted the privilege to practice medicine, the applicant must


show that he possesses all the qualifications and none of the disqualifications. It
must also appear that he has fully complied with all the conditions and
requirements imposed by the law and the licensing authority.[36]

In De Guzman itself, the Court explained that:


A careful reading of Section 20[37] of the Medical Act of 1959 discloses
that the law uses the word shall with respect to the issuance of certificates of
registration. Thus, the petitioners [PRC] shall sign and issue certificates of
registration to those who have satisfactorily complied with the requirements of the
Board. In statutory construction the term shall is a word of command. It is given
imperative meaning. Thus, when an examinee satisfies the requirements for the
grant of his physician's license, the Board is obliged to administer to him his oath
and register him as a physician, pursuant to Section 20 and par. (1) of Section 22
of the Medical Act of 1959.[38]

In this case, there is no doubt as to the competence and qualifications of


respondent. He finished his medical degree from Bicol Christian College of
Medicine. He completed a one-year post graduate internship training at
the Jose Reyes Memorial Medical Center, a government hospital. Then he passed
the Medical Board Examinations which was given on August 8, 1992 with a
general average of 81.83, with scores higher than 80 in 9 of the 12 subjects.

In fine, the only matter being questioned by petitioners is the alleged failure
of respondent to prove that there is reciprocity between the laws of Japan and
the Philippines in admitting foreigners into the practice of medicine. Respondent
has satisfactorily complied with the said requirement and the CA has not
committed any reversible error in rendering its Decision dated November 16,
2004 and Resolution dated October 19, 2003.

WHEREFORE, the petition is hereby DENIED for lack of merit.


SO ORDERED.
G.R. No. L-32049 June 25, 1984

MATAAS NA LUPA TENANTS ASSOCIATION, INC., NICOLAS AGLABAY, and Those


Mentioned in Annex "A" of Complaint, petitioners,
vs.
CARLOS DIMAYUGA and JULIANA DIEZ Vda. de GABRIEL, respondents.

Ramon Gonzales for petitioners.

The Solicitor General and Magno T. Bueses for respondents.

MAKASIAR, J.:

This petition for review on certiorari presents for review the order dated October 30, 1969 of the
defunct Court of First Instance of Manila, Branch IV, which granted the motion to dismiss the
complaint of petitioners in Civil Case No. 75391 on the ground that the same failed to state a cause
of action (p. 16, rec.; pp. 1, 100, CFI rec.).

The undisputed facts are as follows:

On January 17, 1969, petitioners filed a complaint for the exercise of preferential rights with the then
Court of First Instance of Manila, Branch IV, docketed as Civil Case No. 75391 (p. 32, rec.; p. 1, CFI
rec.).

The said complaint alleged that petitioner association has for its members Nicolas Aglabay, et al.,
named and listed in Annex "A" of said complaint, which members are heads of 110 tenant families,
and who have been, for more than ten years prior to 1959, occupants of a parcel of land (with their
110 houses built thereon), formerly owned by the respondent, Juliana Diez Vda. de Gabriel, to whom
petitioners have been paying rents for the lease thereof, but who, on May 14, 1968, without notice to
petitioners, sold the same to respondent Carlos Dimayuga, who, in turn, mortgaged the same to her
for the balance of the purchase price; that according to Republic Act 1162, as amended by Republic
Act 2342, a parcel of land in Manila and suburbs, with at least fifty (50) houses of tenants erected
thereon and actually leased to said tenants for at least ten (10) years prior to June 20, 1959, may not
be sold by the landowner to any person other than such tenants, unless the latter renounced their
rights in a public instrument; that without said tenants-appellants having renounced their preferential
rights in public instrument, respondent Vda. de Gabriel sold the land to respondent Dimayuga; that
petitioners-tenants are willing to purchase said land at the same price and on the same terms and
conditions observed in the contract of sale with respondent Dimayuga; and that since aforesaid
contract of sale is expressly prohibited by law, the same is null and void, while it is mandatory for
respondent Vda. de Gabriel to execute such sale to petitioners, Petitioners therefore prayed that
said contract of sale be declared void, and that respondent Vda. de Gabriel be ordered to execute a
deed of sale in favor of petitioners at the same price and conditions followed in the contract with
respondent Dimayuga, plus attorney's fees and damages (p. 32, rec.; p. 1, CFI rec.).

On January 31, 1969, respondent Vda. de Gabriel filed a motion to dismiss on the ground that the
complaint stated no cause of action because the land subject of the complaint is not a landed estate,
and not being such, the same cannot be expropriated, and not being expropriable, no preferential
rights could be availed of by the tenants (p. 41, rec.; p. 22, CFI rec.).
Respondent Dimayuga filed his answer to aforesaid complaint on February 6, 1969 admitting therein
certain factual allegations, denied some averments, interposed the affirmative defenses that plaintiffs
had no personality to initiate the action since the Land Tenure Administration possessed the power
to institute the proper expropriation proceedings before the competent court and that the subject
complaint stated no cause of action against respondent, alleged a counterclaim to eject plaintiffs
from the property, and prayed for the dismissal of the complaint and other remedies (p. 44, rec.; p.
155, CFI rec.).lwphl @itç

On February 6, 1969, plaintiffs-petitioners filed their opposition to the motion to dismiss, maintaining,
among others, that Republic Act 1162, as amended by Republic Act 2342 (law which respondent
Vda. de Gabriel invoked), does not necessarily refer to landed estates, but to any piece of land
occupied by more than 50 families leasing the same for more than ten (10) years prior to June 20,
1959; that their preferential rights are independent of the expropriability of the land; that therefore,
said rights may be exercised even if the land is not expropriable; and that these rights were granted
pursuant to the police power of the State for the general welfare, with prayer that aforesaid motion to
dismiss be denied (p. 47, rec.; p. 26, CFI rec.).

On February 13, 1969, respondent Vda. de Gabriel replied to the aforesaid opposition to motion to
dismiss, reiterating therein her prayer to dismiss the complaint (p. 57, rec.; p. 38, CFI rec.).

Plaintiffs-petitioners filed their rejoinder to above reply to their opposition on February 19, 1969,
laying emphasis on the alleged distinction between the two ways of acquiring occupied land under
Republic Act 1162, which are expropriation and voluntary disposal of the land by the owner thereof,
and which are exercisable independently of each other (p. 56, rec.; p. 42, CFI rec.).

On October 30, 1969, Branch IV of the Court of First Instance of Manila issued the subject order
which found respondent's motion to dismiss well-taken and thereby dismissed the complaint (p. 69,
rec.; p. 100, CFI rec.).

Petitioners moved for reconsideration of the aforecited order on January 7, 1970, which motion was
denied in the lower court's order of January 27, 1970 (p. 111, 190, CFI rec.).

On February 9, 1970, petitioners filed a notice of appeal with the lower court to which respondent
Vda. de Gabriel moved for dismissal of the same on February 11, 1970 on the alleged ground that
pursuant to Republic Act 5440, petitioners should have appealed from the questioned order by way
of a petition for certiorari to this Court since the matter involved only errors or questions of law (p.
143, CFI rec.).

After a series of motions, reply, rejoinder, sur-rejoinder, and answer between both parties, the lower
court issued its order of May 11, 1970 dismissing petitioners' appeal (p. 225, CFI rec.).

Petitioners thus resorted to this petition.

Petitioners contend that the lower court committed an error in dismissing their complaint on the
ground that since the land is not expropriable, it follows that the tenants therein have no preferential
rights to buy said land, if the same is sold voluntarily. Petitioners' contention is anchored on the
amendment introduced by Republic Act 3516 into Section 1 of Republic Act 1162, which latter law
had been invoked in the decision of the lower court.

According to petitioners, the phrase "any landed estates or haciendas herein authorized to be
expropriated" had been amended to read "any landed estates or haciendas or lands herein
authorized to be expropriated"; hence, Republic Act 1162 does not refer exclusively to landed
estates or haciendas, but even to smaller lands. The particular section as amended reads thus:

The expropriation of landed estates or haciendas, or lands which formerly formed


part thereof, or any piece of land in the City of Manila, Quezon City and suburbs,
which have been and are actually being leased to tenants for at least ten years, is
hereby authorized: Provided, That such lands shall have at least forty families of
tenants thereon. (Sec. 1 of R. A. 3516).

Petitioners likewise invoke the amended title of Republic Act 1162 which had been introduced by
Republic Act 2342 which title now reads as follows:

An Act Providing for the Expropriation of Landed Estates or Haciendas or Lands


Which Formerly Formed Part Thereof or Any Piece of Land in the City of Manila,
Quezon City and Suburbs, Their Subdivision into Small Lots, and the Sale of Such
Lots at Cost or Their Lease on Reasonable Terms, and for Other Purposes
(emphasis supplied).

Petitioners further allege that Republic Act 1162 is both an exercise of the power of eminent domain
and the police power of the State. The exercise of the police power of the State refers to the grant of
preferential rights to the tenants of such land, if the same is disposed of voluntarily. Simply stated,
petitioners theorize that Republic Act 1162 covers both compulsory and voluntary sale; hence, while
expropriability is pertinent to compulsory sale, the same does not relate to voluntary sale. Even if the
land is not expropriable, if the same is however actually leased to the occupants for more than ten
years prior to May 22, 1963 (when R.A. 3516 took effect) with at least 40 families, said land, if sold
voluntarily, is subject to the preferential rights of the tenants.

Respondent Vda. de Gabriel maintains, on the other hand, that there is no more issue regarding the
non-expropriability of subject land, which condition or status was expressly admitted by petitioners in
the lower court; that the title of Republic Act 1162, as amended by Republic Act Nos. 2342 and 3516
clearly embraces expropriation; that the prohibitive acts enumerated in Section 5 of R.A. 1162, as
amended, are entirely dependent on the expropriability of the land in controversy; that there is
nothing in the aforecited law which validly supports the alleged preferential right of petitioners to
purchase the property at the same price and under the same conditions; that the only reasonable
interpretation of the opening lines of Section 5 of Republic Act 1162, as amended, is that pending
expropriation, the landowner shall not sell the land to any other person than the tenant or occupant
unless the latter renounces his rights in a public instrument; but if the land is not expropriable, as
petitioners have admitted, the prohibition does not apply; and that clearly, from the provision of
Section 6 of the amended law, Section 5 thereof may be violated only if the land is "herein
authorized to be expropriated" and since petitioners have admitted the non-expropriability of subject
land, it necessarily follows that said Section 5 cannot apply.

Respondent Dimayuga avers that Section 9, in relation to the title of R.A. 1162, clearly provides that
the preferential right could be exercised only when the land under question is subject to
expropriation, or better still, if the tenanted property which formerly formed part of an hacienda or is
a landed estate, had been expropriated; and, that R.A. 1162, as amended, embraces only landed
estates or haciendas with an extensive area.

The sole issue raised by petitioners is whether or not they have the pre-emptive or preferential rights
to buy the land in question.

WE find for petitioners.


I

The third proviso in Section 5 of Republic Act 3516, which law further amended R.A. 1162, reads:

Provided, furthermore, That no lot or portion thereof actually occupied by a tenant or


occupant shall be sold by the landowner to any other person than such tenant or
occupant, unless the latter renounce in a public instrument his rights under this Act:
Provided, finally, That if there shall be tenants who have constructed bona
fide improvements on the lots leased by them, the rights of these tenants should be
recognized in the sale or in the lease of the lots, the limitation as to area in Section
three notwithstanding.

The provision clearly defines the preferential right of herein petitioners to buy the parcel of land. It
should be noted that respondent Vda. de Gabriel voluntarily sold the land to respondent Dimayuga
without informing the petitioners of the transaction. Respondent Vda. de Gabriel did not give the first
offer to petitioners who were then tenants-lessees and who would have either accepted or refused to
buy the land in a public 7 document. The fact is that on discovery of the sale to respondent dent
Dimayuga, petitioners filed their original claim for preferential rights eight months after the
clandestine sale. Thus, the condition set forth in the aforesaid proviso — that of offering first the sale
of the land to petitioners and the latter's renunciation in a public instrument — were not met when
the land was sold to respondent Dimayuga. Evidently, said sale was made illegally and, therefore,
void. Petitioners have still the first option to buy the land as provided for in the above provision.

II

A brief run down of this Court's decisions easily reveal the adherence to the principle that the test for
a valid expropriation of private land for resale to its occupants, is the number of families to be
benefited thereby, and not the area.

In his book on Constitutional Law, Dean Isagani A. Cruz recapitulates thus:

In the earlier case of Rural Progress Administration v. Reyes, the Supreme Court
held that the criterion for determining the validity of expropriation under this provision
was not the area of the land sought to be taken but the number of people intended to
be benefited thereby. The land, in other words, could be small provided it was
tenanted by a sizable number of people.

This ruling was abandoned in the case of Guido v. Rural Progress


Administration where the Supreme Court declared, also by a split decision as in
the Reyes case, that the test to be applied was the area of the land and not the
number of people who stood to be benefitted by the expropriation. The land should
be a landed estate or one comprising a very fast area. It was stressed that one of the
purposes of the framers was precisely to break up these estates in the hands of only
a few individuals or families and thus more equitably distribute them along the
landless.

xxx xxx xxx

It has also been held that where a landed estate is broken up into reasonable
portions which are thereafter sold to separate purchasers, the resultant portions
cannot be deemed as still subject to expropriation under this provision simply
because they used to form part of a landed estate.
In the case of Tuason v. PHHC, which was a petition for prohibition to nullify a law
directing the expropriation of Tatalon Estate in Quezon City, Justice Fernando
suggested a ruling to the Reyes ruling arguing that the propriety of expropriation
"could not be determined on a purely quantitative or area basis," quoting from Justice
J.B.L. Reyes in his dissenting opinion in the Baylosis Case. ... (p 71,1983 Ed.;
emphasis supplied).

From the Reyes case where the number of beneficiaries test was applied in determining public use
down to the Guido and Baylosis cases where the land or area size test was invoked, then to the
Tuason case where a return to the Reyes decision was made and then up to the recent case
of Pulido vs. Court of Appeals (L-57625, May 3, 1983; 122 SCRA 63) where this Court found it
"unfortunate that petitioner would be deprived of his land holdings, but his interest and that of his
family should not stand in the way of progress and the benefit of the greater majority of the
inhabitants of the country," there has evolved a clear pattern of adherence to the "number of people
to be benefited test.

This is made more manifest by the new constitutional provisions on the equitable diffusion of
property ownership and profits (Sec. 6, Art. 11) and the implementation of an agrarian reform
program aimed at emancipating the tenant from the bondage of the soil (Sec. 12, Art. XIV).

It has been noted with concern that while respondents raised the issue of expropriability of the parcel
of land, petitioners limited themselves to the issue of preferential or pre-emptive rights.

What petitioners might have failed to realize is that had they invoked the expropriability of subject
land, they would have had a foolproof case. Right from the start, they would have had the upper
hand. Ironically, however, instead of anchoring their case on the expropriability of such land, they
concentrated on asserting their preferential right to buy the land. For, Section 1 of R.A. 1162, as
amended by R.A. 3516, specifically authorizes the expropriation of any piece of land in the City of
Manila, Quezon City and suburbs which have been and are actually being leased to tenants for at
least ten (10) years, provided said lands have at least forty families of tenants thereon. The case at
bar comes within the coverage of the aforesaid legal provision since the parcel of land is located in
Manila which was then actually leased to 110 tenant families 20 years prior to the commencement of
this action in the lower court. Clearly, therefore, the land in question is capable of expropriation.

The above situation now brings Us back to the case of J.M. Tuason & Co. vs. Land Tenure
Administration (L-21064, Feb. 18, 1970, 31 SCRA 413-417) where this Court laid down certain basic
doctrines on the power of eminent domain. Thus, this Court, speaking thru then Justice Fernando,
declared:

It does not admit of doubt that the congressional power conferred by the Constitution
is far from limited. It is left to the legislative will to determine what lands may be
expropriated so that they could be subdivided for resale to those in need of them.
Nor can it be doubted either that as to when such authority may be exercised is
purely for Congress to decide. Its discretion on the matter is not to be interfered with.
This is shown by reference to the historical basis of the provision as reflected in the
proceedings of the Constitutional Convention.

Historical discussion while valuable is not necessarily decisive. It is easy to


understand why. The social and economic conditions are not static. They change
with the times. To Identify the text of a written constitution with the circumstances
that inspired its inclusion may render it incapable of being responsive to future
needs. Precisely, it is assumed to be one of the virtues of a written constitution that it
suffices to govern the life of the people not only at the time of its framing but far into
the indefinite future. It is not to be considered as so lacking in flexibility and
suppleness that it may be a bar to measures, novel and unorthodox, as they may
appear to some, but nonetheless imperatively called for.

xxx xxx xxx

The framers of the Constitution were seriously concerned with the grave problems of
inequality of wealth, with its highly divisive tendency, resulting in the generous scope
accorded the police power and eminent domain prerogatives of the state, even if the
exercise thereof would cover terrain of property right previously thought of as beyond
state control, to promote social justice and the general welfare.

As in the case of the more general provision on eminent domain, the power to
expropriate lands under Sec. 4 of Art. XIII of the Constitution requires the payment of
just compensation, the taking to be for the public use, and to meet the exacting
standard of due process and equal protection guaranty of the Constitution.

xxx xxx xxx

The power granted to Congress by the Constitution to "authorize, upon payment of


just compensation, the expropriation of lands to be subdivided into small lots and
conveyed at cost to individuals" is unlimited by any other provision of the
Constitution. Just compensation is in reality a part of the power granted rather than a
limitation thereto, just as just compensation is of the essence in any exercise of the
power of eminent domain as, otherwise it would be plain commandeering.

While the taking must be for public use as a matter of principle, in the judicial
proceeding, the Government need not present evidence of such public use as a fact.
The constitutional provision itself declares the public objective, purpose or use of the
expropriation contemplated, hence, it should follow that as long as a congressional
legislation declares that the condemnation of a particular land is for the specific
purpose stated in the Constitution, it is not for the judiciary to inquire as to whether or
not the taking of such land is for public use. The Constitution itself which is supposed
to be the supreme law on private property rights declares it to be so, and leaves it to
Congress, not to the judiciary, to make the choice of the lands to be taken to attain
the objective the constituent assembly aimed to achieve. The scope and the limit of
the power of the judiciary in this regard is only to determine the existence of enabling
legislation, to see to it that the facts are as contemplated in such enabling act and to
provide the vehicle for compliance with procedural due process in the implementation
of the congressional act.

On the matter of taking for public use, Chief Justice Fernando summarily observed:

The taking to be valid must be for public use. There was a time when it was felt that a
literal meaning should be attached to such a requirement. Whatever project is
undertaken must be for the public to enjoy, as in the case of streets or parks.
Otherwise, expropriation is not allowable. It is not so any more. As long as the
purpose of the taking is public, then the power of eminent domain comes into play.
As just noted, the Constitution in at least two cases, to remove any doubt,
determines what is public use One is the expropriation of lands to be subdivided into
small lots for resale at cost to individuals. The other is the transfer, through the
exercise of this power, of utilities and other private enterprise to the government. It is
accurate to state then that at present whatever may be beneficially employed for the
general welfare satisfies the requirement of public use (The Constitution of the
Philippines, 2nd Ed., 1977, pp. 523-24).

III

This preferential right of petitioners and the power of eminent domain have been further mandated,
strengthened and expanded by recent developments in law and jurisprudence.

It must be recalled that the 1973 Constitution embodies certain original and innovative provisions on
eminent domain. The new Constitution provides thus:

Private property shall not be taken for public use without just compensation" (Sec. 2,
Art. IV).

The Batasang Pambansa may authorize, upon payment of just compensation, the
expropriation of private lands to be subdivided into small lots and conveyed at cost to
deserving citizens (See. 13, Art. XIV).

The State shall promote social justice to ensure the dignity, welfare and security of
affirmatively the people. Toward this end, the State shall regulate the acquisition
Ownership, use, enjoyment and disposition of private property, and equitably diffuse
property ownership and profits (Sec. 6, Art. 11; emphasis supplied).

The State shall establish, maintain, and ensure adequate social services in the field
of education, health, housing, employment, welfare, and social security to guarantee
the enjoyment by the people of a decent standard of living (Sec. 7, Art. 11).

The State shall formulate and implement an agrarian reform program aimed at
emancipating the tenant from the bondage of the soil and achieving the goals
enunciated in this Constitution (Sec. 12, Art. XIV).

The aforequoted Section 6 of Article 11, which is a modified version of the original provision of the
1935 Constitution, "emphasizes the stewardship concept, under which private property is supposed
to be held by the individual only as a trustee for the people in general, who are its real owners. As a
mere steward, the individual must exercise his rights to the proper- 4 ty not for his own exclusive and
selfish benefit but for the good of the entire community or nation" (p. 70, Phil. Political Law, Cruz,
1983 ed.).lwphl @itç

In the case of Almeda vs. Court of Appeals, et al. (L-43800, 78 SCRA 194 [July 29, 1977]), this
Court thus declared:

It is to be noted that under the new Constitution, property ownership is impressed


with social function. Property use must not only be for the benefit of the owner but of
society as well. The State, in the promotion of social justice, may "regulate the
acquisition, ownership, use, enjoyment and disposition of private property, and
equitably diffuse property — ownership and profits." One governmental policy of
recent date projects the emancipation of tenants from the bondage of the soil and the
transfer to them of the ownership of the land they till.
"The Legislature may regulate 'the acquisition, ownership, use, enjoyment and disposition of private
property,' to the end that maximum advantage can be derived from it by the people as a whole.
Thus, it may limit the size of private landholdings, impose higher taxes on agricultural lands that are
not being tilled, or provide for a wider distribution of land among the landless. ... (p. 70, Phil. Political
Law, Cruz, 1983 ed.).

It is obviously in the spirit of Sections 6 and 7 of Article 11 that P.D. No. 1517 on urban land reform
was enacted and the subsequent implementing Proclamation No. 1967 was issued. Significantly
also, the latest amendment to the Constitution on urban land reform and social housing program
which has been proclaimed by the President as having been approved in the recent plebiscite on
January 17, 1984 all the more emphasizes and strengthens the constitutional base for urban land
reform consistent with the provisions on social justice.

Even as we have consistently and explicitly pronounced that the power of eminent domain is a basic
and inherent power of government which does not have to be spelled out by the Constitution, still our
legislators felt such urgent demands for redistribution of land in this country that they had to
incorporate into the 1935 and 1973 Constitutions a specific provision on expropriation of land for
resale. Section 13, Article IV of the 1973 Constitution specially authorizes the expropriation of private
lands for resale.

Thus, as earlier mentioned, P.D. No. 1517 entitled "Proclaiming Urban Land Reform in the
Philippines and Providing for the Implementing Machinery Thereof" was enacted and beer effective
on June 1 1, 1978 and Proclamation No. 1967 was issued on May 14, 1980 as an implementing law.
This decree, which is firmly based on Section 6, Article 11 of the new Constitution, undoubtedly
adopts and crystallizes the greater number of people criterion when it speaks of tenants and
residents in declared urban land reform zones or areas without any mention of the land area covered
by such zones. The focus, therefore, is on people who would stand to benefit and not on the size of
the land involved.

It should now be clarified that Section 22 of the aforecited decree declares thus:

Sec. 22. Repealing Clause. — All laws, decrees, executive orders, rules and
regulations inconsistent herewith are hereby repealed, amended or modified
accordingly.

The decree has, therefore, superseded R.A. Nos. 1162, 2342 and 3516.

The issue of pre-emptive or preferential rights still remains for Our resolution within the purview of
the said decree.

The pertinent provisions of P.D. No. 1517 are as follows:

Sec. 4. Proclamation of Urban Land Reform Zones. — The President shall proclaim
specific parcels of urban and urbanizable lands as Urban Land Reform Zones,
otherwise known as Urban Zones for purposes of this Decree, which may include
Bagong Lipunan Sites, as defined in P.D. 1396 (par. 1 of the section).

xxx xxx xxx

Sec. 6. Land Tenancy in Urban Land Reform A Teas. Within the Urban Zones
legitimate tenants who have resided on the land for ten years or more who have built
their homes on the land and residents who have legally occupied the lands by
contract, continuously for the last ten years shall not be dispossessed of the land and
shall be allowed the right of first refusal to purchase the same within a reasonable
time and at reasonable prices, under terms and conditions to be determined by the
Urban Zone Expropriation and Land Management Committee created by Section 8 of
the Decree.

xxx xxx xxx

Sec. 9. Compulsory Declaration of Sale and Preemptive Rights. Upon the


proclamation by the President of an area as an Urban Land Reform Zone, all
landowners, tenants and residents thereupon are required to declare to the Ministry
any proposal to sell, lease or encumber lands and improvements thereon, including
the proposed price, rent or value of encumbrances and secure approval of said
proposed transactions.

The Ministry shag have the pre-emptive right to acquire the above-mentioned lands
and improvements thereon which shall include, but shag not be limited to lands
occupied by tenants as provided for in Section 6 of this Decree (emphasis supplied).

Pursuant to the above decree and for purposes of making specific the applicability of the same and
other subsequent laws on the matter, the President issued Proclamation No. 1967 dated May 14,
1980 declaring Metropolitan Manila Area as Urban Land Reform Zone. Thus, on page 2, No. 14 of
said proclamation, Mataas na Lupa, the land in controversy, (an area bounded on the northwest by
Quirino Avenue, South Superhighway on the east, San Andres Street on the south, and on the west,
by Anak Bayan Street) was declared as an area for priority development and urban land reform
zone.

The aforequoted provisions of P.D. 1517 and the declaration in the aforesaid proclamation are clear
and leave no room for any interpretation. Evidently, petitioners' case falls squarely within the law.
Under Section 6 of the decree, the 110 tenant-families have been vested with the right of first refusal
to purchase the land in question within a reasonable time and reasonable prices, subject to Ministry
of Human Settlements rules and regulations.

WHEREFORE, THE ORDER DATED OCTOBER 30, 1969 OF THE THEN MANILA COURT OF
FIRST INSTANCE, BRANCH IV, IS HEREBY SET ASIDE AND THE MINISTRY OF HUMAN
SETTLEMENTS IS HEREBY DIRECTED TO FACILITATE AND ADMINISTER THE
IMPLEMENTATION OF THE RIGHTS OF HEREIN PETITIONERS. COSTS AGAINST
RESPONDENTS.
.R. No. 155001. May 5, 2003 En Banc [Non-legislative power of Congress; Police Power;
Delegation of emergency powers]

FACTS:
On October 5, 1994, AEDC submitted an unsolicited proposal to the Government through the
DOTC/MIAA for the development of NAIA International Passenger Terminal III (NAIA IPT
III).

DOTC constituted the Prequalification Bids and Awards Committee (PBAC) for the
implementation of the project and submitted with its endorsement proposal to the NEDA, which
approved the project.

On June 7, 14, and 21, 1996, DOTC/MIAA caused the publication in two daily newspapers of an
invitation for competitive or comparative proposals on AEDC’s unsolicited proposal, in
accordance with Sec. 4-A of RA 6957, as amended.

On September 20, 1996, the consortium composed of People’s Air Cargo and Warehousing Co.,
Inc. (Paircargo), Phil. Air and Grounds Services, Inc. (PAGS) and Security Bank Corp. (Security
Bank) (collectively, Paircargo Consortium) submitted their competitive proposal to the
PBAC. PBAC awarded the project to Paircargo Consortium. Because of that, it was
incorporated into Philippine International Airport Terminals Co., Inc.

AEDC subsequently protested the alleged undue preference given to PIATCO and reiterated its
objections as regards the prequalification of PIATCO.

On July 12, 1997, the Government and PIATCO signed the “Concession Agreement for the
Build-Operate-and-Transfer Arrangement of the NAIA Passenger Terminal III” (1997
Concession Agreement). The Government granted PIATCO the franchise to operate and
maintain the said terminal during the concession period and to collect the fees, rentals and other
charges in accordance with the rates or schedules stipulated in the 1997 Concession
Agreement. The Agreement provided that the concession period shall be for twenty-five (25)
years commencing from the in-service date, and may be renewed at the option of the
Government for a period not exceeding twenty-five (25) years. At the end of the concession
period, PIATCO shall transfer the development facility to MIAA.

Meanwhile, the MIAA which is charged with the maintenance and operation of the NAIA
Terminals I and II, had existing concession contracts with various service providers to offer
international airline airport services, such as in-flight catering, passenger handling, ramp and
ground support, aircraft maintenance and provisions, cargo handling and warehousing, and other
services, to several international airlines at the NAIA.

On September 17, 2002, the workers of the international airline service providers, claiming that
they would lose their job upon the implementation of the questioned agreements, filed a petition
for prohibition. Several employees of MIAA likewise filed a petition assailing the legality of the
various agreements.
During the pendency of the cases, PGMA, on her speech, stated that she will not “honor
(PIATCO) contracts which the Executive Branch’s legal offices have concluded (as) null and
void.”

ISSUE:
Whether or not the State can temporarily take over a business affected with public interest.

RULING:
Yes. PIATCO cannot, by mere contractual stipulation, contravene the Constitutional
provision on temporary government takeover and obligate the government to pay
“reasonable cost for the use of the Terminal and/or Terminal Complex.”

Article XII, Section 17 of the 1987 Constitution provides:


Section 17. In times of national emergency, when the public interest so requires, the State may,
during the emergency and under reasonable terms prescribed by it, temporarily take over or
direct the operation of any privately owned public utility or business affected with public
interest.

The above provision pertains to the right of the State in times of national emergency, and in the
exercise of its police power, to temporarily take over the operation of any business affected with
public interest. The duration of the emergency itself is the determining factor as to how long the
temporary takeover by the government would last. The temporary takeover by the government
extends only to the operation of the business and not to the ownership thereof. As such
the government is not required to compensate the private entity-owner of the said business
as there is no transfer of ownership, whether permanent or temporary. The private entity-
owner affected by the temporary takeover cannot, likewise, claim just compensation for the use
of the said business and its properties as the temporary takeover by the government is in exercise
of its police power and not of its power of eminent domain.

Article XII, section 17 of the 1987 Constitution envisions a situation wherein the exigencies of
the times necessitate the government to “temporarily take over or direct the operation of any
privately owned public utility or business affected with public interest.” It is the welfare and
interest of the public which is the paramount consideration in determining whether or not to
temporarily take over a particular business. Clearly, the State in effecting the temporary takeover
is exercising its police power. Police power is the “most essential, insistent, and illimitable of
powers.” Its exercise therefore must not be unreasonably hampered nor its exercise be a source
of obligation by the government in the absence of damage due to arbitrariness of its exercise.
Thus, requiring the government to pay reasonable compensation for the reasonable use of the
property pursuant to the operation of the business contravenes the Constitution.
A summary of the Agan vs. Piatco case. Warning, though: this is a first year law student’s summary of the case. It

may not be correct. Portions were also directly lifted from the decision.
Title: Agan Jr. vs. Philippine International Air Terminals Co. Inc
Date: – May 5, 2003
Ponente: Puno
Nature of the case: Special Civil Action in the Supreme Court. Prohibition.
Facts: In August 1989 the DOTC had a study conducted to determine whether or not the present Ninoy Aquino
International Airport (NAIA) can cope with traffic development up to the year 2010. A draft final report was submitted
to the DOTC in December 1989.
Four years later, in 1993, six Filipino-Chinese business leaders met with then President Fidel Ramos to explore the
possibility of investing in the construction and operation of a new international airport terminal. The six later formed
the Asia’s Emerging Dragon Corp (AEDC) which submitted an unsolicited proposal for the development of NAIA
International Passenger Terminal III more than a year after the first meeting with Ramos, in October 1994.
In March 1995, the DOTC endorsed the AEDC proposal to the National Economic and Development Authority
(NEDA). In January 1996, NEDA passed Board Resolution No. 2 which approved the NAIA IPT III project.
In June 1996, the DOTC published an invitation for competitive bidding in two daily newspapers, as required by law
(sec 4-A of RA6957). The alternative bidders had to submit three envelopes. The first contains the prequalification
documents, the second the technical proposal, and the third the financial proposal of the proponent.
The bidding was scheduled on September 20, 1996.
The bid documents issued by the Prequalification Bids and Awards Committee said the proponent must have
adequate capability to sustain the financing requirement for the engineering, design, construction, operation, and
maintenance phases of the project. The proponent must have an equity that is at least 30% of the project cost, and
be able to secure external financing for the project.
Government was also guaranteed a five percent share in the gross revenue of the project for the first five years; 7.5%
in the next 10 years, and 10% in the next 10 years. This would be in addition to a fixed annual guaranteed payment to
the government.
The basis for the prequalification shall be the proponent’s compliance with the minimum technical and financial
requirements provided in the bid documents and the IRR of the BOT (build operate and transfer) Law.
The bid documents allowed amendments to the draft concession agreement, but said that these should cover only
items that would not materially affect the preparation of the proponent’s proposal.
In August 1996, during the second pre-bid conference, the PBAC made several clarifications, upon the request of
People’s Air Cargo & Warehousing Co. Inc (PAIRCARGO), which wanted to challenge the AEDC bid.
The PBAC said the list of revenue sources mentioned in the bid documents were “merely indicative.” The project
proponent may add other revenue sources, subject to approval by the DOTC/MIAA. Also, only fees and charges
denominated as “public utility fees” would be subject to regulation, and these could still be revised, because the
PBAC has a pending query with the justice department.
In September 1996, PBAC issued a bid bulletin in which it said that since PAIRCARGO could not meet the required
minimum equity prescribed in the bid documents, it would accept instead an audited financial statement of the
financial capability of all member companies of the consortium.
In September 1996, PAIRCARGO submitted their competitive proposal to the PBAC. The first envelope, containing
the prequalification documents, was opened on September 23, 1996, and PBAC prequalified the PAIRCARGO
consortium the following day.
On September 26, AEDC filed with PBAC its reservations regarding PAIRCARGo, noting the “lack of corporate
approvals and financial capability of PAIRCARGO.” For one, PAIRCARGO included in the computation of its financial
capability the total net worth of Security Bank, when the Banking Law limits to 15% the total investment that a bank
may make on one project. It also questioned the appointment of Lufthansa as facility operator, because Philippine
laws limit to Filipinos the operation of a public utility.
The PBAC, however, said on October 2, 1996 that based on the documents submitted and the prequalification
criteria, PAIRCARGO was prequalified. The DOTC secretary approved PBAC’s findings.
The AEDC reiterated its objections two more times.
On October 16, the third envelope containing the financial proposals were opened, and PAIRCARGO had offered to
pay the government higher.
Both PAIRCARGO and AEDC offered to build the NAIA Passenger Terminal III for at least $350 million at no cost to
the government and to pay the government a 5% share in gross revenues for the first five years of operation; a 7.5%
share in gross revenues for the next 10 years of operation; and a 10% share in gross revenues for the last 10 years
of operation. In addition to this, AEDC offered to pay the government P135 million as guaranteed payment for 27
years. Paircargo Consortium offered a total of P17.75 billion for the same period.
PBAC informed AEDC it had accepted Paircago’s price proposal, and given AEDC 30 working days to match the bid.
When AEDC failed to do so, the DOTC issued a notice on December 11 1996 regarding AEDC’s failure to match the
proposal.
In February 1997, Paircargo Consortium incorporated into Philippine International Airport Terminals Co Inc (PIATCO).
AEDC protested the alleged undue preference given to PIATCO and reiterated its objections regarding the
prequalification of PIATCO. In April 1997, it filed before the Pasig RTC a petition for declaration of nullity of the
proceedings, mandamus and injunction against the DOTC secretary, the PBAC chair and its voting members, and the
chair of the PBAC technical committee.
On April 17, the NEDA ICC conducted an ad referendum to facilitate the approval of the BOT agreement between the
DOTC and PIATCO. Because there were only four instead of the required six signatures, the NEDA merely noted the
agreement.
On July 9, 1997, the DOTC issued the notice of award for the project to PIATCO.
The Concession Agreement was signed on July 12, 1997, granting PIATCO the franchise to operate and maintain the
NAIA Passenger Terminal III for 25 years, with an option to renew for a period not exceeding 25 years. PIATCO was
allowed to collect fees, rentals and other charges in accordance with the rates or schedules in the 1997 Concession
Agreement. At the end of the concession period, PIATCO will transfer the airport to MIAA.
In November 1998, the government and PIATCO signed an Amended and Restated Concession Agreement (ARCA).
The ARCA amended provisions on the special obligations of the government, the exclusivity of the franchise given to
PIATCO, the proceeds of the insurance, the taxes, duties and other charges that may be levied PIATCO, and the
provisions on the termination of the contract. Three more supplements to the ARCA were signed afterwards: in
August 1999, in September 2000, and in June 2001.
The first redefined revenues, required government to construct an access road connecting NAIA II and III, and added
to the special obligations of government.
The second supplement required government to clear structures at the construction site and to pay PIATCO for
these. The third provided for PIATCO’s obligations regarding the construction of the surface road connecting
Terminals II and III.
In September 2002, workers of the international airline service providers filed before the Supreme Court a petition for
prohibition enjoining the enforcement of the agreements. They said the transfer to NAIA III could cost them their jobs,
since under the agreements, PIATCO is not required to honor MIAA’s existing concession contracts with various
service providers for international airline airport services.
In October 2002, the service providers filed a motion for intervention and a petition in intervention, joining the cause
of the petitioning workers.
Three congressmen – Salacnib Baterina, Clavel Martinez and Constantino Jaraula – filed a similar petition shortly
after.
In November 2002, several MIAA employees also filed a petition questioning the legality of the agreements.
In December 2002, another group of congressmen – Jacinto Paras, Rafael Nantes, Eduardo Zialcita, Willie Villarama,
Prospero Nograles, Prospero Pichay Jr., Harlin Cast Abayon and Benasing Macarambon – filed their comment in
intervention defending the validity of the agreements and praying for the dismissal of the petitions.
On December 10, 2002, the court heard the case on oral argument, the required the parties to file their respective
memoranda, and to explore the possibility of arbitration as provided in the challenged contracts.
In their consolidated memorandum, the Office of the Solicitor General and the Office of the Government Corporate
Counsel prayed that the petitions be given due course and that the 1997 Concession Agreement, the ARCA and the
supplements be declared void for being contrary to the Constitution, the BOT Law and its implementing rules and
regulations.
In March 2003, PIATCO commenced arbitration proceedings before the International Chamber of Commerce,
International Court of Arbitration.
Issue: Did the PIATCO agreements – the 1997 Concession Agreement, the ARCA, and the three supplemental
agreements – violate the Constitution and the BOT Law?
Held/Decision: YES.
Ratio:
In the first place, PIATCO was not a qualified bidder. The minimum project cost was estimated to be P9.183 billion,
which meant that Paircargo Consortium had to prove it could provide at least P2.755 billion. Paircargo’s audited
financial statement for 1994 showed it had a net worth of P3.123 billion, but that was because it included in the
computation the total net worth of Security Bank, which as of 1995 was at P3.523 billion. Since banks are allowed to
invest only 15% of it entire net worth, Security Bank could only invest P528 million, which brings down Paircargo’s
equity to P558.384 million, or only 6% of the project cost.
“Disregarding the investment ceilings provided by applicable law would not result in a proper evaluation of whether or
not a bidder is pre-qualified to undertake the project as for all intents and purposes, such ceiling or legal restriction
determines the true maximum amount which a bidder may invest in the project…If the maximum amount of equity
that a bidder may invest in the project at the time the bids are submitted falls short of the minimum amounts required
to be put up by the bidder, said bidder should be properly disqualified…we hold that Paircargo Consortium was not a
qualified bidder. Thus the award of the contract by the PBAC to the Paircargo consortium is null and void.”
Other issues:
1. The signed agreement was different from the draft that was bidded on. Under the law, substantial changes require
another bidding. The three principles in public bidding are: an offer to the public; opportunity for competition; and
basis for the exact comparison of bids. Changing the parameters would change the agreement, which might have
changed the technical and financial parameters of other bidders had they known that such terms were available.
The 1997 Concession Agreement signed between PIATCO and the government was substantially different from the
draft concession agreement that was bidded on. While it was a draft and was expected to amended from time to time,
the PBAC bid bulletin also said that the amendments should only cover items that would not materially affect the
preparation of the proposal. The changes should not be substantial or material enough to alter the basic parameters
of the contract, and constitute a denial to the other bidders of the opportunity to bid on the same terms.
There were two main differences between the draft agreement and the one that was signed. These concerned the
fees that may be imposed and collected by PIATCO, and the extent of control and regulation that MIAA has over the
fees that PIATCO will charge.
The draft agreement classified aircraft parking and tacking fees, groundhandling fees, rentals and airlines offices,
check-in counter rentals and porterage fees under those that are regulated – subject to periodic adjustment of once
every two years and in accordance to a certain formula. The signed agreement said fees subject to MIAA approval
are “public utility fees” and took out groundhandling and rentals and airlines offices from the list. There was also “an
obvious relaxation of the extent of control and regulation by MIAA with respect to the particular fees that may be
charged by PIATCO.” The signed agreement also allowed PIATCO to charge in US dollars, while paying the
government in pesos.
“When taken as a whole, the changes under the 1997 Concession Agreement with respect to reduction in the types
of fees that are subject to MIAA regulation and the relaxation of such regulation with respect to other fees are
significant amendments that substantially distinguish the draft Concession Agreement from the 1997 Concession
Agreement. The 1997 Concession Agreement, in this respect, clearly gives PIATCO more favorable terms than what
was available to other bidders at the time the contract was bidded out. It is not very difficult to see that the changes in
the 1997 Concession Agreement translate to direct and concrete financial advantages for PIATCO which were not
available at the time the contract was offered for bidding. It cannot be denied that under the 1997 Concession
Agreement only “Public Utility Revenues” are subject to MIAA regulation. Adjustments of all other fees imposed and
collected by PIATCO are entirely within its control. Moreover, with respect to terminal fees, under the 1997
Concession Agreement, the same is further subject to “Interim Adjustments” not previously stipulated in the draft
Concession Agreement. Finally, the change in the currency stipulated for “Public Utility Revenues” under the 1997
Concession Agreement, except terminal fees, gives PIATCO an added benefit which was not available at the time of
bidding.
2. Under the draft Concession Agreement, default by PIATCO of its obligations does not result in the assumption by
government of these liabilities. Under the signed agreement, default by PIATCO of its loans used to finance the
project eventually leads to government assumption of the liability for the loans.
This is in violation of the BOT Law, which prohibits direct government guarantees. “If a proposal can be denied by
reason of the existence of direct government guarantee, then its inclusion in the contract executed after the said
proposal has been accepted is likewise sufficient to invalidate the contract itself…To declare the PIATCO contracts
valid despite the clear statutory prohibition against a direct government guarantee would not only make a mockery of
what the BOT Law seeks to prevent – which is to expose the government to the risk of incurring a monetary
obligation resulting from a contract of loan between the project proponent and its lenders and to which the
Government is not a party to – but would also render the BOT Law useless for what it seeks to achieve – to make use
of the resources of the private sector in the financing, operation and maintenance of infrastructure and development
projects which are necessary for national growth and development but which the government, unfortunately, could ill-
afford to finance at this point in time.”
3. Sec. 5.10 of the 1997 Concession Agreement violates Article XII, Sec. 12 of the 1987 Constitution.
The Constitutional provision allows for temporary takeover of public facilities in times of national emergency. Since
the takeover is temporary and extends only to the operation of the business and not the ownership, government is not
required to compensate the owner. Neither can the owner claim just compensation for the use of the business and its
properties because the takeover is in exercise of the State’s police power and not of its power of eminent domain.
The 1997 Concession Agreement, on the other hand, says that in the event of a takeover, “Concessionaire shall be
entitled to reasonable compensation for the duration of the temporary takeover…”
“PIATCO cannot, by mere contractual stipulation, contravene the Constitutional provision on temporary government
and obligate the government pay reasonable cost for the use of Terminal and/or Terminal Complex.
Police power is the most essential, insistent, and illimitable of powers. Its exercise must not be unreasonably
hampered nor its exercise be a source of obligation by the government in the absence of damage due to arbitrariness
of its exercise.
4. The 1987 Constitution strictly regulates monopolies. Art XII, Sec. 19 says: The State shall regulate or prohibit
monopolies when the public interest so requires.
The 1997 Concession Agreement gave PIATCO the exclusive right to operate a commercial international passenger
terminal within the island of Luzon, with the exception of already existing terminals such as those in the Subic Bay
Freeport, Clark Special Economic Zone, and in Laoag City. This privilege, however, is subject to reasonable
regulation and supervision and should not violate the rights of third parties.
There are service providers at the NAIA I with existing contracts with the MIAA valid until 2010; since the 1997
Concession Agreement says PIATCO is not bound to honor existing contracts with MIAA, transferring operations
from NAIA I to NAIA III would unduly prejudice them. “PIATCO cannot, by law and certainly not by contract, render a
valid and binding contract nugatory. PIATCO, by the mere expedient claiming an exclusive right to operate, cannot
require the Government to break its contractual obligations to the service providers.”
IN SUM, THIS COURT RULES THAT IN VIEW OF THE ABSENCE OF THE REQUISITE FINANCIAL CAPACITY OF
THE PAIRCARGO CONSORTIUM, PREDECESSOR OF RESPONDENT PIATCO, THE AWARD BY THE PBAC OF
THE CONTRACT FOR THE CONSTRUCTION, OPERATION AND MAINTENANCE OF THE NAIA IPT III IS NULL
AND VOID. FURTHER, CONSIDERING THAT THE 1997 CONCESSION AGREEMENT CONTAINS MATERIAL
AND SUBSTANTIAL AMENDMENTS, WHICH AMENDMENTS HAD THE EFFECT OF CONVERTING THE 1997
CONCESSION AGREEMENT INTO AN ENTIRELY DIFFERENT AGREEMENT FROM THE CONTRACT BIDDED
UPON, THE 1997 CONCESSION AGREEMENT IS SIMILARLY NULL AND VOID FOR BEING CONTRARY TO
PUBLIC POLICY. THE PROVISIONS UNDER SECTIONS 4.04(B) AND (C) IN RELATION TO SECTION 1.06 OF
THE 1997 CONCESSION AGREEMENT AND SECTION 4.04 (C) IN RELATION TO SECTION 1.06 OF THE ARCA,
WHICH CONSTITUTE A DIRECT GOVERNMENT GUARANTEE EXPRESSLY PROHIBITED BY, AMONG
OTHERS, THE BOT LAW AND ITS IMPLEMENTING RULES AND REGULATIONS ARE ALSO NULL AND VOID.
THE SUPPLEMENTS, BEING ACCESSORY CONTRACTS TO THE ARCA, ARE LIKEWISE NULL AND VOID.
JUDGMENT: The 1997 Concession Agreement, the Amended and Restated Concession Agreement and the
Supplements thereto are set aside for being null and void.
***VITUG: separate opinion – court has no jurisdiction. Petition prays for nullification of contract and does not involve
judicial, quasi-judicial or ministerial functions. The parties allege contentious evidentiary facts and “it would be difficult
to decide the jurisdictional issue on the basis of the contradictory factual submissions made by the parties.”
***issues on standing: piatco tried to raise the issue of jurisdiction saying they had filed a case with the international
chamber of commerce, international court of arbitration for arbitration. The court however pointed out that it includes
multiple contracts and not all of the petitioners are parties to the contract.
La Bugal-B’laan Tribal Association, Inc. vs. Ramos

Ponente: Panganiban Date: 1 December 2004 Nature: Special


Civil Action for Prohibition and Mandamus

Parties Involved:

La Bugal-B’laan Tribal Association, Inc., represented by its Victor Ramos, DENR Sec.; Horacio Ramos, MGB-DENR Dir.;
Chairman F’long Miguel M. Lumayong; et. al. (petitioner Ruben Torres, Exec. Sec. WMCP, Inc. (respondent)

Facts and Background of the Case

- 24 January 2007: the Court en banc declared unconstitutional: (a) certain provisions of RA No. 7942 (Mining
Law), (b) its Implementing Rules and Regulations DAO No. 96-40, and (c) the Financial and Technical Assistance
Agreement (FTAA) dated March 30, 1995, executed by the government with Western Mining Corporation
(Philippines), Inc. (WMCP), mainly on the finding that FTAAs are service contracts prohibited by the 1987 Constitution
- Respondents filed separate Motions for Reconsideration

Legal Issues

A. W/N case has been rendered moot by the sale of WMC shares in WMCP to Sagittarius Mines, Inc. (a Filipino
corporation) and by the subsequent transfer and registration of the FTAA from WMCP to Sagittarius.
B. W/N it is still proper to resolve constitutionality of the assailed provisions, assuming that the case has been rendered
moot.
C. the proper interpretation of the phrase: Agreements Involving Either Technical or Financial Assistance, contained in
paragraph 4 of Section 2 of Article XII of the Constitution.
D. thus, W/N provisions are unconstitutional.

Contentions and Resolutions

<petitioners> Court <respondents>


A. The case has not become moot: A. 1- Nowhere in the provision is there
1- the alleged invalidity of the transfer of the any express limitation or restriction
WMCP shares to Sagittarius violates the insofar as arrangements other than the
fourth paragraph of Section 2 of Article XII of three mentioned contractual schemes
the Constitution. Petitioners claim that a are concerned. There is no reason to
Filipino corporation is not allowed by the
believe that the framers of the
Constitution to enter into an FTAA with the
government. Constitution, a majority of whom where
2-it is contrary to the provisions of the WMCP obviously concerned with furthering the
FTA itself, in that the FTAA was intended to development and utilization of the
apply solely to a foreign corporation, as can country’s natural resources, could have
allegedly be seen from the provisions therein. wanted to restrict Filipino participation in
[Section 12, which provides for international that area.
commercial arbitration under the auspices of 2- This provision does not necessarily
the International Chamber of Commerce, imply that the WMCP FTAA cannot be
after local remedies are exhausted—clearly
transferred to and assumed by a Filipino
intended to apply only to a foreign contractor]
3-the sale of the shares is suspect and corporation like Sagittarius, in which
should therefore be the subject of a case in event the said provision should simply be
which it validity may be properly litigated. disregarded as a superfluity.
Section 40 of RA 7942 allegedly requires the 3- Section 40 expressly applies to the
President’s prior approval of a transfer. assignment or transfer of the FTAA, not
to the sale and transfer of shares of
stock in WMCP. When the transfer of the
FTAA happens to be a Filipino
Petitioners also insist that FTAA is corporation, the need for such safeguard
void, and hence, cannot be is not critical.
transferred. Its transfer does not
operate to cure the constitutional infirmity
that is inherent in it; neither will a change Petitioners have assumed as fact that
in the circumstances of one of the parties which has yet to be established. The
serve to ratify the void contract. Decision of the Court declaring the FTAA
void has not yet become final. The FTAA
is not per se defective or
unconstitutional. It was questioned only
because it had been issued to an
allegedly non-qualified, foreign-owned
corporation.
B. 1- What is at issue is not only the B. The court concedes that there exists B. The courts will decide a question—
validity of the WMCP-FTAA, but also the the distinct possibility that one or more of otherwise moot and academic—if it is
constitutionality of RA 7942 and its the future FTAAs will be the subject of “capable of repetition yet evading
Implementing Rules and Regulations yet another suit grounded on review.”
2- the acts of private respondent cannot constitutional issues. [as of June 2002,
operate to cure the law of its alleged some 43 FTAAs had been filed]
unconstitutionality or to divest this Court The Court must recognize the
of its jurisdiction to decide. exceptional character of the situation and
3- the Constitution imposes upon the the paramount public interest involved,
Supreme Court the duty to declare as well as the necessity for a ruling to
invalid any law that offends the put an end to the uncertainties
Constitution. plaguing the mining industry and the
affected communities as a result of
doubts cast upon the constitutionality
and validity of the Mining Act, the
subject FTAA and future FTAAs, and
the need to avert a multiplicity of
suits.
The real issue in this case is whether
paragraph 4 of Section 2 of Article XII of
the Constitution is contravened by RA
7942 and DAO 96-40. By the mere
enactment of the questioned law or
the approval of the challenged action,
the dispute is said to have ripened
into a judicial controversy even
without any other overt act.
C. Petitioners claim that the phrase C. The use of the word “involving” C. The FTAA allowed in the Constitution
”agreements x x x involving either signifies the possibility of the is a comprehensive agreement for the
technical or financial assistance” simply inclusion of other forms of assistance foreign-owned corporation’s integrated
means technical assistance or financial or activities having to do with, otherwise exploration, development and utilization
assistance agreements, nothing more related to or compatible with financial or of mineral, petroleum or other mineral
and nothing else, and such a limitation, technical assistance. These agreements oils on a large-scale basis.
they argue, excludes foreign with foreign corporations are not limited
management and operation of a to mere financial or technical assistance.
mining enterprise. Otherwise, the language of the drafters
Court’s previous decision was correct, in would have certainly been so
that the FTAA allowed the foreign unmistakably restrictive and stringent as
contractor to have direct and exclusive to leave no doubt in anyone’s mind about
management of a mining enterprise, their true intent. There was a conscious
and vests in the foreign company and deliberate decision to avoid the
beneficial ownership of our mineral use of restrictive wording that
resources. bespeaks an intent not to use the
expression “agreements x x x
involving either technical or financial
assistance” in an exclusionary and
limiting manner.
The deletion or omission from the 1987
Constitution of the term “service Such intent cannot be definitively and
contracts” found in the 1973 Constitution conclusively established from the mere
sufficiently proves the drafters’ intent to failure to carry the same expression or
exclude foreigners from the management term over to the new Constitution, absent
of the affected enterprises. amore specific, explicit and unequivocal
statement to that effect. Pertinent
portions of the deliberations of the
members of the ConCom conclusively
show that they discussed agreements
involving either technical or financial
assistance in the same breadth as
service contracts and used the terms
interchangeably.
D. RA 7942, as well as its Implementing D. The gamut of requirements,
Rules and Regulations, makes it regulations, restrictions and limitations
possible for FTAA contracts to cede imposed upon the FTAA contractor by
full control and management of the statute and regulations easily
mining enterprises over to fully overturns petitioners’ contention. The
foreign-owned corporations, with the FTAA contractor is not free to do
result that the State is allegedly whatever it pleases and get away with
reduced to a passive regulator it; on the contrary, it will have to
dependent on submitted plans and follow the government line if it wants
reports, with weak review and audit to stay in the enterprise. Ineluctably
powers. The law, the implementing then, RA 7942 and DAO 96-40 vest in
regulations, and the WMCP FTAA cede the government more than a sufficient
“beneficial ownership” of the mineral degree of control and supervision
resources to the foreign contractor. over the conduct of mining
operations.
Section 3 of RA 7942—which allows a
foreign contractor to apply for and hold The exploration permit serves a practical
an exploration permit—is and legitimate purpose in that it protects
unconstitutional. the interests and preserves the rights of
the exploration permit grantee (the
would-be contractor)—foreign or local—
during the period of time that it is
spending heavily on exploration works,
without yet being able to earn revenues
to recoup any of its investments and
expenditures.

The evaluation and analysis of the


FTAA provisions sufficiently overturn
petitioners’ litany of objections to and
criticisms of the State’s alleged lack
of control. The provisions vest the State
with control and supervision over
RA 7942 allegedly limits the State’s practically all aspects of the operations
share in FTAAs with foreign contractors of the FTAA contractor, including the
to just taxes, fees and duties, and charging of pre-operating and operating
depriving the State of a share in the expenses, and the disposition of mineral
after-tax income of the enterprise. products.

The Court does not share the view


that in FTAAs with foreign contractors
under RA 7942, the government’s
share is limited to taxes, fees and
duties. The inclusion of the phrase
“among other things” in the second
paragraph of Section 81 clearly and
unmistakable reveals the legislative
intent to have the State collect more than
just the usual taxes, duties and fees.

BUT, the WMCP FTAA has invalid


provisions:
- Section 7.9 of the WMCP FTAA clearly
renders illusory the State’s 60 percent
share of WMCP’s revenues in providing
that: should WMCP’s foreign
stockholders sell 60 percent or more of
their equity to a Filipino citizen or
corporation, the State loses its right to
receive its share in net mining revenues
under Section 7.7, with any offsetting
compensation to the State.
- Section 7.8 is likewise invalid, since by
allowing the sums spent by government
for the benefit of the contractor to be
deductible from the State’s share in net
mining revenues, it results in benefiting
the contractor twice over. This constitute
unjust enrichment on the part of the
contractor, at the expense of the
government

Judgment

A. Because of the validity of sale and the transfer, and that the FTAA is not void per se, the case has become moot.
However,
B. there is a need to resolve the unconstitutionality of the assailed provisions.
C. The phrase “agreements involving either technical and financial assistance,” as used interchangeably with the term
“service contracts” by the drafters of the Constitution and as interpreted by the court, is not exclusionary and limiting.
D. The assailed provisions are not unconstitutional.

Important Notes
- The FTAA is for the exploration, development and commercial exploitation of mineral deposits in South Cotabato,
Sultan Kudarat, Davao del Sur and North Cotabato, covering an area of 99,837 ha. WMC is wholly owned subsidiary of
Western Mining Corporation Holdings Limited, a publicly listed major Australian mining and exploration company.
- A service contract has been defined as a contractual agreement for engaging in the exploitation and development of
petroleum, mineral, energy, land and other natural resources, whereby a government or an agency thereof, or a private
person granted a right or privilege by said government, authorizes the other party—the service contractor—to engage
or participate in the exercise of such right or the enjoyment of the privilege, by providing financial or technical
resources, undertaking the exploitation or production of a given resource, or directly managing the productive
enterprise, operations of the exploration and exploitation of the resources, or the disposition or marketing of said
resources.

Personal Notes

As to the case being a political question, the EPILOGUE provides:

“The crux of the controversy is the amount of discretion to be accorded the Executive Department, particularly the President
of the Republic, in respect of negotiations over the terms of FTAAs, particularly when it comes to the government share of
financial benefits from FTAAs.”

“Verily, under the doctrine of separation of powers and due respect for co-equal and coordinate branches of government, this
Court must restrain itself from intruding into policy matters and must allow the President and Congress maximum discretion in
using the resources of our country and in securing the assistance of foreign groups to eradicate the grinding poverty of our
people and answer their cry or viable employment opportunities in this country.”

“Let the development of the mining industry be the responsibility of the political branches of the government. And let not this
Court interfere inordinately and unnecessarily.”

- sir pointed out: although the issue is very technical, we must all go back to the basic issue of whether or not Congress, DENR
and the President acted within jurisdiction. The issue is always about jurisdiction.

> and in this case, the court found that they did act within their respective jurisdictions.

Vous aimerez peut-être aussi